Another Test Problem for Integer Programming       

      

Jsun Yui Wong

The computer program listed below seeks to solve the following test problem in Parsopoulos and Vrahatis [85, Test Problem 4]:

minimize         f = (9 * X(1) ^ 2 + 2 * X(2) ^ 2 – 11) ^ 2 + (3 * X(1) + 4 * X(2) ^ 2 – 7) ^ 2

with solution at x*=(1, 1) and f(x*)=0, Parsopoulos and Vrahatis [85, Test Problem 4].

  .   

0 REM DEFDBL A-Z

1 REM   DEFINT K

2 DIM B(99), N(99), A(2002), H(99), L(99), U(99), X(2002), D(111), P(111), PS(33), J44(2002), J(99), AA(99), HR(32), HHR(32), LHS(44), PLHS(44), LB(22), UB(22), PX(22), CC(20), RR(20), WW(20), AL(50), SW(50), SV(50), C2(22), C3(22), C4(22), C5(22)

12 FOR JJJJ = -32000 TO 32000 STEP .01

    13 RANDOMIZE JJJJ

    16 M = -1D+37

    19 FOR J44 = 1 TO 2

        21 A(J44) = -10 + FIX(RND * 21)

    22 NEXT J44

    128 FOR I = 1 TO 100

        129 FOR KKQQ = 1 TO 2

            130 X(KKQQ) = A(KKQQ)

        131 NEXT KKQQ

        135 FOR IPP = 1 TO FIX(1 + RND ^ 5 * 5)

            143 J = 1 + FIX(RND * 4)

            154 IF RND < .5 THEN GOTO 156 ELSE GOTO 162

            156 R = (1 – RND * 2) * A(J)

            158 X(J) = A(J) + (RND ^ (RND * 30)) * R

            161 GOTO 165

            162 REM    IF RND < .16666 THEN X(J) = A(J) – FIX(1 + RND * 2.3) ELSE IF RND < .33333 THEN X(J) = A(J) + FIX(1 + RND * 2.3) ELSE IF RND < .5 THEN X(J) = A(J) – FIX(1 + RND * 20.3) ELSE IF RND < .66666 THEN X(J) = A(J) + FIX(1 + RND * 20.3) ELSE IF RND < .83333 THEN X(J) = A(J) – FIX(1 + RND * 200.3) ELSE X(J) = A(J) + FIX(1 + RND * 200.3)

            164 IF RND < .5 THEN X(J) = A(J) – FIX(1 + RND * .3) ELSE X(J) = A(J) + FIX(1 + RND * .3)

        165 NEXT IPP

        166 FOR J44 = 1 TO 2

            167 X(J44) = INT(X(J44))

        168 NEXT J44

        182 FOR J44 = 1 TO 2

            183 IF X(J44) < -100 THEN 1670

            184 IF X(J44) > 100 THEN 1670

        185 NEXT J44

        1020 P = -(9 * X(1) ^ 2 + 2 * X(2) ^ 2 – 11) ^ 2 – (3 * X(1) + 4 * X(2) ^ 2 – 7) ^ 2

        1111 IF P <= M THEN 1670

        1420 M = P

        1444 FOR KLX = 1 TO 2

            1455 A(KLX) = X(KLX)

        1456 NEXT KLX

        1557 GOTO 128

    1670 NEXT I

    1890 REM IF M < 30665 THEN 1999

    1926 PRINT A(1), A(2), M, JJJJ

1999 NEXT JJJJ

This BASIC computer program was run with qb64v1000-win [120].  The complete output of one run through JJJJ =  -31999.84 is shown below:

1      -1      0      -32000

-1      1      -36      -31999.99

-1      1      -36      -31999.98

-1      1      -36      -31999.97

1      -1      0      -31999.96

-1      1      -36      -31999.95

1      -1      0      -31999.94

-1      -1      -36      -31999.93

-1      1      -36      -31999.92

-1      -1      -36      -31999.91

-1      -1      -36      -31999.9

1      -1      0      -31999.89

-1      1      -36      -31999.88

-1      1      -36      -31999.87

-1      -1      -36      -31999.86

-1      1      -36      -31999.85

1      1      0       -31999.84

Two distinct solutions are shown above at x*=(1, -1) and x*=(1, 1).

Above there is no rounding by hand; it is just straight copying by hand from the monitor screen.

The system properties of the computer system used include Intel Core 2 Duo CPU   E8400 @ 3.00GHz  3.00GHz, 4.00GB of RAM, and qb64v1000-win [120].  The wall-clock time (not CPU time) for obtaining the output through JJJJ = -31999.84 was 2 seconds, counting from “Starting program…”.  One can compare the computational results above with those in Parsopoulos and Vrahatis [85, Test Problem 4].  

Acknowledgement

I would like to acknowledge the encouragement of Roberta Clark and Tom Clark.

References

[1]  M. Abdollahi, A. Isazadeh, D. Abdollahi (2013), Imperialist competitive algorithm for solving systems of nonlinear equations, Computers and Mathematics with Applications, vol. 65, number 12, pp. 1894-1908, 2013.  (One can directly read this on Google.)

[2] Siby Abraham, Sugata Sanyal, Mukund Sanglikar (2010), Particle Swarm Optimisation Based Diophantine Equation Solver,  Int. J. of Bio-Inspired Computation, 2 (2), 100-114, 2010. 

[3] Siby Abraham, Sugata Sanyal, Mukund Sangrikar (2013), A Connectionist Network Approach to Find Numerical Solutions of Diophantine Equations, Int. J. of Engg. Science and Mgmt., Vol. III, Issue 1, January-June 2013.

[4] Andre R. S. Amaral (2006), On the Exact Solution of a Facility Layout Problem. European Journal of Operational Research 173 (2006), pp. 508-518.

[5] Andre R. S. Amaral (2008), An Exact Approach to the One-Dimensional Facility Layout Problem. Operations Research, Vol. 56, No. 4 (July-August, 2008), pp. 1026-1033.

[6] Andre R. S. Amaral (2011), Optimal Solutions for the Double Row Layout Problem. Optimization Letters, DOI 10.1007/s11590-011-0426-8, published on line 30 November 2011, Springer-Verlag 2011.

[7] Andre R. S. Amaral (2012), The Corridor Allocation Problem. Computers and Operations Research 39 (2012), pp. 3325-3330.

[8]  George Anescu (2016), Gradual and cumulative improvements to the classical differential evolution scheme through experiments, Analele  Universitatii de Vest, Timisoara, Seria Mathematica–Informatica, Vol. LIV, 2, (2016), 13-35.   (One can directly read this on Google.)

[9] Oscar Augusto, Bennis Fouad, Stephane Caro (2012). A new method for decision making in multi-objective optimization problems. Pesquisa Operacional, Sociedade Brasileira de Pesquisa Operacional, 2012 32 (3), pp.331-369.

[10] Miguel F. Anjos, Anthony Vannelli, Computing Globally Optimal Solutions for Single-Row Layout Problems Using Semidefinite Programming and Cutting Planes. INFORMS Journal on Computing, Vol. 20, No. 4, Fall 2008, pp. 611-617.

[11] David L. Applegate, Robert E. Bixby, Vasek Chvatal, William J. Cook, The Traveling Salesman Problem: A Computational Study. Princeton and Oxford: Princeton University Press, 2006.

[12] Ritu Arora, S. R. Arora (2015). A cutting plane approach for multi-objective integer indefinite quadratic programming problem: OPSEARCH of the Operational Research Society of India (April-June 2015), 52(2):367-381.

[13]   H. Bernau (1990).  Active constraint strategies in optimization.  Geographical data inversion methods and applications, pp. 15-31.

[14] Victor Blanco, Justo Puerto (2011). Some algebraic methods for solving multiobjective polynomial integer programs, Journal of Symbolic Computation, 46 (2011), pp. 511-533.

[15] Jerome Bracken, Garth P. McCormick, Selected Applications of Nonlinear Programming. New York: John Wiley and Sons, Inc., 1968.

[16] Neima Brauner, Moredechai Shacham, Michael Cutlip (winter 1996)  Computational Results – How Reliable Are They? Chemical Engineering Educatiobn, 30, 1,  pp. 20-25 (1996).  (One can read this via Google.)

[17] Richard L. Burden, Douglas J. Faires, Annette M. Burden, Numerical Analysis, Tenth Edition, 2016, Cengage Learning.

[18] R. C. Carlson and G. L. Nemhauser, Scheduling To Minimize Interaction Cost. Operations Research, Vol. 14, No. 1 (Jan. – Feb., 1966), pp. 52-58.

[19] Ta-Cheng Chen (2006). IAs based approach for reliability redundany allocation problems. Applied Mathematics and Computation 182 (2006) 1556-1567.

[20]  Ward Cheney, David Kincaid (2008), Numerical Mathematics and Computing, Sixth Edition, 2008, Thomsom Brooks/Coles.

[21] Leandro dos Santos Coelho (2009), Self-Organizing Migrating Strategies Applied to Reliability-Redundany Optimization of Systems. IEEE Transactions on Reliability, Vol. 58, No. 3, 2009 September, pp. 501-519.

[22]  A. J. Conejo, L. Baringo (2018), Power System Operations, Springer International Publishing, 2018.

[23] William Conley (1981). Optimization: A Simplified Approach. Published 1981 by Petrocelli Books in New York.

[24] Lino Costa, Pedro (2001). Evolutionary algorithms approach to the solution of mixed integer non-linear programming problems. Computers and Chemical Engineering, Vol. 25, pp. 257-266, 2001.

[25] Michael B. Cutlip, Mordechai Shacham (1998), POLYMATH verssion 4.1 user-friendly numerical analysis programs.  http://www.polymath-software.com.

[26] George B. Dantzig, Discrete-Variable Extremum Problems. Operations Research, Vol. 5, No. 2 (Apr., 1957), pp. 266-277.

[27] Pintu Das, Tapan Kumar Roy (2014). Multi-objective geometric programming and its application in gravel box problem. Journal of global research in computer science volume 5. no.7, July 2014. http://www.jgrcs.info

[28] Kalyanmoy Deb, Amrit Pratap, Subrajyoti Moitra (2000). Mechanical component design for multi objectives using elitist non-dominated sorting GA. Proceedings of the Parallel Probl.Solv. Nat. PPSN VI Conference, Paris, France, pp. 859-868 (2000). (Please see pp. 1-10 in Technical Report No. 200002 via Google search.)

[29] Kusum Deep, Krishna Pratap Singh, M. L. Kansal, C. Mohan (2009), A real coded genetic algorithm for solving integer and mixed integer optimization problems. Applied Mathematics and Computation 212 (2009) 505-518.

[30] Wassila Drici, Mustapha Moulai (2019): An exact method for solving multi-objective integer indefinite quadratic programs, Optimization Methods and Software.

[31] R. J. Duffin, E. L. Peterson, C. Zener (1967), Geometric Programming. John Wiley, New York (1967).

[32]  Simge Ekiz, Pakize Erdogmus, Busra Ozgur (2017 November), Solving constrained  optimization problems with sine-cosine algorithm, Periodicals of Engineering and Natural Sciences, vol. 5, no. 3, November 2017, pp.378~386.  http://pen.ius.edu.ba.  

[33] C. A. Floudas, A. R. Ciric (1989), Strategies for Overcoming Uncertainties in Heat Exchanger Network Synthesis. Computers and Chemical Engineering, Vol 13, No. 10, pp. 1133-1152, 1989.

[34] C. A. Floudas, A. Aggarwal, A. R. Ciric (1989), Global Optimum Search for Nonconvex NLP and MINLP Problems. Computers and Chemical Engineering, Vol 13, No. 10, pp. 1117-1132, 1989.

[35] C. A. Floudas, A. Aggarwal, A. R. Ciric (1989), Global Optimum Search for Nonconvex NLP and MINLP Problems. Computers and Chemical Engineering, Vol 13, No. 10, pp. 1117-1132, 1989.

[36] C. A. Floudas, P. M. Pardalos, A Collection of Test Problems for Constrained Global Optimization Algorithms. Springer-Verlag, 1990.  

[37] C. A. Floudas(1995),  Nonlinear and mixed-integer optimization:  fundamentals and applications.  Oxford University Press, 1995.

[38] Diptesh Ghosh, Ravi Kothari, Population Heuristics for the Corridor Allocation Problem, W.P. No. 2012-09-02, September 2012. Retrieved on September 14 2012 from Google search.

[39]  Crina Grosan, Ajith Abraham  (2008),    A New Approach for Solving Nonlinear Equations Systems.  IEEE Transactions on Systems, Man and Cybernetics–Part A:  Systems and Humans, Volume 38, Number 3, May 2008, pp. 698-714.

[40]  Crina Grosan, Ajith Abraham  (2008), Multiple Solutions for a System pf Nonlinear Equations. International Journal of Innovative Computing, Information and Control, Volume x, Number x, x 2008.

[41]  GUO Chuang-xin,  HU Jia-sheng, YE Bin, CAO Yi-jia (2004), Swarm intelligence for mixed-variable design optimization, J. of Zhejiang University SCIENCE 2004 5(7):851-860.  One can directly read this on Google.

[42] Neha Gupta, Irfan Ali, Abdul Bari (2013). An optimal chance constraint multivariate stratified sampling design using auxiliary infotmation. Journal of Mathematical Modelling and Algorithms, January 2013.

[43] R. Gupta, R. Malhotra (1995). Multi-criteria integer linear fractional programming problem, Optimization, 35:4, 373-389.

[44] M. Hashish, M. P. duPlessis (1979). Prediction equations relating high velocity jet cutting performance to stand-off-distance and multipasses. Transactions of ASME: Journal of Engineering for Industry 101 (1979) 311-318.

[45]  Abdel-Rahman Hedar, Masao Fukushima (2006)  Derivative-free simulated annealing method for constrained continuous optimization, J. of Global Optimization, January 2006.   (One can directly view this on Google.)

[46]  www – optima.amp.i.kyoto – u.ac.jp/member/student/hedar/Hedar_fil…

[47]  Frederick S. Hillier, Gerald J. Liberman (1990), Introduction to Mathematical Programming, McGraw-Hill Publishing Company, 1990.

[48] Willi Hock, Klaus Schittkowski, Test Examples for Nonlinear Programming Codes. Berlin: Springer-Verlag, 1981.

[49] Sana Iftekhar, M. J. Ahsan, Qazi Mazhar Ali (2015). An optimum multivariate stratified sampling design. Research Journal of Mathematical and Statistical Sciences, vol. 3(1),10-14, January (2015).

[50] Philipp Hungerlaender (April 2012), Single-Row Equidistant Facility Layout as a Special Case of Single-Row Facility Layout. Retrieved from http://www.optimization-online.org./DB_HTML/2012/04/3432.html

[51] Golam Reza Jahanshahloo, Farhad Hosseinzadeh, Nagi Shoja, Ghasem Tohidi (2003) A method for solvong 0-1 multiple objective liner programming problem using DEA. Journal of the Operations Research Society of Japan (2003), 46 (2): 189-202. http://www.orsj.or.jp/~archive/pdf/e_mag/Vol.46_02_189.pdf

[52] Ekta Jain, Kalpana Dahiya, Vanita Verma (2018): Integer quadratic fractional programming problems with bounded variables, Annals of Operations Research (October 2018) 269: pp. 269-295.

[53] N. K. Jain, V. K. Jain, K. Deb (2007). Optimization of process parameters of mechanical type advanced machining processes using genetic algorithms. International Journal of Machine Tools and Manufacture 47 (2007), 900-919.

[54] Michael Junger, Thomas M. Liebling, Dennis Naddef, George L. Nemhauser, William R. Pulleybank, Gerhart Reinelt, Giovanni Rinaldi, Lawrence A. Wolsey–Editors, 50 Years of Integer Programming 1958-2008. Berlin: Springer, 2010.

[55] Adhe Kania, Kuntjoro Adji Sidarto (2016). Solving mixed integer nonlinear programming problems using spiral dynamics optimization algorithm. AIP Conference Proceedings 1716, 020004 (2016).  29, 2016.  https://doi:org/10.1063/1.4942987.

[56] Reena Kapoor, S. R. Arora (2006). Linearization of a 0-1 quadratic fractional programming problem: OPSEARCH of the Operational Research Society of India (2006), 43(2):190-207.

[57] M. G. M. Khan, E. A. Khan, M. J. Ahsan (2003). An optimal multivariate stratified sampling design using dynami programming. Australian and New Zealand Journal of Statistics, vol. 45, no. 1, 2003, pp. 107-113.

[58] M. G. M. Khan, T. Maiti, M. J. Ahsan (2010). An optimal multivariate stratified sampling design using auxiliary information: an integer solution using goal programming approach. Journal of Official Statistics, vol. 26, no. 4, 2010, pp. 695-708.

[59] A. H. Land, A. G. Doig, An Automatic Method of Solving Discrete Programming Problems. Econometrica, Vol. 28, No. 3 (Jul., 1960), pp. 497-520.

[60]  E. L. Lawler, M. D. Bell, A Method for Solving Discrete Optimization Problems. Operations Research, Vol. 14, No. 6 (Nov.-Dec., 1966), pp. 1098-1112.

[61]  Sven Leyffer (December 1993)  Deterministic Methods for Mixed Integer Nonlinear Progrmming, PhD Thesis, Department of Mathematics and Computer Science, University of Dundee, Dundee, December 1993.  (One can directly read this on Google.)

[62]  S. Leyffer, J. Linderoth, J. Luedtke, A. Miller, T. Munson (2009), Applications and algorithms for mixed integer nonlinear programming, J. of Physics: Conference Series 180 (2009) 012014, IOP Publishing.  (One can directly read this on Google.)

[63]  Yugui Li, Yanxu Wei, Yantao Chu (2015), Research on solving systems of nonlinear equations based on improved PSO, Mathematical Problems in Engineering, volume 2015, article ID 727218, 13 pages, Hindawi Publishing Corporration.   One can directly read this article on Google.

[64]  F. H. F. Liu, C. C. Huang, Y. L. Yen (2000): Using DEA to obtain efficient solutions for multi-objective 0-1 linear programs. European Journal of Operational Research 126 (2000) 51-68.

[65] Gia-Shi Liu (2006), A combination method for reliability-redundancy optimization, Engineering Optimization, 38:04, 485-499.

[66] Yubao Liu, Guihe Qin (2014), A hybrid TS-DE algorithm for reliability redundancy optimization problem, Journal of Computers, 9, No. 9, September 2014, pp. 2050-2057.

[67] Rein Luus (1975). Optimization of System Reliability by a New Nonlinear Integer Programming Procedure. IEEE Transactions on Reliability, Vol. R-24, No. 1, April 1975, pp. 14-16.

[68] Milos Madic, Miroslav Radovanovic (2014). Optimization of machining processes using pattern search algorithm. International Journal of Industrial Engineering Computations 5 (2014) 223-234. Homepage: http://www.GrowingScience.com/ijiec

[69] F. Masedu, M Angelozzi (2008). Modelling optimum fraction assignment in the 4X100 m relay race by integer linear programming. Italian Journal of Sports Sciences, Anno 13, No. 1, 2008, pp. 74-77.

[70] Mohamed Arezki Mellal, Enrico Zio (2016). A Guided Stochastic Fractal Search Approach for System Reliability Optimization. Reliability Engineering and System Safety 152 (2016) 213-227.

[71] Mohamed Arezki Mellal, Edward J. Williams (2016). Parameter optimization of advanced machining processes using cuckoo optimization algorithm and hoopla heuristic. Journal of Intelligent Manufacturing (2016) 27 (5): 927-942.

[72] Mohamed Arezki Mellal, Edward J. Williams (2018). Large-scale reliability-redundancy allocation optimization problem using three soft computing methods. In Mangey Ram, Editor, in Modeling and simulation based analysis in reliability engineering. Published July 2018, CRC Press.

[73] Microsoft Corp., BASIC, Second Edition (May 1982), Version 1.10. Boca Raton, Florida: IBM Corp., Personal Computer, P. O. Box 1328-C, Boca Raton, Florida 33432, 1981.

[74]  Joaquin Moreno, Miguel Lopez, Raquel Martinez (2018)  A new method for solving  nonlinear systems of equations that is based on functional iterations, Open Physics 2018; 16: 605-630.    

[75] Yuji Nakagawa, Mitsunori Hikita, Hiroshi Kamada (1984). Surrogate Constraints for Reliability Optimization Problems with Multiple Constraints. IEEE Transactions on Reliability, Vol. R-33, No. 4, October 1984, pp. 301-305.

[76] Subhash C. Narula, H. Roland Weistroffer (1989). A flexible method for nonlinear criteria decisionmaking problems. Ieee Transactions on Systems, Man and Cybernetics, vol. 19 , no. 4, July/August 1989, pp. 883-887.

[77] C. E. Nugent, T. E. Vollmann, J. Ruml (1968), An Experimental Comparison of Techniques for the Assignment of Facilities to Locations, Operations Research 16 (1968), pp. 150-173.

[78] A. K. Ojha, K. K. Biswal (2010). Multi-objective geometric programming problem with weighted mean method. (IJCSIS) International Journal of Computer Science and Information Security, vol. 7, no. 2, pp. 82-86, 2010.

[79] Rashmi Ranjan Ota, jayanta kumar dASH, A. K. Ojha (2014). A hybrid optimization techniques for solving non-linear multi-objective optimization problem. amo-advanced modelling and optimization, volume 16,number 1, 2014.

[80] Rashmi Ranjan Ota, A. K. Ojha (2015). A comparative study on optimization techniques for solving multi-objective geometric programming problems. Applied mathematical sciences, vol. 9, 2015, no. 22, 1077-1085. http://sites.google.com/site/ijcsis/

[81] R. R. Ota, J. C. Pati, A. K. Ojha (2019). Geometric programming technique to optimize power distribution system. OPSEARCH of the Operational Research Society of India (2019), 56, pp. 282-299.

[82] OPTI Toolbox, Mixed Integer Nonlinear Program (MINLP). https://www.inverseproblem.co.nz/OPTI/index.php/Probs/MINLP

[83] Panos Y. Papalambros, Douglass J. Wilde, Principles of Optimal Design, Second Edition. Cambridge University Press, 2000.

[84]  K. E. Parsopoulos, M. N. Vrahatis, Particle swarm optimization method for constrained optimization.  Intelligent Technologies–Theory…, 2002. 

(One can directly read this on Google Scholar.)

[85]  K. E. Parsopoulos, M. N. Vrahatis, Particle swarm optimization for integer programming,  IEEE 2002. 

(One can directly read this article on Google Scholar.)

[86]  O. Perez, I. Amaya, R. Correa (2013), Numerical solution of certain exponential and nonlinear diophantine systems of equations by using a discrete particles swarm optimization algorithm.  Applied Mathematics and Computation, Volume 225, 1 December, 2013, pp. 737-746.

[87] Yashpal Singh Raghav, Irfan Ali, Abdul Bari (2014) Multi-Objective Nonlinear Programming Problem Approach in Multivariate Stratified Sample Surveys in the Case of the Non-Response, Journal of Statitiscal Computation ans Simulation 84:1, 22-36, doi:10.1080/00949655.2012.692370.

[88] R. V. Rao, P. J. Pawar, J. P. Davim (2010). Parameter optimization of ultrasonic machining process using nontraditional optimization algorihms. Materials and Manufacturing Processes, 25 (10),1120-1130.

[89]  W. Rheinboldt,  Some Nonlinear Testproblems.  One can google this 10-page paper.

[90]  John Rice, Numerical Methods, Software, and Analysis, Second Edition, 1993, Academic Press.

[91] H. S. Ryoo, N. V. Sahinidis (1995), Global Optimization of Nonconvex NLPs and MINLPs with Applications in Process Design. Computers and Chemical Engineering, Vol. 19, No. 5, pp. 551-566, 1995.

[92] Ali Sadollah, Hadi Eskandar, Joong Hoon Kim (2015). Water cycle algorithm for solvinfg constrained multi-objective optimization problems. Applied Soft Computing 27 (2015) 279-298.

[93] Shafiullah, Irfan Ali, Abdul Bari (2015). Fuzzy geometric programming approach in multi-objective multivariate stratified sample surveys in presence of non-respnse, International Journal of Operations Research, Vol. 12, No. 2, pp. 021-035 (2015).

[94] Mordechai Shacham (1986). Numerical solution of constrained nonlinear algebraic equations, International Journal for Numerical Methods in Engineering, Vol. 23, pp. 1455-1481 (1986).  (One can read this via Google.)

[95] M. Shacham,  R. S. H. Mah  (1978).  A Newton Type Linearization Method for Solution of Nonlinear Equations, Computers and Chemical Engineering, Vol. 2, pp. 64-66 (1978).  (One can read this via Google.)

[96]  Yi Shang (1997),  Global search methods for solving nonlinear optimization problems, Ph. D. Thesis, University of Illinois at Urbana-Champaign, 1997.

[97] Vikas Sharma (2012). Multiobjective integer nonlinear fractional programming problem: A cutting plane approach, OPSEARCH of the Operational Research Society of India (April-June 2012), 49(2):133-153.

[98] Vikas Sharma, Kalpana Dahiya, Vanita Verma (2017). A ranking algorithm for bi-objective quadratic fractional integer programming problems, Optimization, 66:11, 1913-1929.

[99]  K. A. Sidarto, Adhe Kania (2015),  Finding all solutions of systems of nonlinear equations using spiral dynamics inspired optimization with clustering,

Journal of Advanced Computational Intelligence and Intelligent Informatics, pp. 697-708, vol. 19, number 5, 2015.  (One can directly read this via Google.)

[100] G. Stephanopoulos, A. W. Westerberg, The Use of Hestenes’ Method of Multipliers to Resolve Dual Gaps in Engineering System Optimization. Journal of Optimization Theory and Applications, Vol.15, No. 3, pp. 285-309, 1975.

[101]  Sonja Surjanovic, Derek Bingham (2013), Virtual library of simulation experiments: test functions and datasets.  Simon Fraser University, Burnaby, BC, Canada, 2013.  (One can directly read this on Google.)

[102] Hardi Tambunan, Herman Mawengkang (2016). Solving Mixed Integer Non-Linear Programming Using Active Constraint. Global Journal of Pure and Applied Mathematics, Volume 12, Number 6 (2016), pp. 5267-5281. http://www.ripublication.com/gjpam.htm

[103] Mohamed Tawhid, Vimal Savsani (2018). Epsilon-constraint heat transfer search (epsilon-HTS) algorithm for solvinfg multi-objective engineering design problems. journal of computational design and engineering 5 (2018) 104-119.

[104] Frank A. Tillman, Ching-Lai Hwang, Way Kuo (1977). Determining Component Reliability and Redundancy for Optimun System Reliability. IEEE Transactions on Reliability, Vol. R-26, No. 3, Augusr 1977, pp. 162-165.

[105] Rahul Varshney, Srikant Gupta, Irfan Ali (2017). An optimum multivariate-multiobjective stratified samplinr design: fuzzy programming approach. Pakistan Journal of Statistics and Operations Research, pp. 829-855, December 2017

[106] V. Verma, H. C. Bakhshi, M. C. Puri (1990) Ranking in integer linear fractional programming problems ZOR – Methods and Models of Operations Research (1990) 34:325-334.

[107] Tawan Wasanapradit, Nalinee Mukdasanit, Nachol Chaiyaratana, Thongchai Srinophakun (2011). Solving mixed-integer nonlinear programming problems using improved genetic algorithms. Korean Joutnal of Chemical Engineering 28 (1):32-40 January 2011.

[108] Rahul Varshney, Mradula (2019 May 25). Optimum allocation in multivariate stratified sampling design in the presence of nonresponse with Gamma cost function, Journal of Statistical Computation and Simulation (2019) 89:13, pp. 2454-2467.

[109] Rahul Varshney, Najmussehar, M. J. Ahsan (2012). An optimum multivariate stratified double sampling design in presence of non-response, Optimization Letters (2012) 6: pp. 993-1008.

[110] Rahul Varshney, M. G. M. Khan, Ummatul Fatima, M. J. Ahsan (2015). Integer compromise allocation in multivariate stratified surveys, Annals of Operations Research (2015) 226:659-668.

[111] Rahul Varshney, Srikant Gupta, Irfan Ali (2017). An optimum multivariate-multiobjective stratified sampling design: fuzzy programming approach. Pakistan Journal of Statistics and Operations Research, pp. 829-855, December 2017

[112] V. Verma, H. C. Bakhshi, M. C. Puri (1990) Ranking in integer linear fractional programming problems ZOR – Methods and Models of Operations Research (1990) 34:325-334.

[113] Tawan Wasanapradit, Nalinee Mukdasanit, Nachol Chaiyaratana, Thongchai Srinophakun (2011). Solving mixed-integer nonlinear programming problems using improved genetic algorithms. Korean Joutnal of Chemical Engineering 28 (1):32-40 January 2011.

[114]  Eric W. Weisstein, “Diophantine Equation–8th Powers.”  https://mathworld.wolfram.com/DiophantineEquation8thPowers.html.

[115]  Eric W. Weisstein, “Euler’s Sum of 0wers Conjecture.”  https://mathworld.wolfram.com/EulersSumofPowersConjecture.html.

[116]  Eric W. Weisstein, “Diophantine Equation–5th Powers.”  https://mathworld.wolfram.com/DiophantineEquation5thPowers.html.

[117]  Eric W. Weisstein, “Diophantine Equation–10th Powers.”  https://mathworld.wolfram.com/DiophantineEquation10thPowers.html.

[118]  Eric W. Weisstein, “Diophantine Equation–9th Powers.”  https://mathworld.wolfram.com/DiophantineEquation9thPowers.html.

[119]  Rick Wicklin (2018), Solving a system of nonlinear equations with SAS.  blogs.sas.com>iml>2018/02/28.  One can directly read this on Google.

[120] Wikipedia, QB64, https://en.wikipedia.org/wiki/QB64.

[121] Wikipedia, Test functions for optimization, https://en.wikipedia.org/wiki/Test_functions_for_optimization.

[122] Jsun Yui Wong (2014, March 13).   The Domino Method Applied to a System of Four Simultaneous Nonlinear Equations.  Retrieved from http://myblogsubstance.typepad.com/substance/2014/03.

[123] Zhongkai Xu, Way Kuo, Hsin-Hui Lin (1990). Optimization Limits in Improving System Reliability. IEEE Transactions on Reliability, Vol. 39, No. 1, 1990 April, pp. 51-60.

A Test Problem for Integer Programming    

    

Jsun Yui Wong

The computer program listed below seeks to solve the following test problem in Parsopoulos and Vrahatis [85, Test Problem 6]:

minimize         f =              2 * X(1) ^ 2 + 3 * X(2) ^ 2 + 4 * X(1) * X(2) – 6 * X(1) – 3 * X(2) ,

with solution at x*=(2, -1) and f*= -6, Parsopoulos and Vrahatis [85, Test Problem 6].

0 REM DEFDBL A-Z

1 REM   DEFINT K

2 DIM B(99), N(99), A(2002), H(99), L(99), U(99), X(2002), D(111), P(111), PS(33), J44(2002), J(99), AA(99), HR(32), HHR(32), LHS(44), PLHS(44), LB(22), UB(22), PX(22), CC(20), RR(20), WW(20), AL(50), SW(50), SV(50), C2(22), C3(22), C4(22), C5(22)

12 FOR JJJJ = -32000 TO 32000 STEP .01

    13 RANDOMIZE JJJJ

    16 M = -1D+37

    19 FOR J44 = 1 TO 2

        21 A(J44) = -10 + FIX(RND * 21)

    22 NEXT J44

    128 FOR I = 1 TO 100

        129 FOR KKQQ = 1 TO 2

            130 X(KKQQ) = A(KKQQ)

        131 NEXT KKQQ

        135 FOR IPP = 1 TO FIX(1 + RND ^ 5 * 5)

            143 J = 1 + FIX(RND * 2)

            154 IF RND < .5 THEN GOTO 156 ELSE GOTO 162

            156 R = (1 – RND * 2) * A(J)

            158 X(J) = A(J) + (RND ^ (RND * 30)) * R

            161 GOTO 165

            162 REM    IF RND < .16666 THEN X(J) = A(J) – FIX(1 + RND * 2.3) ELSE IF RND < .33333 THEN X(J) = A(J) + FIX(1 + RND * 2.3) ELSE IF RND < .5 THEN X(J) = A(J) – FIX(1 + RND * 20.3) ELSE IF RND < .66666 THEN X(J) = A(J) + FIX(1 + RND * 20.3) ELSE IF RND < .83333 THEN X(J) = A(J) – FIX(1 + RND * 200.3) ELSE X(J) = A(J) + FIX(1 + RND * 200.3)

            164 IF RND < .5 THEN X(J) = A(J) – FIX(1 + RND * .3) ELSE X(J) = A(J) + FIX(1 + RND * .3)

        165 NEXT IPP

        166 FOR J44 = 1 TO 2

            167 X(J44) = INT(X(J44))

        168 NEXT J44

        182 FOR J44 = 1 TO 2

            183 IF X(J44) < -100 THEN 1670

            184 IF X(J44) > 100 THEN 1670

        185 NEXT J44

        1016 P = -2 * X(1) ^ 2 – 3 * X(2) ^ 2 – 4 * X(1) * X(2) + 6 * X(1) + 3 * X(2)

        1111 IF P <= M THEN 1670

        1420 M = P

        1444 FOR KLX = 1 TO 2

            1455 A(KLX) = X(KLX)

        1456 NEXT KLX

        1557 GOTO 128

    1670 NEXT I

    1890 REM IF M < 30665 THEN 1999

    1926 PRINT A(1), A(2), M, JJJJ

1999 NEXT JJJJ

This BASIC computer program was run with qb64v1000-win [120].  The complete output of one run through JJJJ =  -31999.9 is shown below:

2  -1  6  -32000

2  -1  6  -31999.99

2  -1  6  -31999.98

2  -1  6  -31999.97

3  -1  6  -31999.96

0  1  0    -31999.95

0  1  0    -31999.94

2  -1  6  -31999.93

4  -2  6  -31999.92

6  -4  0  -31999.91

3  -1  6  -31999.9

Three distinct solutions are shown above at x*=(2, -1), x*=(3, -1), and x*=(4, -2).

Above there is no rounding by hand; it is just straight copying by hand from the monitor screen.

The system properties of the computer system used include Intel Core 2 Duo CPU   E8400 @ 3.00GHz  3.00GHz, 4.00GB of RAM, and qb64v1000-win [120].  The wall-clock time (not CPU time) for obtaining the output through JJJJ = -31999.9 was 2 seconds, counting from “Starting program…”.  One can compare the computational results above with those in Parsopoulos and Vrahatis [85, Test Problem 6].  

Acknowledgement

I would like to acknowledge the encouragement of Roberta Clark and Tom Clark.

References

[1]  M. Abdollahi, A. Isazadeh, D. Abdollahi (2013), Imperialist competitive algorithm for solving systems of nonlinear equations, Computers and Mathematics with Applications, vol. 65, number 12, pp. 1894-1908, 2013.  (One can directly read this on Google.)

[2] Siby Abraham, Sugata Sanyal, Mukund Sanglikar (2010), Particle Swarm Optimisation Based Diophantine Equation Solver,  Int. J. of Bio-Inspired Computation, 2 (2), 100-114, 2010. 

[3] Siby Abraham, Sugata Sanyal, Mukund Sangrikar (2013), A Connectionist Network Approach to Find Numerical Solutions of Diophantine Equations, Int. J. of Engg. Science and Mgmt., Vol. III, Issue 1, January-June 2013.

[4] Andre R. S. Amaral (2006), On the Exact Solution of a Facility Layout Problem. European Journal of Operational Research 173 (2006), pp. 508-518.

[5] Andre R. S. Amaral (2008), An Exact Approach to the One-Dimensional Facility Layout Problem. Operations Research, Vol. 56, No. 4 (July-August, 2008), pp. 1026-1033.

[6] Andre R. S. Amaral (2011), Optimal Solutions for the Double Row Layout Problem. Optimization Letters, DOI 10.1007/s11590-011-0426-8, published on line 30 November 2011, Springer-Verlag 2011.

[7] Andre R. S. Amaral (2012), The Corridor Allocation Problem. Computers and Operations Research 39 (2012), pp. 3325-3330.

[8]  George Anescu (2016), Gradual and cumulative improvements to the classical differential evolution scheme through experiments, Analele  Universitatii de Vest, Timisoara, Seria Mathematica–Informatica, Vol. LIV, 2, (2016), 13-35.   (One can directly read this on Google.)

[9] Oscar Augusto, Bennis Fouad, Stephane Caro (2012). A new method for decision making in multi-objective optimization problems. Pesquisa Operacional, Sociedade Brasileira de Pesquisa Operacional, 2012 32 (3), pp.331-369.

[10] Miguel F. Anjos, Anthony Vannelli, Computing Globally Optimal Solutions for Single-Row Layout Problems Using Semidefinite Programming and Cutting Planes. INFORMS Journal on Computing, Vol. 20, No. 4, Fall 2008, pp. 611-617.

[11] David L. Applegate, Robert E. Bixby, Vasek Chvatal, William J. Cook, The Traveling Salesman Problem: A Computational Study. Princeton and Oxford: Princeton University Press, 2006.

[12] Ritu Arora, S. R. Arora (2015). A cutting plane approach for multi-objective integer indefinite quadratic programming problem: OPSEARCH of the Operational Research Society of India (April-June 2015), 52(2):367-381.

[13]   H. Bernau (1990).  Active constraint strategies in optimization.  Geographical data inversion methods and applications, pp. 15-31.

[14] Victor Blanco, Justo Puerto (2011). Some algebraic methods for solving multiobjective polynomial integer programs, Journal of Symbolic Computation, 46 (2011), pp. 511-533.

[15] Jerome Bracken, Garth P. McCormick, Selected Applications of Nonlinear Programming. New York: John Wiley and Sons, Inc., 1968.

[16] Neima Brauner, Moredechai Shacham, Michael Cutlip (winter 1996)  Computational Results – How Reliable Are They? Chemical Engineering Educatiobn, 30, 1,  pp. 20-25 (1996).  (One can read this via Google.)

[17] Richard L. Burden, Douglas J. Faires, Annette M. Burden, Numerical Analysis, Tenth Edition, 2016, Cengage Learning.

[18] R. C. Carlson and G. L. Nemhauser, Scheduling To Minimize Interaction Cost. Operations Research, Vol. 14, No. 1 (Jan. – Feb., 1966), pp. 52-58.

[19] Ta-Cheng Chen (2006). IAs based approach for reliability redundany allocation problems. Applied Mathematics and Computation 182 (2006) 1556-1567.

[20]  Ward Cheney, David Kincaid (2008), Numerical Mathematics and Computing, Sixth Edition, 2008, Thomsom Brooks/Coles.

[21] Leandro dos Santos Coelho (2009), Self-Organizing Migrating Strategies Applied to Reliability-Redundany Optimization of Systems. IEEE Transactions on Reliability, Vol. 58, No. 3, 2009 September, pp. 501-519.

[22]  A. J. Conejo, L. Baringo (2018), Power System Operations, Springer International Publishing, 2018.

[23] William Conley (1981). Optimization: A Simplified Approach. Published 1981 by Petrocelli Books in New York.

[24] Lino Costa, Pedro (2001). Evolutionary algorithms approach to the solution of mixed integer non-linear programming problems. Computers and Chemical Engineering, Vol. 25, pp. 257-266, 2001.

[25] Michael B. Cutlip, Mordechai Shacham (1998), POLYMATH verssion 4.1 user-friendly numerical analysis programs.  http://www.polymath-software.com.

[26] George B. Dantzig, Discrete-Variable Extremum Problems. Operations Research, Vol. 5, No. 2 (Apr., 1957), pp. 266-277.

[27] Pintu Das, Tapan Kumar Roy (2014). Multi-objective geometric programming and its application in gravel box problem. Journal of global research in computer science volume 5. no.7, July 2014. http://www.jgrcs.info

[28] Kalyanmoy Deb, Amrit Pratap, Subrajyoti Moitra (2000). Mechanical component design for multi objectives using elitist non-dominated sorting GA. Proceedings of the Parallel Probl.Solv. Nat. PPSN VI Conference, Paris, France, pp. 859-868 (2000). (Please see pp. 1-10 in Technical Report No. 200002 via Google search.)

[29] Kusum Deep, Krishna Pratap Singh, M. L. Kansal, C. Mohan (2009), A real coded genetic algorithm for solving integer and mixed integer optimization problems. Applied Mathematics and Computation 212 (2009) 505-518.

[30] Wassila Drici, Mustapha Moulai (2019): An exact method for solving multi-objective integer indefinite quadratic programs, Optimization Methods and Software.

[31] R. J. Duffin, E. L. Peterson, C. Zener (1967), Geometric Programming. John Wiley, New York (1967).

[32]  Simge Ekiz, Pakize Erdogmus, Busra Ozgur (2017 November), Solving constrained  optimization problems with sine-cosine algorithm, Periodicals of Engineering and Natural Sciences, vol. 5, no. 3, November 2017, pp.378~386.  http://pen.ius.edu.ba.  

[33] C. A. Floudas, A. R. Ciric (1989), Strategies for Overcoming Uncertainties in Heat Exchanger Network Synthesis. Computers and Chemical Engineering, Vol 13, No. 10, pp. 1133-1152, 1989.

[34] C. A. Floudas, A. Aggarwal, A. R. Ciric (1989), Global Optimum Search for Nonconvex NLP and MINLP Problems. Computers and Chemical Engineering, Vol 13, No. 10, pp. 1117-1132, 1989.

[35] C. A. Floudas, A. Aggarwal, A. R. Ciric (1989), Global Optimum Search for Nonconvex NLP and MINLP Problems. Computers and Chemical Engineering, Vol 13, No. 10, pp. 1117-1132, 1989.

[36] C. A. Floudas, P. M. Pardalos, A Collection of Test Problems for Constrained Global Optimization Algorithms. Springer-Verlag, 1990.  

[37] C. A. Floudas(1995),  Nonlinear and mixed-integer optimization:  fundamentals and applications.  Oxford University Press, 1995.

[38] Diptesh Ghosh, Ravi Kothari, Population Heuristics for the Corridor Allocation Problem, W.P. No. 2012-09-02, September 2012. Retrieved on September 14 2012 from Google search.

[39]  Crina Grosan, Ajith Abraham  (2008),    A New Approach for Solving Nonlinear Equations Systems.  IEEE Transactions on Systems, Man and Cybernetics–Part A:  Systems and Humans, Volume 38, Number 3, May 2008, pp. 698-714.

[40]  Crina Grosan, Ajith Abraham  (2008), Multiple Solutions for a System pf Nonlinear Equations. International Journal of Innovative Computing, Information and Control, Volume x, Number x, x 2008.

[41]  GUO Chuang-xin,  HU Jia-sheng, YE Bin, CAO Yi-jia (2004), Swarm intelligence for mixed-variable design optimization, J. of Zhejiang University SCIENCE 2004 5(7):851-860.  One can directly read this on Google.

[42] Neha Gupta, Irfan Ali, Abdul Bari (2013). An optimal chance constraint multivariate stratified sampling design using auxiliary infotmation. Journal of Mathematical Modelling and Algorithms, January 2013.

[43] R. Gupta, R. Malhotra (1995). Multi-criteria integer linear fractional programming problem, Optimization, 35:4, 373-389.

[44] M. Hashish, M. P. duPlessis (1979). Prediction equations relating high velocity jet cutting performance to stand-off-distance and multipasses. Transactions of ASME: Journal of Engineering for Industry 101 (1979) 311-318.

[45]  Abdel-Rahman Hedar, Masao Fukushima (2006)  Derivative-free simulated annealing method for constrained continuous optimization, J. of Global Optimization, January 2006.   (One can directly view this on Google.)

[46]  www – optima.amp.i.kyoto – u.ac.jp/member/student/hedar/Hedar_fil…

[47]  Frederick S. Hillier, Gerald J. Liberman (1990), Introduction to Mathematical Programming, McGraw-Hill Publishing Company, 1990.

[48] Willi Hock, Klaus Schittkowski, Test Examples for Nonlinear Programming Codes. Berlin: Springer-Verlag, 1981.

[49] Sana Iftekhar, M. J. Ahsan, Qazi Mazhar Ali (2015). An optimum multivariate stratified sampling design. Research Journal of Mathematical and Statistical Sciences, vol. 3(1),10-14, January (2015).

[50] Philipp Hungerlaender (April 2012), Single-Row Equidistant Facility Layout as a Special Case of Single-Row Facility Layout. Retrieved from http://www.optimization-online.org./DB_HTML/2012/04/3432.html

[51] Golam Reza Jahanshahloo, Farhad Hosseinzadeh, Nagi Shoja, Ghasem Tohidi (2003) A method for solvong 0-1 multiple objective liner programming problem using DEA. Journal of the Operations Research Society of Japan (2003), 46 (2): 189-202. http://www.orsj.or.jp/~archive/pdf/e_mag/Vol.46_02_189.pdf

[52] Ekta Jain, Kalpana Dahiya, Vanita Verma (2018): Integer quadratic fractional programming problems with bounded variables, Annals of Operations Research (October 2018) 269: pp. 269-295.

[53] N. K. Jain, V. K. Jain, K. Deb (2007). Optimization of process parameters of mechanical type advanced machining processes using genetic algorithms. International Journal of Machine Tools and Manufacture 47 (2007), 900-919.

[54] Michael Junger, Thomas M. Liebling, Dennis Naddef, George L. Nemhauser, William R. Pulleybank, Gerhart Reinelt, Giovanni Rinaldi, Lawrence A. Wolsey–Editors, 50 Years of Integer Programming 1958-2008. Berlin: Springer, 2010.

[55] Adhe Kania, Kuntjoro Adji Sidarto (2016). Solving mixed integer nonlinear programming problems using spiral dynamics optimization algorithm. AIP Conference Proceedings 1716, 020004 (2016).  29, 2016.  https://doi:org/10.1063/1.4942987.

[56] Reena Kapoor, S. R. Arora (2006). Linearization of a 0-1 quadratic fractional programming problem: OPSEARCH of the Operational Research Society of India (2006), 43(2):190-207.

[57] M. G. M. Khan, E. A. Khan, M. J. Ahsan (2003). An optimal multivariate stratified sampling design using dynami programming. Australian and New Zealand Journal of Statistics, vol. 45, no. 1, 2003, pp. 107-113.

[58] M. G. M. Khan, T. Maiti, M. J. Ahsan (2010). An optimal multivariate stratified sampling design using auxiliary information: an integer solution using goal programming approach. Journal of Official Statistics, vol. 26, no. 4, 2010, pp. 695-708.

[59] A. H. Land, A. G. Doig, An Automatic Method of Solving Discrete Programming Problems. Econometrica, Vol. 28, No. 3 (Jul., 1960), pp. 497-520.

[60]  E. L. Lawler, M. D. Bell, A Method for Solving Discrete Optimization Problems. Operations Research, Vol. 14, No. 6 (Nov.-Dec., 1966), pp. 1098-1112.

[61]  Sven Leyffer (December 1993)  Deterministic Methods for Mixed Integer Nonlinear Progrmming, PhD Thesis, Department of Mathematics and Computer Science, University of Dundee, Dundee, December 1993.  (One can directly read this on Google.)

[62]  S. Leyffer, J. Linderoth, J. Luedtke, A. Miller, T. Munson (2009), Applications and algorithms for mixed integer nonlinear programming, J. of Physics: Conference Series 180 (2009) 012014, IOP Publishing.  (One can directly read this on Google.)

[63]  Yugui Li, Yanxu Wei, Yantao Chu (2015), Research on solving systems of nonlinear equations based on improved PSO, Mathematical Problems in Engineering, volume 2015, article ID 727218, 13 pages, Hindawi Publishing Corporration.   One can directly read this article on Google.

[64]  F. H. F. Liu, C. C. Huang, Y. L. Yen (2000): Using DEA to obtain efficient solutions for multi-objective 0-1 linear programs. European Journal of Operational Research 126 (2000) 51-68.

[65] Gia-Shi Liu (2006), A combination method for reliability-redundancy optimization, Engineering Optimization, 38:04, 485-499.

[66] Yubao Liu, Guihe Qin (2014), A hybrid TS-DE algorithm for reliability redundancy optimization problem, Journal of Computers, 9, No. 9, September 2014, pp. 2050-2057.

[67] Rein Luus (1975). Optimization of System Reliability by a New Nonlinear Integer Programming Procedure. IEEE Transactions on Reliability, Vol. R-24, No. 1, April 1975, pp. 14-16.

[68] Milos Madic, Miroslav Radovanovic (2014). Optimization of machining processes using pattern search algorithm. International Journal of Industrial Engineering Computations 5 (2014) 223-234. Homepage: http://www.GrowingScience.com/ijiec

[69] F. Masedu, M Angelozzi (2008). Modelling optimum fraction assignment in the 4X100 m relay race by integer linear programming. Italian Journal of Sports Sciences, Anno 13, No. 1, 2008, pp. 74-77.

[70] Mohamed Arezki Mellal, Enrico Zio (2016). A Guided Stochastic Fractal Search Approach for System Reliability Optimization. Reliability Engineering and System Safety 152 (2016) 213-227.

[71] Mohamed Arezki Mellal, Edward J. Williams (2016). Parameter optimization of advanced machining processes using cuckoo optimization algorithm and hoopla heuristic. Journal of Intelligent Manufacturing (2016) 27 (5): 927-942.

[72] Mohamed Arezki Mellal, Edward J. Williams (2018). Large-scale reliability-redundancy allocation optimization problem using three soft computing methods. In Mangey Ram, Editor, in Modeling and simulation based analysis in reliability engineering. Published July 2018, CRC Press.

[73] Microsoft Corp., BASIC, Second Edition (May 1982), Version 1.10. Boca Raton, Florida: IBM Corp., Personal Computer, P. O. Box 1328-C, Boca Raton, Florida 33432, 1981.

[74]  Joaquin Moreno, Miguel Lopez, Raquel Martinez (2018)  A new method for solving  nonlinear systems of equations that is based on functional iterations, Open Physics 2018; 16: 605-630.    

[75] Yuji Nakagawa, Mitsunori Hikita, Hiroshi Kamada (1984). Surrogate Constraints for Reliability Optimization Problems with Multiple Constraints. IEEE Transactions on Reliability, Vol. R-33, No. 4, October 1984, pp. 301-305.

[76] Subhash C. Narula, H. Roland Weistroffer (1989). A flexible method for nonlinear criteria decisionmaking problems. Ieee Transactions on Systems, Man and Cybernetics, vol. 19 , no. 4, July/August 1989, pp. 883-887.

[77] C. E. Nugent, T. E. Vollmann, J. Ruml (1968), An Experimental Comparison of Techniques for the Assignment of Facilities to Locations, Operations Research 16 (1968), pp. 150-173.

[78] A. K. Ojha, K. K. Biswal (2010). Multi-objective geometric programming problem with weighted mean method. (IJCSIS) International Journal of Computer Science and Information Security, vol. 7, no. 2, pp. 82-86, 2010.

[79] Rashmi Ranjan Ota, jayanta kumar dASH, A. K. Ojha (2014). A hybrid optimization techniques for solving non-linear multi-objective optimization problem. amo-advanced modelling and optimization, volume 16,number 1, 2014.

[80] Rashmi Ranjan Ota, A. K. Ojha (2015). A comparative study on optimization techniques for solving multi-objective geometric programming problems. Applied mathematical sciences, vol. 9, 2015, no. 22, 1077-1085. http://sites.google.com/site/ijcsis/

[81] R. R. Ota, J. C. Pati, A. K. Ojha (2019). Geometric programming technique to optimize power distribution system. OPSEARCH of the Operational Research Society of India (2019), 56, pp. 282-299.

[82] OPTI Toolbox, Mixed Integer Nonlinear Program (MINLP). https://www.inverseproblem.co.nz/OPTI/index.php/Probs/MINLP

[83] Panos Y. Papalambros, Douglass J. Wilde, Principles of Optimal Design, Second Edition. Cambridge University Press, 2000.

[84]  K. E. Parsopoulos, M. N. Vrahatis, Particle swarm optimization method for constrained optimization.  Intelligent Technologies–Theory…, 2002. 

(One can directly read this on Google Scholar.)

[85]  K. E. Parsopoulos, M. N. Vrahatis, Particle swarm optimization for integer programming, 2002 IEEE.  

(One can directly read this article on Google Scholar.)

[86]  O. Perez, I. Amaya, R. Correa (2013), Numerical solution of certain exponential and nonlinear diophantine systems of equations by using a discrete particles swarm optimization algorithm.  Applied Mathematics and Computation, Volume 225, 1 December, 2013, pp. 737-746.

[87] Yashpal Singh Raghav, Irfan Ali, Abdul Bari (2014) Multi-Objective Nonlinear Programming Problem Approach in Multivariate Stratified Sample Surveys in the Case of the Non-Response, Journal of Statitiscal Computation ans Simulation 84:1, 22-36, doi:10.1080/00949655.2012.692370.

[88] R. V. Rao, P. J. Pawar, J. P. Davim (2010). Parameter optimization of ultrasonic machining process using nontraditional optimization algorihms. Materials and Manufacturing Processes, 25 (10),1120-1130.

[89]  W. Rheinboldt,  Some Nonlinear Testproblems.  One can google this 10-page paper.

[90]  John Rice, Numerical Methods, Software, and Analysis, Second Edition, 1993, Academic Press.

[91] H. S. Ryoo, N. V. Sahinidis (1995), Global Optimization of Nonconvex NLPs and MINLPs with Applications in Process Design. Computers and Chemical Engineering, Vol. 19, No. 5, pp. 551-566, 1995.

[92] Ali Sadollah, Hadi Eskandar, Joong Hoon Kim (2015). Water cycle algorithm for solvinfg constrained multi-objective optimization problems. Applied Soft Computing 27 (2015) 279-298.

[93] Shafiullah, Irfan Ali, Abdul Bari (2015). Fuzzy geometric programming approach in multi-objective multivariate stratified sample surveys in presence of non-respnse, International Journal of Operations Research, Vol. 12, No. 2, pp. 021-035 (2015).

[94] Mordechai Shacham (1986). Numerical solution of constrained nonlinear algebraic equations, International Journal for Numerical Methods in Engineering, Vol. 23, pp. 1455-1481 (1986).  (One can read this via Google.)

[95] M. Shacham,  R. S. H. Mah  (1978).  A Newton Type Linearization Method for Solution of Nonlinear Equations, Computers and Chemical Engineering, Vol. 2, pp. 64-66 (1978).  (One can read this via Google.)

[96]  Yi Shang (1997),  Global search methods for solving nonlinear optimization problems, Ph. D. Thesis, University of Illinois at Urbana-Champaign, 1997.

[97] Vikas Sharma (2012). Multiobjective integer nonlinear fractional programming problem: A cutting plane approach, OPSEARCH of the Operational Research Society of India (April-June 2012), 49(2):133-153.

[98] Vikas Sharma, Kalpana Dahiya, Vanita Verma (2017). A ranking algorithm for bi-objective quadratic fractional integer programming problems, Optimization, 66:11, 1913-1929.

[99]  K. A. Sidarto, Adhe Kania (2015),  Finding all solutions of systems of nonlinear equations using spiral dynamics inspired optimization with clustering,

Journal of Advanced Computational Intelligence and Intelligent Informatics, pp. 697-708, vol. 19, number 5, 2015.  (One can directly read this via Google.)

[100] G. Stephanopoulos, A. W. Westerberg, The Use of Hestenes’ Method of Multipliers to Resolve Dual Gaps in Engineering System Optimization. Journal of Optimization Theory and Applications, Vol.15, No. 3, pp. 285-309, 1975.

[101]  Sonja Surjanovic, Derek Bingham (2013), Virtual library of simulation experiments: test functions and datasets.  Simon Fraser University, Burnaby, BC, Canada, 2013.  (One can directly read this on Google.)

[102] Hardi Tambunan, Herman Mawengkang (2016). Solving Mixed Integer Non-Linear Programming Using Active Constraint. Global Journal of Pure and Applied Mathematics, Volume 12, Number 6 (2016), pp. 5267-5281. http://www.ripublication.com/gjpam.htm

[103] Mohamed Tawhid, Vimal Savsani (2018). Epsilon-constraint heat transfer search (epsilon-HTS) algorithm for solvinfg multi-objective engineering design problems. journal of computational design and engineering 5 (2018) 104-119.

[104] Frank A. Tillman, Ching-Lai Hwang, Way Kuo (1977). Determining Component Reliability and Redundancy for Optimun System Reliability. IEEE Transactions on Reliability, Vol. R-26, No. 3, Augusr 1977, pp. 162-165.

[105] Rahul Varshney, Srikant Gupta, Irfan Ali (2017). An optimum multivariate-multiobjective stratified samplinr design: fuzzy programming approach. Pakistan Journal of Statistics and Operations Research, pp. 829-855, December 2017

[106] V. Verma, H. C. Bakhshi, M. C. Puri (1990) Ranking in integer linear fractional programming problems ZOR – Methods and Models of Operations Research (1990) 34:325-334.

[107] Tawan Wasanapradit, Nalinee Mukdasanit, Nachol Chaiyaratana, Thongchai Srinophakun (2011). Solving mixed-integer nonlinear programming problems using improved genetic algorithms. Korean Joutnal of Chemical Engineering 28 (1):32-40 January 2011.

[108] Rahul Varshney, Mradula (2019 May 25). Optimum allocation in multivariate stratified sampling design in the presence of nonresponse with Gamma cost function, Journal of Statistical Computation and Simulation (2019) 89:13, pp. 2454-2467.

[109] Rahul Varshney, Najmussehar, M. J. Ahsan (2012). An optimum multivariate stratified double sampling design in presence of non-response, Optimization Letters (2012) 6: pp. 993-1008.

[110] Rahul Varshney, M. G. M. Khan, Ummatul Fatima, M. J. Ahsan (2015). Integer compromise allocation in multivariate stratified surveys, Annals of Operations Research (2015) 226:659-668.

[111] Rahul Varshney, Srikant Gupta, Irfan Ali (2017). An optimum multivariate-multiobjective stratified sampling design: fuzzy programming approach. Pakistan Journal of Statistics and Operations Research, pp. 829-855, December 2017

[112] V. Verma, H. C. Bakhshi, M. C. Puri (1990) Ranking in integer linear fractional programming problems ZOR – Methods and Models of Operations Research (1990) 34:325-334.

[113] Tawan Wasanapradit, Nalinee Mukdasanit, Nachol Chaiyaratana, Thongchai Srinophakun (2011). Solving mixed-integer nonlinear programming problems using improved genetic algorithms. Korean Joutnal of Chemical Engineering 28 (1):32-40 January 2011.

[114]  Eric W. Weisstein, “Diophantine Equation–8th Powers.”  https://mathworld.wolfram.com/DiophantineEquation8thPowers.html.

[115]  Eric W. Weisstein, “Euler’s Sum of 0wers Conjecture.”  https://mathworld.wolfram.com/EulersSumofPowersConjecture.html.

[116]  Eric W. Weisstein, “Diophantine Equation–5th Powers.”  https://mathworld.wolfram.com/DiophantineEquation5thPowers.html.

[117]  Eric W. Weisstein, “Diophantine Equation–10th Powers.”  https://mathworld.wolfram.com/DiophantineEquation10thPowers.html.

[118]  Eric W. Weisstein, “Diophantine Equation–9th Powers.”  https://mathworld.wolfram.com/DiophantineEquation9thPowers.html.

[119]  Rick Wicklin (2018), Solving a system of nonlinear equations with SAS.  blogs.sas.com>iml>2018/02/28.  One can directly read this on Google.

[120] Wikipedia, QB64, https://en.wikipedia.org/wiki/QB64.

[121] Wikipedia, Test functions for optimization, https://en.wikipedia.org/wiki/Test_functions_for_optimization.

[122] Jsun Yui Wong (2014, March 13).   The Domino Method Applied to a System of Four Simultaneous Nonlinear Equations.  Retrieved from http://myblogsubstance.typepad.com/substance/2014/03.

[123] Zhongkai Xu, Way Kuo, Hsin-Hui Lin (1990). Optimization Limits in Improving System Reliability. IEEE Transactions on Reliability, Vol. 39, No. 1, 1990 April, pp. 51-60.

Another Test Problem for Constrained Optimization       

    

Jsun Yui Wong

The computer program listed below seeks to solve the following test problem in Parsopoulos and Vrahatis [84, Test Problem 1]:

minimize         f =     (X(1) -2)^2+(X(2)-1)^2  

subject  to  

          X(1)=2 * X(2) – 1,

          X(1)^2/4+  X(2)^2 – 1  <= 0.                                                                                                   

The best known solution is f*= 1.3934651, [84].

0 REM DEFDBL A-Z

1 REM   DEFINT K

2 DIM B(99), N(99), A(2002), H(99), L(99), U(99), X(2002), D(111), P(111), PS(33), J44(2002), J(99), AA(99), HR(32), HHR(32), LHS(44), PLHS(44), LB(22), UB(22), PX(22), CC(20), RR(20), WW(20), AL(50), SW(50), SV(50), C2(22), C3(22), C4(22), C5(22)

12 FOR JJJJ = -32000 TO 32000 STEP .01

    13 RANDOMIZE JJJJ

    16 M = -1D+37

      19 FOR J44 = 1 TO 2

        21 A(J44) = -5 + RND * 10

    22 NEXT J44

    128 FOR I = 0 TO 10000

        129 FOR KKQQ = 1 TO 2

            130 X(KKQQ) = A(KKQQ)

        131 NEXT KKQQ

        135 FOR IPP = 1 TO FIX(1 + RND ^ 5 * 5)

            143 j = 1 + FIX(RND * 5)

            154 IF RND < .5 THEN GOTO 156 ELSE GOTO 162

            156 R = (1 – RND * 2) * A(j)

            158 X(j) = A(j) + (RND ^ (RND * 30)) * R

            161 GOTO 165

            162 REM IF RND<.16666 THEN X(j)= …….

            164 IF RND < .5 THEN X(j) = A(j) – FIX(1 + RND * .3) ELSE X(j) = A(j) + FIX(1 + RND * .3)

        165 NEXT IPP

        166 X(1) = 2 * X(2) – 1

        167 IF X(1) ^ 2 / 4 + X(2) ^ 2 – 1 > 0 THEN 1670

        1014 P = -(X(1) – 2) ^ 2 – (X(2) – 1) ^ 2

        1111 IF P <= M THEN 1670

        1420 M = P

        1444 FOR KLX = 1 TO 2

            1455 A(KLX) = X(KLX)

        1456 NEXT KLX

        1557 GOTO 128

    1670 NEXT I

    1890 REM   IF M < 30665 THEN 1999

    1926 PRINT A(1), A(2), M, JJJJ

1999 NEXT JJJJ

This BASIC computer program was run with qb64v1000-win [120].  The complete output of one run through JJJJ =  -31999.96 is shown below:

.8228756      .9114378      -1.393465      -32000

.8228756      .9114378      -1.393465      -31999.99

.8228756      .9114378      -1.393465      -31999.98

.8228756      .9114378      -1.393465      -31999.97

.8228756      .9114378      -1.393465      -31999.96

Above there is no rounding by hand; it is just straight copying by hand from the monitor screen.

The system properties of the computer system used include Intel Core 2 Duo CPU   E8400 @ 3.00GHz  3.00GHz, 4.00GB of RAM, and qb64v1000-win [120].  The wall-clock time (not CPU time) for obtaining the output through JJJJ = -31999.96 was 2 seconds, counting from “Starting program…”.  One can compare the computational results above with those in Parsopoulos and Vrahatis [84, Test Problem 1, Table 1].  

Acknowledgement

I would like to acknowledge the encouragement of Roberta Clark and Tom Clark.

References

[1]  M. Abdollahi, A. Isazadeh, D. Abdollahi (2013), Imperialist competitive algorithm for solving systems of nonlinear equations, Computers and Mathematics with Applications, vol. 65, number 12, pp. 1894-1908, 2013.  (One can directly read this on Google.)

[2] Siby Abraham, Sugata Sanyal, Mukund Sanglikar (2010), Particle Swarm Optimisation Based Diophantine Equation Solver,  Int. J. of Bio-Inspired Computation, 2 (2), 100-114, 2010. 

[3] Siby Abraham, Sugata Sanyal, Mukund Sangrikar (2013), A Connectionist Network Approach to Find Numerical Solutions of Diophantine Equations, Int. J. of Engg. Science and Mgmt., Vol. III, Issue 1, January-June 2013.

[4] Andre R. S. Amaral (2006), On the Exact Solution of a Facility Layout Problem. European Journal of Operational Research 173 (2006), pp. 508-518.

[5] Andre R. S. Amaral (2008), An Exact Approach to the One-Dimensional Facility Layout Problem. Operations Research, Vol. 56, No. 4 (July-August, 2008), pp. 1026-1033.

[6] Andre R. S. Amaral (2011), Optimal Solutions for the Double Row Layout Problem. Optimization Letters, DOI 10.1007/s11590-011-0426-8, published on line 30 November 2011, Springer-Verlag 2011.

[7] Andre R. S. Amaral (2012), The Corridor Allocation Problem. Computers and Operations Research 39 (2012), pp. 3325-3330.

[8]  George Anescu (2016), Gradual and cumulative improvements to the classical differential evolution scheme through experiments, Analele  Universitatii de Vest, Timisoara, Seria Mathematica–Informatica, Vol. LIV, 2, (2016), 13-35.   (One can directly read this on Google.)

[9] Oscar Augusto, Bennis Fouad, Stephane Caro (2012). A new method for decision making in multi-objective optimization problems. Pesquisa Operacional, Sociedade Brasileira de Pesquisa Operacional, 2012 32 (3), pp.331-369.

[10] Miguel F. Anjos, Anthony Vannelli, Computing Globally Optimal Solutions for Single-Row Layout Problems Using Semidefinite Programming and Cutting Planes. INFORMS Journal on Computing, Vol. 20, No. 4, Fall 2008, pp. 611-617.

[11] David L. Applegate, Robert E. Bixby, Vasek Chvatal, William J. Cook, The Traveling Salesman Problem: A Computational Study. Princeton and Oxford: Princeton University Press, 2006.

[12] Ritu Arora, S. R. Arora (2015). A cutting plane approach for multi-objective integer indefinite quadratic programming problem: OPSEARCH of the Operational Research Society of India (April-June 2015), 52(2):367-381.

[13]   H. Bernau (1990).  Active constraint strategies in optimization.  Geographical data inversion methods and applications, pp. 15-31.

[14] Victor Blanco, Justo Puerto (2011). Some algebraic methods for solving multiobjective polynomial integer programs, Journal of Symbolic Computation, 46 (2011), pp. 511-533.

[15] Jerome Bracken, Garth P. McCormick, Selected Applications of Nonlinear Programming. New York: John Wiley and Sons, Inc., 1968.

[16] Neima Brauner, Moredechai Shacham, Michael Cutlip (winter 1996)  Computational Results – How Reliable Are They? Chemical Engineering Educatiobn, 30, 1,  pp. 20-25 (1996).  (One can read this via Google.)

[17] Richard L. Burden, Douglas J. Faires, Annette M. Burden, Numerical Analysis, Tenth Edition, 2016, Cengage Learning.

[18] R. C. Carlson and G. L. Nemhauser, Scheduling To Minimize Interaction Cost. Operations Research, Vol. 14, No. 1 (Jan. – Feb., 1966), pp. 52-58.

[19] Ta-Cheng Chen (2006). IAs based approach for reliability redundany allocation problems. Applied Mathematics and Computation 182 (2006) 1556-1567.

[20]  Ward Cheney, David Kincaid (2008), Numerical Mathematics and Computing, Sixth Edition, 2008, Thomsom Brooks/Coles.

[21] Leandro dos Santos Coelho (2009), Self-Organizing Migrating Strategies Applied to Reliability-Redundany Optimization of Systems. IEEE Transactions on Reliability, Vol. 58, No. 3, 2009 September, pp. 501-519.

[22]  A. J. Conejo, L. Baringo (2018), Power System Operations, Springer International Publishing, 2018.

[23] William Conley (1981). Optimization: A Simplified Approach. Published 1981 by Petrocelli Books in New York.

[24] Lino Costa, Pedro (2001). Evolutionary algorithms approach to the solution of mixed integer non-linear programming problems. Computers and Chemical Engineering, Vol. 25, pp. 257-266, 2001.

[25] Michael B. Cutlip, Mordechai Shacham (1998), POLYMATH verssion 4.1 user-friendly numerical analysis programs.  http://www.polymath-software.com.

[26] George B. Dantzig, Discrete-Variable Extremum Problems. Operations Research, Vol. 5, No. 2 (Apr., 1957), pp. 266-277.

[27] Pintu Das, Tapan Kumar Roy (2014). Multi-objective geometric programming and its application in gravel box problem. Journal of global research in computer science volume 5. no.7, July 2014. http://www.jgrcs.info

[28] Kalyanmoy Deb, Amrit Pratap, Subrajyoti Moitra (2000). Mechanical component design for multi objectives using elitist non-dominated sorting GA. Proceedings of the Parallel Probl.Solv. Nat. PPSN VI Conference, Paris, France, pp. 859-868 (2000). (Please see pp. 1-10 in Technical Report No. 200002 via Google search.)

[29] Kusum Deep, Krishna Pratap Singh, M. L. Kansal, C. Mohan (2009), A real coded genetic algorithm for solving integer and mixed integer optimization problems. Applied Mathematics and Computation 212 (2009) 505-518.

[30] Wassila Drici, Mustapha Moulai (2019): An exact method for solving multi-objective integer indefinite quadratic programs, Optimization Methods and Software.

[31] R. J. Duffin, E. L. Peterson, C. Zener (1967), Geometric Programming. John Wiley, New York (1967).

[32]  Simge Ekiz, Pakize Erdogmus, Busra Ozgur (2017 November), Solving constrained  optimization problems with sine-cosine algorithm, Periodicals of Engineering and Natural Sciences, vol. 5, no. 3, November 2017, pp.378~386.  http://pen.ius.edu.ba.  

[33] C. A. Floudas, A. R. Ciric (1989), Strategies for Overcoming Uncertainties in Heat Exchanger Network Synthesis. Computers and Chemical Engineering, Vol 13, No. 10, pp. 1133-1152, 1989.

[34] C. A. Floudas, A. Aggarwal, A. R. Ciric (1989), Global Optimum Search for Nonconvex NLP and MINLP Problems. Computers and Chemical Engineering, Vol 13, No. 10, pp. 1117-1132, 1989.

[35] C. A. Floudas, A. Aggarwal, A. R. Ciric (1989), Global Optimum Search for Nonconvex NLP and MINLP Problems. Computers and Chemical Engineering, Vol 13, No. 10, pp. 1117-1132, 1989.

[36] C. A. Floudas, P. M. Pardalos, A Collection of Test Problems for Constrained Global Optimization Algorithms. Springer-Verlag, 1990.  

[37] C. A. Floudas(1995),  Nonlinear and mixed-integer optimization:  fundamentals and applications.  Oxford University Press, 1995.

[38] Diptesh Ghosh, Ravi Kothari, Population Heuristics for the Corridor Allocation Problem, W.P. No. 2012-09-02, September 2012. Retrieved on September 14 2012 from Google search.

[39]  Crina Grosan, Ajith Abraham  (2008),    A New Approach for Solving Nonlinear Equations Systems.  IEEE Transactions on Systems, Man and Cybernetics–Part A:  Systems and Humans, Volume 38, Number 3, May 2008, pp. 698-714.

[40]  Crina Grosan, Ajith Abraham  (2008), Multiple Solutions for a System pf Nonlinear Equations. International Journal of Innovative Computing, Information and Control, Volume x, Number x, x 2008.

[41]  GUO Chuang-xin,  HU Jia-sheng, YE Bin, CAO Yi-jia (2004), Swarm intelligence for mixed-variable design optimization, J. of Zhejiang University SCIENCE 2004 5(7):851-860.  One can directly read this on Google.

[42] Neha Gupta, Irfan Ali, Abdul Bari (2013). An optimal chance constraint multivariate stratified sampling design using auxiliary infotmation. Journal of Mathematical Modelling and Algorithms, January 2013.

[43] R. Gupta, R. Malhotra (1995). Multi-criteria integer linear fractional programming problem, Optimization, 35:4, 373-389.

[44] M. Hashish, M. P. duPlessis (1979). Prediction equations relating high velocity jet cutting performance to stand-off-distance and multipasses. Transactions of ASME: Journal of Engineering for Industry 101 (1979) 311-318.

[45]  Abdel-Rahman Hedar, Masao Fukushima (2006)  Derivative-free simulated annealing method for constrained continuous optimization, J. of Global Optimization, January 2006.   (One can directly view this on Google.)

[46]  www – optima.amp.i.kyoto – u.ac.jp/member/student/hedar/Hedar_fil…

[47]  Frederick S. Hillier, Gerald J. Liberman (1990), Introduction to Mathematical Programming, McGraw-Hill Publishing Company, 1990.

[48] Willi Hock, Klaus Schittkowski, Test Examples for Nonlinear Programming Codes. Berlin: Springer-Verlag, 1981.

[49] Sana Iftekhar, M. J. Ahsan, Qazi Mazhar Ali (2015). An optimum multivariate stratified sampling design. Research Journal of Mathematical and Statistical Sciences, vol. 3(1),10-14, January (2015).

[50] Philipp Hungerlaender (April 2012), Single-Row Equidistant Facility Layout as a Special Case of Single-Row Facility Layout. Retrieved from http://www.optimization-online.org./DB_HTML/2012/04/3432.html

[51] Golam Reza Jahanshahloo, Farhad Hosseinzadeh, Nagi Shoja, Ghasem Tohidi (2003) A method for solvong 0-1 multiple objective liner programming problem using DEA. Journal of the Operations Research Society of Japan (2003), 46 (2): 189-202. http://www.orsj.or.jp/~archive/pdf/e_mag/Vol.46_02_189.pdf

[52] Ekta Jain, Kalpana Dahiya, Vanita Verma (2018): Integer quadratic fractional programming problems with bounded variables, Annals of Operations Research (October 2018) 269: pp. 269-295.

[53] N. K. Jain, V. K. Jain, K. Deb (2007). Optimization of process parameters of mechanical type advanced machining processes using genetic algorithms. International Journal of Machine Tools and Manufacture 47 (2007), 900-919.

[54] Michael Junger, Thomas M. Liebling, Dennis Naddef, George L. Nemhauser, William R. Pulleybank, Gerhart Reinelt, Giovanni Rinaldi, Lawrence A. Wolsey–Editors, 50 Years of Integer Programming 1958-2008. Berlin: Springer, 2010.

[55] Adhe Kania, Kuntjoro Adji Sidarto (2016). Solving mixed integer nonlinear programming problems using spiral dynamics optimization algorithm. AIP Conference Proceedings 1716, 020004 (2016).  29, 2016.  https://doi:org/10.1063/1.4942987.

[56] Reena Kapoor, S. R. Arora (2006). Linearization of a 0-1 quadratic fractional programming problem: OPSEARCH of the Operational Research Society of India (2006), 43(2):190-207.

[57] M. G. M. Khan, E. A. Khan, M. J. Ahsan (2003). An optimal multivariate stratified sampling design using dynami programming. Australian and New Zealand Journal of Statistics, vol. 45, no. 1, 2003, pp. 107-113.

[58] M. G. M. Khan, T. Maiti, M. J. Ahsan (2010). An optimal multivariate stratified sampling design using auxiliary information: an integer solution using goal programming approach. Journal of Official Statistics, vol. 26, no. 4, 2010, pp. 695-708.

[59] A. H. Land, A. G. Doig, An Automatic Method of Solving Discrete Programming Problems. Econometrica, Vol. 28, No. 3 (Jul., 1960), pp. 497-520.

[60]  E. L. Lawler, M. D. Bell, A Method for Solving Discrete Optimization Problems. Operations Research, Vol. 14, No. 6 (Nov.-Dec., 1966), pp. 1098-1112.

[61]  Sven Leyffer (December 1993)  Deterministic Methods for Mixed Integer Nonlinear Progrmming, PhD Thesis, Department of Mathematics and Computer Science, University of Dundee, Dundee, December 1993.  (One can directly read this on Google.)

[62]  S. Leyffer, J. Linderoth, J. Luedtke, A. Miller, T. Munson (2009), Applications and algorithms for mixed integer nonlinear programming, J. of Physics: Conference Series 180 (2009) 012014, IOP Publishing.  (One can directly read this on Google.)

[63]  Yugui Li, Yanxu Wei, Yantao Chu (2015), Research on solving systems of nonlinear equations based on improved PSO, Mathematical Problems in Engineering, volume 2015, article ID 727218, 13 pages, Hindawi Publishing Corporration.   One can directly read this article on Google.

[64]  F. H. F. Liu, C. C. Huang, Y. L. Yen (2000): Using DEA to obtain efficient solutions for multi-objective 0-1 linear programs. European Journal of Operational Research 126 (2000) 51-68.

[65] Gia-Shi Liu (2006), A combination method for reliability-redundancy optimization, Engineering Optimization, 38:04, 485-499.

[66] Yubao Liu, Guihe Qin (2014), A hybrid TS-DE algorithm for reliability redundancy optimization problem, Journal of Computers, 9, No. 9, September 2014, pp. 2050-2057.

[67] Rein Luus (1975). Optimization of System Reliability by a New Nonlinear Integer Programming Procedure. IEEE Transactions on Reliability, Vol. R-24, No. 1, April 1975, pp. 14-16.

[68] Milos Madic, Miroslav Radovanovic (2014). Optimization of machining processes using pattern search algorithm. International Journal of Industrial Engineering Computations 5 (2014) 223-234. Homepage: http://www.GrowingScience.com/ijiec

[69] F. Masedu, M Angelozzi (2008). Modelling optimum fraction assignment in the 4X100 m relay race by integer linear programming. Italian Journal of Sports Sciences, Anno 13, No. 1, 2008, pp. 74-77.

[70] Mohamed Arezki Mellal, Enrico Zio (2016). A Guided Stochastic Fractal Search Approach for System Reliability Optimization. Reliability Engineering and System Safety 152 (2016) 213-227.

[71] Mohamed Arezki Mellal, Edward J. Williams (2016). Parameter optimization of advanced machining processes using cuckoo optimization algorithm and hoopla heuristic. Journal of Intelligent Manufacturing (2016) 27 (5): 927-942.

[72] Mohamed Arezki Mellal, Edward J. Williams (2018). Large-scale reliability-redundancy allocation optimization problem using three soft computing methods. In Mangey Ram, Editor, in Modeling and simulation based analysis in reliability engineering. Published July 2018, CRC Press.

[73] Microsoft Corp., BASIC, Second Edition (May 1982), Version 1.10. Boca Raton, Florida: IBM Corp., Personal Computer, P. O. Box 1328-C, Boca Raton, Florida 33432, 1981.

[74]  Joaquin Moreno, Miguel Lopez, Raquel Martinez (2018)  A new method for solving  nonlinear systems of equations that is based on functional iterations, Open Physics 2018; 16: 605-630.    

[75] Yuji Nakagawa, Mitsunori Hikita, Hiroshi Kamada (1984). Surrogate Constraints for Reliability Optimization Problems with Multiple Constraints. IEEE Transactions on Reliability, Vol. R-33, No. 4, October 1984, pp. 301-305.

[76] Subhash C. Narula, H. Roland Weistroffer (1989). A flexible method for nonlinear criteria decisionmaking problems. Ieee Transactions on Systems, Man and Cybernetics, vol. 19 , no. 4, July/August 1989, pp. 883-887.

[77] C. E. Nugent, T. E. Vollmann, J. Ruml (1968), An Experimental Comparison of Techniques for the Assignment of Facilities to Locations, Operations Research 16 (1968), pp. 150-173.

[78] A. K. Ojha, K. K. Biswal (2010). Multi-objective geometric programming problem with weighted mean method. (IJCSIS) International Journal of Computer Science and Information Security, vol. 7, no. 2, pp. 82-86, 2010.

[79] Rashmi Ranjan Ota, jayanta kumar dASH, A. K. Ojha (2014). A hybrid optimization techniques for solving non-linear multi-objective optimization problem. amo-advanced modelling and optimization, volume 16,number 1, 2014.

[80] Rashmi Ranjan Ota, A. K. Ojha (2015). A comparative study on optimization techniques for solving multi-objective geometric programming problems. Applied mathematical sciences, vol. 9, 2015, no. 22, 1077-1085. http://sites.google.com/site/ijcsis/

[81] R. R. Ota, J. C. Pati, A. K. Ojha (2019). Geometric programming technique to optimize power distribution system. OPSEARCH of the Operational Research Society of India (2019), 56, pp. 282-299.

[82] OPTI Toolbox, Mixed Integer Nonlinear Program (MINLP). https://www.inverseproblem.co.nz/OPTI/index.php/Probs/MINLP

[83] Panos Y. Papalambros, Douglass J. Wilde, Principles of Optimal Design, Second Edition. Cambridge University Press, 2000.

[84]  K. E. Parsopoulos, M. N. Vrahatis, Particle swarm optimization method for constrained optimization.  Intelligent Technologies–Theory…, 2002. 

(One can directly read this on Google Scholar.)

[85]  O. Perez, I. Amaya, R. Correa (2013), Numerical solution of certain exponential and nonlinear diophantine systems of equations by using a discrete particles swarm optimization algorithm.  Applied Mathematics and Computation, Volume 225, 1 December, 2013, pp. 737-746.

[86] Ciara Pike-Burke. Multi-Objective Optimization. https://www.lancaster.ac.uk/pg/pikeburc/report1.pdf.

[87] Yashpal Singh Raghav, Irfan Ali, Abdul Bari (2014) Multi-Objective Nonlinear Programming Problem Approach in Multivariate Stratified Sample Surveys in the Case of the Non-Response, Journal of Statitiscal Computation ans Simulation 84:1, 22-36, doi:10.1080/00949655.2012.692370.

[88] R. V. Rao, P. J. Pawar, J. P. Davim (2010). Parameter optimization of ultrasonic machining process using nontraditional optimization algorihms. Materials and Manufacturing Processes, 25 (10),1120-1130.

[89]  W. Rheinboldt,  Some Nonlinear Testproblems.  One can google this 10-page paper.

[90]  John Rice, Numerical Methods, Software, and Analysis, Second Edition, 1993, Academic Press.

[91] H. S. Ryoo, N. V. Sahinidis (1995), Global Optimization of Nonconvex NLPs and MINLPs with Applications in Process Design. Computers and Chemical Engineering, Vol. 19, No. 5, pp. 551-566, 1995.

[92] Ali Sadollah, Hadi Eskandar, Joong Hoon Kim (2015). Water cycle algorithm for solvinfg constrained multi-objective optimization problems. Applied Soft Computing 27 (2015) 279-298.

[93] Shafiullah, Irfan Ali, Abdul Bari (2015). Fuzzy geometric programming approach in multi-objective multivariate stratified sample surveys in presence of non-respnse, International Journal of Operations Research, Vol. 12, No. 2, pp. 021-035 (2015).

[94] Mordechai Shacham (1986). Numerical solution of constrained nonlinear algebraic equations, International Journal for Numerical Methods in Engineering, Vol. 23, pp. 1455-1481 (1986).  (One can read this via Google.)

[95] M. Shacham,  R. S. H. Mah  (1978).  A Newton Type Linearization Method for Solution of Nonlinear Equations, Computers and Chemical Engineering, Vol. 2, pp. 64-66 (1978).  (One can read this via Google.)

[96]  Yi Shang (1997),  Global search methods for solving nonlinear optimization problems, Ph. D. Thesis, University of Illinois at Urbana-Champaign, 1997.

[97] Vikas Sharma (2012). Multiobjective integer nonlinear fractional programming problem: A cutting plane approach, OPSEARCH of the Operational Research Society of India (April-June 2012), 49(2):133-153.

[98] Vikas Sharma, Kalpana Dahiya, Vanita Verma (2017). A ranking algorithm for bi-objective quadratic fractional integer programming problems, Optimization, 66:11, 1913-1929.

[99]  K. A. Sidarto, Adhe Kania (2015),  Finding all solutions of systems of nonlinear equations using spiral dynamics inspired optimization with clustering,

Journal of Advanced Computational Intelligence and Intelligent Informatics, pp. 697-708, vol. 19, number 5, 2015.  (One can directly read this via Google.)

[100] G. Stephanopoulos, A. W. Westerberg, The Use of Hestenes’ Method of Multipliers to Resolve Dual Gaps in Engineering System Optimization. Journal of Optimization Theory and Applications, Vol.15, No. 3, pp. 285-309, 1975.

[101]  Sonja Surjanovic, Derek Bingham (2013), Virtual library of simulation experiments: test functions and datasets.  Simon Fraser University, Burnaby, BC, Canada, 2013.  (One can directly read this on Google.)

[102] Hardi Tambunan, Herman Mawengkang (2016). Solving Mixed Integer Non-Linear Programming Using Active Constraint. Global Journal of Pure and Applied Mathematics, Volume 12, Number 6 (2016), pp. 5267-5281. http://www.ripublication.com/gjpam.htm

[103] Mohamed Tawhid, Vimal Savsani (2018). Epsilon-constraint heat transfer search (epsilon-HTS) algorithm for solvinfg multi-objective engineering design problems. journal of computational design and engineering 5 (2018) 104-119.

[104] Frank A. Tillman, Ching-Lai Hwang, Way Kuo (1977). Determining Component Reliability and Redundancy for Optimun System Reliability. IEEE Transactions on Reliability, Vol. R-26, No. 3, Augusr 1977, pp. 162-165.

[105] Rahul Varshney, Srikant Gupta, Irfan Ali (2017). An optimum multivariate-multiobjective stratified samplinr design: fuzzy programming approach. Pakistan Journal of Statistics and Operations Research, pp. 829-855, December 2017

[106] V. Verma, H. C. Bakhshi, M. C. Puri (1990) Ranking in integer linear fractional programming problems ZOR – Methods and Models of Operations Research (1990) 34:325-334.

[107] Tawan Wasanapradit, Nalinee Mukdasanit, Nachol Chaiyaratana, Thongchai Srinophakun (2011). Solving mixed-integer nonlinear programming problems using improved genetic algorithms. Korean Joutnal of Chemical Engineering 28 (1):32-40 January 2011.

[108] Rahul Varshney, Mradula (2019 May 25). Optimum allocation in multivariate stratified sampling design in the presence of nonresponse with Gamma cost function, Journal of Statistical Computation and Simulation (2019) 89:13, pp. 2454-2467.

[109] Rahul Varshney, Najmussehar, M. J. Ahsan (2012). An optimum multivariate stratified double sampling design in presence of non-response, Optimization Letters (2012) 6: pp. 993-1008.

[110] Rahul Varshney, M. G. M. Khan, Ummatul Fatima, M. J. Ahsan (2015). Integer compromise allocation in multivariate stratified surveys, Annals of Operations Research (2015) 226:659-668.

[111] Rahul Varshney, Srikant Gupta, Irfan Ali (2017). An optimum multivariate-multiobjective stratified sampling design: fuzzy programming approach. Pakistan Journal of Statistics and Operations Research, pp. 829-855, December 2017

[112] V. Verma, H. C. Bakhshi, M. C. Puri (1990) Ranking in integer linear fractional programming problems ZOR – Methods and Models of Operations Research (1990) 34:325-334.

[113] Tawan Wasanapradit, Nalinee Mukdasanit, Nachol Chaiyaratana, Thongchai Srinophakun (2011). Solving mixed-integer nonlinear programming problems using improved genetic algorithms. Korean Joutnal of Chemical Engineering 28 (1):32-40 January 2011.

[114]  Eric W. Weisstein, “Diophantine Equation–8th Powers.”  https://mathworld.wolfram.com/DiophantineEquation8thPowers.html.

[115]  Eric W. Weisstein, “Euler’s Sum of 0wers Conjecture.”  https://mathworld.wolfram.com/EulersSumofPowersConjecture.html.

[116]  Eric W. Weisstein, “Diophantine Equation–5th Powers.”  https://mathworld.wolfram.com/DiophantineEquation5thPowers.html.

[117]  Eric W. Weisstein, “Diophantine Equation–10th Powers.”  https://mathworld.wolfram.com/DiophantineEquation10thPowers.html.

[118]  Eric W. Weisstein, “Diophantine Equation–9th Powers.”  https://mathworld.wolfram.com/DiophantineEquation9thPowers.html.

[119]  Rick Wicklin (2018), Solving a system of nonlinear equations with SAS.  blogs.sas.com>iml>2018/02/28.  One can directly read this on Google.

[120] Wikipedia, QB64, https://en.wikipedia.org/wiki/QB64.

[121] Wikipedia, Test functions for optimization, https://en.wikipedia.org/wiki/Test_functions_for_optimization.

[122] Jsun Yui Wong (2014, March 13).   The Domino Method Applied to a System of Four Simultaneous Nonlinear Equations.  Retrieved from http://myblogsubstance.typepad.com/substance/2014/03.

[123] Zhongkai Xu, Way Kuo, Hsin-Hui Lin (1990). Optimization Limits in Improving System Reliability. IEEE Transactions on Reliability, Vol. 39, No. 1, 1990 April, pp. 51-60.

Another Test Problem for Constrained Optimization

Jsun Yui Wong

The computer program listed below seeks to solve the following test problem in Parsopoulos and Vrahatis [84, Test Problem 4]:

minimize f = 5.3578547 * X(3) ^ 2 + .8356891 * X(1) * X(5) + 37.293239 * X(1) – 40792.141

subject to

     85.334407 + .0056858 * TONE + TTWO * X(1) * X(4) - .0022053 * X(3) * X(5) >=0,

     85.334407 + .0056858 * TONE + TTWO * X(1) * X(4) - .0022053 * X(3) * X(5)  <= 92,                                                                                                                 

     80.51249 + .0071317 * X(2) * X(5) + .0029955 * X(1) * X(2) + .0021813 * X(3) ^ 2 >= 90,

     80.51249 + .0071317 * X(2) * X(5) + .0029955 * X(1) * X(2) + .0021813 * X(3) ^ 2 <= 110,

     9.300961 + .0047026 * X(3) * X(5) + .0012547 * X(1) * X(3) + .0019085 * X(3) * X(4) >= 20,

     9.300961 + .0047026 * X(3) * X(5) + .0012547 * X(1) * X(3) + .0019085 * X(3) * X(4) <= 25,

     78 <= X(1)<=  102, 33 <= X(2)<=  45,  27 <=  X(i)<=  45, i=3, 4, 5,

    where TONE = X(2) * X(5) and TTWO = .0006262,    

      -10 <= X(i) <= 10     where i=1 through 5.                 

The best known solution is f*= -30665.538.

One notes that the following computer program takes advantage of the “what if” question that (at least) one of the six long constraints is active. See line 170, which is 170 X(3) = (85.334407 + .0056858 * TONE + TTWO * X(1) * X(4) – 92) / (.0022053 * X(5)).

0 REM DEFDBL A-Z

1 REM DEFINT K

2 DIM B(99), N(99), A(2002), H(99), L(99), U(99), X(2002), D(111), P(111), PS(33), J44(2002), J(99), AA(99), HR(32), HHR(32), LHS(44), PLHS(44), LB(22), UB(22), PX(22), CC(20), RR(20), WW(20), AL(50), SW(50), SV(50), C2(22), C3(22), C4(22), C5(22)

12 FOR JJJJ = -32000 TO 32000 STEP .01

13 RANDOMIZE JJJJ

16 M = -1D+37

17 A(1) = 78 + RND * 24

18 A(2) = 33 + RND * 12

19 FOR J44 = 3 TO 5

    21 A(J44) = 27 + RND * 18

22 NEXT J44

128 FOR I = 0 TO 3000000

    129 FOR KKQQ = 1 TO 5

        130 X(KKQQ) = A(KKQQ)
    131 NEXT KKQQ

    135 FOR IPP = 1 TO FIX(1 + RND ^ 5 * 5)
        143 j = 1 + FIX(RND * 5)
        154 IF RND < .5 THEN GOTO 156 ELSE GOTO 162
        156 R = (1 - RND * 2) * A(j)
        158 X(j) = A(j) + (RND ^ (RND * 30)) * R

        161 GOTO 165

        162 REM IF RND<.16666 THEN X(j)= .......

        164 IF RND < .5 THEN X(j) = A(j) - FIX(1 + RND * .3) ELSE X(j) = A(j) + FIX(1 + RND * .3)

    165 NEXT IPP

    169 TONE = X(2) * X(5): TTWO = .0006262

    170 X(3) = (85.334407 + .0056858 * TONE + TTWO * X(1) * X(4) - 92) / (.0022053 * X(5))

    171 IF 85.334407 + .0056858 * TONE + TTWO * X(1) * X(4) - .0022053 * X(3) * X(5) < 0## THEN 1670


    173 IF 80.51249 + .0071317 * X(2) * X(5) + .0029955 * X(1) * X(2) + .0021813 * X(3) ^ 2 < 90## THEN 1670
    174 IF 80.51249 + .0071317 * X(2) * X(5) + .0029955 * X(1) * X(2) + .0021813 * X(3) ^ 2 > 110## THEN 1670


    175 IF 9.300961 + .0047026 * X(3) * X(5) + .0012547 * X(1) * X(3) + .0019085 * X(3) * X(4) < 20## THEN 1670

    176 IF 9.300961 + .0047026 * X(3) * X(5) + .0012547 * X(1) * X(3) + .0019085 * X(3) * X(4) > 25## THEN 1670


    177 IF X(1) < 78 THEN 1670
    178 IF X(1) > 102 THEN 1670
    179 IF X(2) < 33 THEN 1670
    180 IF X(2) > 45 THEN 1670


    182 FOR J44 = 3 TO 5
        183 IF X(J44) < 27 THEN 1670
        184 IF X(J44) > 45 THEN 1670
    185 NEXT J44


    1012 P = -5.3578547 * X(3) ^ 2 - .8356891 * X(1) * X(5) - 37.293239 * X(1) + 40792.141

    1111 IF P <= M THEN 1670

    1420 M = P

    1444 FOR KLX = 1 TO 5

        1455 A(KLX) = X(KLX)
    1456 NEXT KLX

    1557 GOTO 128
1670 NEXT I
1890 IF M < 30665 THEN 1999
1926 PRINT A(1), A(2), A(3), A(4), A(5), M, JJJJ

1999 NEXT JJJJ

This BASIC computer program was run with qb64v1000-win [120]. The complete output of one run through JJJJ = -31999.96 is shown below:

78 33 29.99526 44.99902 36.77621
30665.51 -31000

78 33.00006 29.99529 44.99998 36.77573
30665.53 -31999.99

78 33.00011 29.99532 44.99996 36.77568
30665.53 -31999.98

78 33 29.99526 44.99972 36.77593
30665.53 -31999.97

78 33.00003 29.99527 44.99997 36.77579
30665.54 -31999.96

Above there is no rounding by hand; it is just straight copying by hand from the monitor screen.
The system properties of the computer system used include Intel Core 2 Duo CPU E8400 @ 3.00GHz 3.00GHz, 4.00GB of RAM, and qb64v1000-win [120]. The wall-clock time (not CPU time) for obtaining the output through JJJJ = -31999.96 was 160 seconds, counting from “Starting program…”. One can compare the computational results above with those in Parsopoulos and Vrahatis [84, Test Problem 4, Table 1].

Acknowledgement
I would like to acknowledge the encouragement of Roberta Clark and Tom Clark.
References
[1] M. Abdollahi, A. Isazadeh, D. Abdollahi (2013), Imperialist competitive algorithm for solving systems of nonlinear equations, Computers and Mathematics with Applications, vol. 65, number 12, pp. 1894-1908, 2013. (One can directly read this on Google.)
[2] Siby Abraham, Sugata Sanyal, Mukund Sanglikar (2010), Particle Swarm Optimisation Based Diophantine Equation Solver, Int. J. of Bio-Inspired Computation, 2 (2), 100-114, 2010.
[3] Siby Abraham, Sugata Sanyal, Mukund Sangrikar (2013), A Connectionist Network Approach to Find Numerical Solutions of Diophantine Equations, Int. J. of Engg. Science and Mgmt., Vol. III, Issue 1, January-June 2013.
[4] Andre R. S. Amaral (2006), On the Exact Solution of a Facility Layout Problem. European Journal of Operational Research 173 (2006), pp. 508-518.
[5] Andre R. S. Amaral (2008), An Exact Approach to the One-Dimensional Facility Layout Problem. Operations Research, Vol. 56, No. 4 (July-August, 2008), pp. 1026-1033.

[6] Andre R. S. Amaral (2011), Optimal Solutions for the Double Row Layout Problem. Optimization Letters, DOI 10.1007/s11590-011-0426-8, published on line 30 November 2011, Springer-Verlag 2011.

[7] Andre R. S. Amaral (2012), The Corridor Allocation Problem. Computers and Operations Research 39 (2012), pp. 3325-3330.

[8] George Anescu (2016), Gradual and cumulative improvements to the classical differential evolution scheme through experiments, Analele Universitatii de Vest, Timisoara, Seria Mathematica–Informatica, Vol. LIV, 2, (2016), 13-35. (One can directly read this on Google.)

[9] Oscar Augusto, Bennis Fouad, Stephane Caro (2012). A new method for decision making in multi-objective optimization problems. Pesquisa Operacional, Sociedade Brasileira de Pesquisa Operacional, 2012 32 (3), pp.331-369.

[10] Miguel F. Anjos, Anthony Vannelli, Computing Globally Optimal Solutions for Single-Row Layout Problems Using Semidefinite Programming and Cutting Planes. INFORMS Journal on Computing, Vol. 20, No. 4, Fall 2008, pp. 611-617.

[11] David L. Applegate, Robert E. Bixby, Vasek Chvatal, William J. Cook, The Traveling Salesman Problem: A Computational Study. Princeton and Oxford: Princeton University Press, 2006.

[12] Ritu Arora, S. R. Arora (2015). A cutting plane approach for multi-objective integer indefinite quadratic programming problem: OPSEARCH of the Operational Research Society of India (April-June 2015), 52(2):367-381.

[13] H. Bernau (1990). Active constraint strategies in optimization. Geographical data inversion methods and applications, pp. 15-31.

[14] Victor Blanco, Justo Puerto (2011). Some algebraic methods for solving multiobjective polynomial integer programs, Journal of Symbolic Computation, 46 (2011), pp. 511-533.

[15] Jerome Bracken, Garth P. McCormick, Selected Applications of Nonlinear Programming. New York: John Wiley and Sons, Inc., 1968.

[16] Neima Brauner, Moredechai Shacham, Michael Cutlip (winter 1996) Computational Results – How Reliable Are They? Chemical Engineering Educatiobn, 30, 1, pp. 20-25 (1996). (One can read this via Google.)

[17] Richard L. Burden, Douglas J. Faires, Annette M. Burden, Numerical Analysis, Tenth Edition, 2016, Cengage Learning.

[18] R. C. Carlson and G. L. Nemhauser, Scheduling To Minimize Interaction Cost. Operations Research, Vol. 14, No. 1 (Jan. – Feb., 1966), pp. 52-58.

[19] Ta-Cheng Chen (2006). IAs based approach for reliability redundany allocation problems. Applied Mathematics and Computation 182 (2006) 1556-1567.

[20] Ward Cheney, David Kincaid (2008), Numerical Mathematics and Computing, Sixth Edition, 2008, Thomsom Brooks/Coles.

[21] Leandro dos Santos Coelho (2009), Self-Organizing Migrating Strategies Applied to Reliability-Redundany Optimization of Systems. IEEE Transactions on Reliability, Vol. 58, No. 3, 2009 September, pp. 501-519.

[22] A. J. Conejo, L. Baringo (2018), Power System Operations, Springer International Publishing, 2018.

[23] William Conley (1981). Optimization: A Simplified Approach. Published 1981 by Petrocelli Books in New York.

[24] Lino Costa, Pedro (2001). Evolutionary algorithms approach to the solution of mixed integer non-linear programming problems. Computers and Chemical Engineering, Vol. 25, pp. 257-266, 2001.

[25] Michael B. Cutlip, Mordechai Shacham (1998), POLYMATH verssion 4.1 user-friendly numerical analysis programs. http://www.polymath-software.com.

[26] George B. Dantzig, Discrete-Variable Extremum Problems. Operations Research, Vol. 5, No. 2 (Apr., 1957), pp. 266-277.

[27] Pintu Das, Tapan Kumar Roy (2014). Multi-objective geometric programming and its application in gravel box problem. Journal of global research in computer science volume 5. no.7, July 2014. http://www.jgrcs.info

[28] Kalyanmoy Deb, Amrit Pratap, Subrajyoti Moitra (2000). Mechanical component design for multi objectives using elitist non-dominated sorting GA. Proceedings of the Parallel Probl.Solv. Nat. PPSN VI Conference, Paris, France, pp. 859-868 (2000). (Please see pp. 1-10 in Technical Report No. 200002 via Google search.)

[29] Kusum Deep, Krishna Pratap Singh, M. L. Kansal, C. Mohan (2009), A real coded genetic algorithm for solving integer and mixed integer optimization problems. Applied Mathematics and Computation 212 (2009) 505-518.

[30] Wassila Drici, Mustapha Moulai (2019): An exact method for solving multi-objective integer indefinite quadratic programs, Optimization Methods and Software.

[31] R. J. Duffin, E. L. Peterson, C. Zener (1967), Geometric Programming. John Wiley, New York (1967).

[32] Simge Ekiz, Pakize Erdogmus, Busra Ozgur (2017 November), Solving constrained optimization problems with sine-cosine algorithm, Periodicals of Engineering and Natural Sciences, vol. 5, no. 3, November 2017, pp.378~386. http://pen.ius.edu.ba.

[33] C. A. Floudas, A. R. Ciric (1989), Strategies for Overcoming Uncertainties in Heat Exchanger Network Synthesis. Computers and Chemical Engineering, Vol 13, No. 10, pp. 1133-1152, 1989.

[34] C. A. Floudas, A. Aggarwal, A. R. Ciric (1989), Global Optimum Search for Nonconvex NLP and MINLP Problems. Computers and Chemical Engineering, Vol 13, No. 10, pp. 1117-1132, 1989.

[35] C. A. Floudas, A. Aggarwal, A. R. Ciric (1989), Global Optimum Search for Nonconvex NLP and MINLP Problems. Computers and Chemical Engineering, Vol 13, No. 10, pp. 1117-1132, 1989.

[36] C. A. Floudas, P. M. Pardalos, A Collection of Test Problems for Constrained Global Optimization Algorithms. Springer-Verlag, 1990.

[37] C. A. Floudas(1995), Nonlinear and mixed-integer optimization: fundamentals and applications. Oxford University Press, 1995.

[38] Diptesh Ghosh, Ravi Kothari, Population Heuristics for the Corridor Allocation Problem, W.P. No. 2012-09-02, September 2012. Retrieved on September 14 2012 from Google search.

[39] Crina Grosan, Ajith Abraham (2008), A New Approach for Solving Nonlinear Equations Systems. IEEE Transactions on Systems, Man and Cybernetics–Part A: Systems and Humans, Volume 38, Number 3, May 2008, pp. 698-714.

[40] Crina Grosan, Ajith Abraham (2008), Multiple Solutions for a System pf Nonlinear Equations. International Journal of Innovative Computing, Information and Control, Volume x, Number x, x 2008.

[41] GUO Chuang-xin, HU Jia-sheng, YE Bin, CAO Yi-jia (2004), Swarm intelligence for mixed-variable design optimization, J. of Zhejiang University SCIENCE 2004 5(7):851-860. One can directly read this on Google.

[42] Neha Gupta, Irfan Ali, Abdul Bari (2013). An optimal chance constraint multivariate stratified sampling design using auxiliary infotmation. Journal of Mathematical Modelling and Algorithms, January 2013.

[43] R. Gupta, R. Malhotra (1995). Multi-criteria integer linear fractional programming problem, Optimization, 35:4, 373-389.

[44] M. Hashish, M. P. duPlessis (1979). Prediction equations relating high velocity jet cutting performance to stand-off-distance and multipasses. Transactions of ASME: Journal of Engineering for Industry 101 (1979) 311-318.

[45] Abdel-Rahman Hedar, Masao Fukushima (2006) Derivative-free simulated annealing method for constrained continuous optimization, J. of Global Optimization, January 2006. (One can directly view this on Google.)

[46] www – optima.amp.i.kyoto – u.ac.jp/member/student/hedar/Hedar_fil…
[47] Frederick S. Hillier, Gerald J. Liberman (1990), Introduction to Mathematical Programming, McGraw-Hill Publishing Company, 1990.

[48] Willi Hock, Klaus Schittkowski, Test Examples for Nonlinear Programming Codes. Berlin: Springer-Verlag, 1981.

[49] Sana Iftekhar, M. J. Ahsan, Qazi Mazhar Ali (2015). An optimum multivariate stratified sampling design. Research Journal of Mathematical and Statistical Sciences, vol. 3(1),10-14, January (2015).

[50] Philipp Hungerlaender (April 2012), Single-Row Equidistant Facility Layout as a Special Case of Single-Row Facility Layout. Retrieved from http://www.optimization-online.org./DB_HTML/2012/04/3432.html

[51] Golam Reza Jahanshahloo, Farhad Hosseinzadeh, Nagi Shoja, Ghasem Tohidi (2003) A method for solvong 0-1 multiple objective liner programming problem using DEA. Journal of the Operations Research Society of Japan (2003), 46 (2): 189-202. http://www.orsj.or.jp/~archive/pdf/e_mag/Vol.46_02_189.pdf

[52] Ekta Jain, Kalpana Dahiya, Vanita Verma (2018): Integer quadratic fractional programming problems with bounded variables, Annals of Operations Research (October 2018) 269: pp. 269-295.

[53] N. K. Jain, V. K. Jain, K. Deb (2007). Optimization of process parameters of mechanical type advanced machining processes using genetic algorithms. International Journal of Machine Tools and Manufacture 47 (2007), 900-919.

[54] Michael Junger, Thomas M. Liebling, Dennis Naddef, George L. Nemhauser, William R. Pulleybank, Gerhart Reinelt, Giovanni Rinaldi, Lawrence A. Wolsey–Editors, 50 Years of Integer Programming 1958-2008. Berlin: Springer, 2010.

[55] Adhe Kania, Kuntjoro Adji Sidarto (2016). Solving mixed integer nonlinear programming problems using spiral dynamics optimization algorithm. AIP Conference Proceedings 1716, 020004 (2016). 29, 2016. https://doi:org/10.1063/1.4942987.

[56] Reena Kapoor, S. R. Arora (2006). Linearization of a 0-1 quadratic fractional programming problem: OPSEARCH of the Operational Research Society of India (2006), 43(2):190-207.

[57] M. G. M. Khan, E. A. Khan, M. J. Ahsan (2003). An optimal multivariate stratified sampling design using dynami programming. Australian and New Zealand Journal of Statistics, vol. 45, no. 1, 2003, pp. 107-113.

[58] M. G. M. Khan, T. Maiti, M. J. Ahsan (2010). An optimal multivariate stratified sampling design using auxiliary information: an integer solution using goal programming approach. Journal of Official Statistics, vol. 26, no. 4, 2010, pp. 695-708.

[59] A. H. Land, A. G. Doig, An Automatic Method of Solving Discrete Programming Problems. Econometrica, Vol. 28, No. 3 (Jul., 1960), pp. 497-520.

[60] E. L. Lawler, M. D. Bell, A Method for Solving Discrete Optimization Problems. Operations Research, Vol. 14, No. 6 (Nov.-Dec., 1966), pp. 1098-1112.

[61] Sven Leyffer (December 1993) Deterministic Methods for Mixed Integer Nonlinear Progrmming, PhD Thesis, Department of Mathematics and Computer Science, University of Dundee, Dundee, December 1993. (One can directly read this on Google.)

[62] S. Leyffer, J. Linderoth, J. Luedtke, A. Miller, T. Munson (2009), Applications and algorithms for mixed integer nonlinear programming, J. of Physics: Conference Series 180 (2009) 012014, IOP Publishing. (One can directly read this on Google.)

[63] Yugui Li, Yanxu Wei, Yantao Chu (2015), Research on solving systems of nonlinear equations based on improved PSO, Mathematical Problems in Engineering, volume 2015, article ID 727218, 13 pages, Hindawi Publishing Corporration. One can directly read this article on Google.

[64] F. H. F. Liu, C. C. Huang, Y. L. Yen (2000): Using DEA to obtain efficient solutions for multi-objective 0-1 linear programs. European Journal of Operational Research 126 (2000) 51-68.

[65] Gia-Shi Liu (2006), A combination method for reliability-redundancy optimization, Engineering Optimization, 38:04, 485-499.

[66] Yubao Liu, Guihe Qin (2014), A hybrid TS-DE algorithm for reliability redundancy optimization problem, Journal of Computers, 9, No. 9, September 2014, pp. 2050-2057.

[67] Rein Luus (1975). Optimization of System Reliability by a New Nonlinear Integer Programming Procedure. IEEE Transactions on Reliability, Vol. R-24, No. 1, April 1975, pp. 14-16.

[68] Milos Madic, Miroslav Radovanovic (2014). Optimization of machining processes using pattern search algorithm. International Journal of Industrial Engineering Computations 5 (2014) 223-234. Homepage: http://www.GrowingScience.com/ijiec

[69] F. Masedu, M Angelozzi (2008). Modelling optimum fraction assignment in the 4X100 m relay race by integer linear programming. Italian Journal of Sports Sciences, Anno 13, No. 1, 2008, pp. 74-77.

[70] Mohamed Arezki Mellal, Enrico Zio (2016). A Guided Stochastic Fractal Search Approach for System Reliability Optimization. Reliability Engineering and System Safety 152 (2016) 213-227.

[71] Mohamed Arezki Mellal, Edward J. Williams (2016). Parameter optimization of advanced machining processes using cuckoo optimization algorithm and hoopla heuristic. Journal of Intelligent Manufacturing (2016) 27 (5): 927-942.

[72] Mohamed Arezki Mellal, Edward J. Williams (2018). Large-scale reliability-redundancy allocation optimization problem using three soft computing methods. In Mangey Ram, Editor, in Modeling and simulation based analysis in reliability engineering. Published July 2018, CRC Press.

[73] Microsoft Corp., BASIC, Second Edition (May 1982), Version 1.10. Boca Raton, Florida: IBM Corp., Personal Computer, P. O. Box 1328-C, Boca Raton, Florida 33432, 1981.

[74] Joaquin Moreno, Miguel Lopez, Raquel Martinez (2018) A new method for solving nonlinear systems of equations that is based on functional iterations, Open Physics 2018; 16: 605-630.

[75] Yuji Nakagawa, Mitsunori Hikita, Hiroshi Kamada (1984). Surrogate Constraints for Reliability Optimization Problems with Multiple Constraints. IEEE Transactions on Reliability, Vol. R-33, No. 4, October 1984, pp. 301-305.

[76] Subhash C. Narula, H. Roland Weistroffer (1989). A flexible method for nonlinear criteria decisionmaking problems. Ieee Transactions on Systems, Man and Cybernetics, vol. 19 , no. 4, July/August 1989, pp. 883-887.

[77] C. E. Nugent, T. E. Vollmann, J. Ruml (1968), An Experimental Comparison of Techniques for the Assignment of Facilities to Locations, Operations Research 16 (1968), pp. 150-173.

[78] A. K. Ojha, K. K. Biswal (2010). Multi-objective geometric programming problem with weighted mean method. (IJCSIS) International Journal of Computer Science and Information Security, vol. 7, no. 2, pp. 82-86, 2010.

[79] Rashmi Ranjan Ota, jayanta kumar dASH, A. K. Ojha (2014). A hybrid optimization techniques for solving non-linear multi-objective optimization problem. amo-advanced modelling and optimization, volume 16,number 1, 2014.

[80] Rashmi Ranjan Ota, A. K. Ojha (2015). A comparative study on optimization techniques for solving multi-objective geometric programming problems. Applied mathematical sciences, vol. 9, 2015, no. 22, 1077-1085. http://sites.google.com/site/ijcsis/

[81] R. R. Ota, J. C. Pati, A. K. Ojha (2019). Geometric programming technique to optimize power distribution system. OPSEARCH of the Operational Research Society of India (2019), 56, pp. 282-299.

[82] OPTI Toolbox, Mixed Integer Nonlinear Program (MINLP). https://www.inverseproblem.co.nz/OPTI/index.php/Probs/MINLP

[83] Panos Y. Papalambros, Douglass J. Wilde, Principles of Optimal Design, Second Edition. Cambridge University Press, 2000.

[84] K. E. Parsopoulos, M. N. Vrahatis, Particle swarm optimization method for constrained optimization. Intelligent Technologies–Theory…, 2002. (One can directly read this on Google Scholar.)

[85] O. Perez, I. Amaya, R. Correa (2013), Numerical solution of certain exponential and nonlinear diophantine systems of equations by using a discrete particles swarm optimization algorithm. Applied Mathematics and Computation, Volume 225, 1 December, 2013, pp. 737-746.

[86] Ciara Pike-Burke. Multi-Objective Optimization. https://www.lancaster.ac.uk/pg/pikeburc/report1.pdf.

[87] Yashpal Singh Raghav, Irfan Ali, Abdul Bari (2014) Multi-Objective Nonlinear Programming Problem Approach in Multivariate Stratified Sample Surveys in the Case of the Non-Response, Journal of Statitiscal Computation ans Simulation 84:1, 22-36, doi:10.1080/00949655.2012.692370.

[88] R. V. Rao, P. J. Pawar, J. P. Davim (2010). Parameter optimization of ultrasonic machining process using nontraditional optimization algorihms. Materials and Manufacturing Processes, 25 (10),1120-1130.

[89] W. Rheinboldt, Some Nonlinear Testproblems. One can google this 10-page paper.

[90] John Rice, Numerical Methods, Software, and Analysis, Second Edition, 1993, Academic Press.

[91] H. S. Ryoo, N. V. Sahinidis (1995), Global Optimization of Nonconvex NLPs and MINLPs with Applications in Process Design. Computers and Chemical Engineering, Vol. 19, No. 5, pp. 551-566, 1995.

[92] Ali Sadollah, Hadi Eskandar, Joong Hoon Kim (2015). Water cycle algorithm for solvinfg constrained multi-objective optimization problems. Applied Soft Computing 27 (2015) 279-298.

[93] Shafiullah, Irfan Ali, Abdul Bari (2015). Fuzzy geometric programming approach in multi-objective multivariate stratified sample surveys in presence of non-respnse, International Journal of Operations Research, Vol. 12, No. 2, pp. 021-035 (2015).

[94] Mordechai Shacham (1986). Numerical solution of constrained nonlinear algebraic equations, International Journal for Numerical Methods in Engineering, Vol. 23, pp. 1455-1481 (1986). (One can read this via Google.)

[95] M. Shacham, R. S. H. Mah (1978). A Newton Type Linearization Method for Solution of Nonlinear Equations, Computers and Chemical Engineering, Vol. 2, pp. 64-66 (1978). (One can read this via Google.)

[96] Yi Shang (1997), Global search methods for solving nonlinear optimization problems, Ph. D. Thesis, University of Illinois at Urbana-Champaign, 1997.

[97] Vikas Sharma (2012). Multiobjective integer nonlinear fractional programming problem: A cutting plane approach, OPSEARCH of the Operational Research Society of India (April-June 2012), 49(2):133-153.

[98] Vikas Sharma, Kalpana Dahiya, Vanita Verma (2017). A ranking algorithm for bi-objective quadratic fractional integer programming problems, Optimization, 66:11, 1913-1929.

[99] K. A. Sidarto, Adhe Kania (2015), Finding all solutions of systems of nonlinear equations using spiral dynamics inspired optimization with clustering,
Journal of Advanced Computational Intelligence and Intelligent Informatics, pp. 697-708, vol. 19, number 5, 2015. (One can directly read this via Google.)

[100] G. Stephanopoulos, A. W. Westerberg, The Use of Hestenes’ Method of Multipliers to Resolve Dual Gaps in Engineering System Optimization. Journal of Optimization Theory and Applications, Vol.15, No. 3, pp. 285-309, 1975.

[101] Sonja Surjanovic, Derek Bingham (2013), Virtual library of simulation experiments: test functions and datasets. Simon Fraser University, Burnaby, BC, Canada, 2013. (One can directly read this on Google.)

[102] Hardi Tambunan, Herman Mawengkang (2016). Solving Mixed Integer Non-Linear Programming Using Active Constraint. Global Journal of Pure and Applied Mathematics, Volume 12, Number 6 (2016), pp. 5267-5281. http://www.ripublication.com/gjpam.htm

[103] Mohamed Tawhid, Vimal Savsani (2018). Epsilon-constraint heat transfer search (epsilon-HTS) algorithm for solvinfg multi-objective engineering design problems. journal of computational design and engineering 5 (2018) 104-119.

[104] Frank A. Tillman, Ching-Lai Hwang, Way Kuo (1977). Determining Component Reliability and Redundancy for Optimun System Reliability. IEEE Transactions on Reliability, Vol. R-26, No. 3, Augusr 1977, pp. 162-165.

[105] Rahul Varshney, Srikant Gupta, Irfan Ali (2017). An optimum multivariate-multiobjective stratified samplinr design: fuzzy programming approach. Pakistan Journal of Statistics and Operations Research, pp. 829-855, December 2017

[106] V. Verma, H. C. Bakhshi, M. C. Puri (1990) Ranking in integer linear fractional programming problems ZOR – Methods and Models of Operations Research (1990) 34:325-334.

[107] Tawan Wasanapradit, Nalinee Mukdasanit, Nachol Chaiyaratana, Thongchai Srinophakun (2011). Solving mixed-integer nonlinear programming problems using improved genetic algorithms. Korean Joutnal of Chemical Engineering 28 (1):32-40 January 2011.

[108] Rahul Varshney, Mradula (2019 May 25). Optimum allocation in multivariate stratified sampling design in the presence of nonresponse with Gamma cost function, Journal of Statistical Computation and Simulation (2019) 89:13, pp. 2454-2467.

[109] Rahul Varshney, Najmussehar, M. J. Ahsan (2012). An optimum multivariate stratified double sampling design in presence of non-response, Optimization Letters (2012) 6: pp. 993-1008.

[110] Rahul Varshney, M. G. M. Khan, Ummatul Fatima, M. J. Ahsan (2015). Integer compromise allocation in multivariate stratified surveys, Annals of Operations Research (2015) 226:659-668.

[111] Rahul Varshney, Srikant Gupta, Irfan Ali (2017). An optimum multivariate-multiobjective stratified sampling design: fuzzy programming approach. Pakistan Journal of Statistics and Operations Research, pp. 829-855, December 2017

[112] V. Verma, H. C. Bakhshi, M. C. Puri (1990) Ranking in integer linear fractional programming problems ZOR – Methods and Models of Operations Research (1990) 34:325-334.

[113] Tawan Wasanapradit, Nalinee Mukdasanit, Nachol Chaiyaratana, Thongchai Srinophakun (2011). Solving mixed-integer nonlinear programming problems using improved genetic algorithms. Korean Joutnal of Chemical Engineering 28 (1):32-40 January 2011.

[114] Eric W. Weisstein, “Diophantine Equation–8th Powers.” https://mathworld.wolfram.com/DiophantineEquation8thPowers.html.

[115] Eric W. Weisstein, “Euler’s Sum of 0wers Conjecture.” https://mathworld.wolfram.com/EulersSumofPowersConjecture.html.

[116] Eric W. Weisstein, “Diophantine Equation–5th Powers.” https://mathworld.wolfram.com/DiophantineEquation5thPowers.html.

[117] Eric W. Weisstein, “Diophantine Equation–10th Powers.” https://mathworld.wolfram.com/DiophantineEquation10thPowers.html.

[118] Eric W. Weisstein, “Diophantine Equation–9th Powers.” https://mathworld.wolfram.com/DiophantineEquation9thPowers.html.

[119] Rick Wicklin (2018), Solving a system of nonlinear equations with SAS. blogs.sas.com>iml>2018/02/28. One can directly read this on Google.

[120] Wikipedia, QB64, https://en.wikipedia.org/wiki/QB64.

[121] Wikipedia, Test functions for optimization, https://en.wikipedia.org/wiki/Test_functions_for_optimization.

[122] Jsun Yui Wong (2014, March 13). The Domino Method Applied to a System of Four Simultaneous Nonlinear Equations. Retrieved from http://myblogsubstance.typepad.com/substance/2014/03.

[123] Zhongkai Xu, Way Kuo, Hsin-Hui Lin (1990). Optimization Limits in Improving System Reliability. IEEE Transactions on Reliability, Vol. 39, No. 1, 1990 April, pp. 51-60.

A Test Problem for Constrained Optimization 

     

Jsun Yui Wong

The computer program listed below seeks to solve the following test problem in Parsopoulos and Vrahatis [84, Test Problem 2]:

minimize           f=    (X(1) – 10) ^ 3 + (X(2) – 20) ^ 3

subject  to  

    100 – (X(1) – 5) ^ 2 – (X(2) – 5) ^ 2 <= 0,

    (X(1) – 6) ^ 2 + (X(2) – 5) ^ 2 – 82.81 <= 0

           13 <= X(1) <=100,            

        0 <= X(2) <=100,            

The best known solution is f*= -6961.81381.

0 REM      DEFDBL A-Z

2 REM    DEFINT K

3 DIM B(99), N(99), A(100255), H(99), L(99), U(99), X(100250), D(111), P(511), PS(33), J(30003), J44(30003), KKQQ(30003), KLX(30003), W(10111)

12 FOR JJJJ = -32000 TO 32000 STEP .01

    13 RANDOMIZE JJJJ

    16 M = -1D+37

    21 A(1) = RND * 50

    22 A(2) = RND * 50

    128 FOR I = 1 TO 300000

        129 FOR KKQQ = 1 TO 2

            130 X(KKQQ) = A(KKQQ)

        131 NEXT KKQQ

        135 FOR IPP = 1 TO (1 + FIX(RND ^ 5 * 5))

            148 J = 1 + FIX(RND * 2)

            153 REM    GOTO 162

            154 IF RND < .5 THEN GOTO 156 ELSE GOTO 162

            156 REM

            157 R = (1 – RND * 2) * (A(J))

            160 X(J) = A(J) + (RND ^ (RND * 30)) * R

            161 GOTO 165

            162 REM    IF RND < .16666 THEN X(J) = A(J) – FIX(1 + RND * 2.3) ELSE IF RND < .33333 THEN X(J) = A(J) + FIX(1 + RND * 2.3) ELSE IF RND < .5 THEN X(J) = A(J) – FIX(1 + RND * 20.3) ELSE IF RND < .66666 THEN X(J) = A(J) + FIX(1 + RND * 20.3) ELSE IF RND < .83333 THEN X(J) = A(J) – FIX(1 + RND * 200.3) ELSE X(J) = A(J) + FIX(1 + RND * 200.3)

            164 IF RND < .5 THEN X(J) = A(J) – FIX(1 + RND * .3) ELSE X(J) = A(J) + FIX(1 + RND * .3)

        165 NEXT IPP

        175 IF 100 – (X(1) – 5) ^ 2 – (X(2) – 5) ^ 2 > 0## THEN 1670

        177 IF (X(1) – 6) ^ 2 + (X(2) – 5) ^ 2 – 82.81 > 0## THEN 1670

        197 IF X(1) < 13 THEN 1670

        198 IF X(1) > 100 THEN 1670

        199 IF X(2) < 0 THEN 1670

        200 IF X(2) > 100 THEN 1670

        959 P = -(X(1) – 10) ^ 3 – (X(2) – 20) ^ 3

        1111 IF P <= M THEN 1670

        1452 M = P

        1454 FOR KLX = 1 TO 2

            1459 A(KLX) = X(KLX)

        1460 NEXT KLX

        1557 GOTO 128

    1670 NEXT I

    1677 IF M < -9999 THEN 1999

    1908 PRINT A(1), A(2), M, JJJJ

1999 NEXT JJJJ

This BASIC computer program was run with qb64v1000-win [120].  The complete output of one run through JJJJ =  -31999.66 is shown below:

14.09501      .8429765      6961.796      -31999.99

14.09501      .8429726      6961.801      -31999.83

14.09501      .8429765      6961.796      -31999.79

14.09501      .8429765      6961.796      -31999.77

14.09501      .8429706      6961.803      -31999.7

14.09501      .8429706      6961.803      -31999.67

14.095          .842969        6961.805      -31999.66

Above there is no rounding by hand; it is just straight copying by hand from the monitor screen.

The system properties of the computer system used include Intel Core 2 Duo CPU   E8400 @ 3.00GHz  3.00GHz, 4.00GB of RAM, and qb64v1000-win [120].  The wall-clock time (not CPU time) for obtaining the output through JJJJ = -31999.66 was 16 seconds, counting from “Starting program…”.  One can compare the computational results above with those in Parsopoulos and Vrahatis [84, Test Problem 2, Table 1].  

Acknowledgement

I would like to acknowledge the encouragement of Roberta Clark and Tom Clark.

References

[1]  M. Abdollahi, A. Isazadeh, D. Abdollahi (2013), Imperialist competitive algorithm for solving systems of nonlinear equations, Computers and Mathematics with Applications, vol. 65, number 12, pp. 1894-1908, 2013.  (One can directly read this on Google.)

[2] Siby Abraham, Sugata Sanyal, Mukund Sanglikar (2010), Particle Swarm Optimisation Based Diophantine Equation Solver,  Int. J. of Bio-Inspired Computation, 2 (2), 100-114, 2010. 

[3] Siby Abraham, Sugata Sanyal, Mukund Sangrikar (2013), A Connectionist Network Approach to Find Numerical Solutions of Diophantine Equations, Int. J. of Engg. Science and Mgmt., Vol. III, Issue 1, January-June 2013.

[4] Andre R. S. Amaral (2006), On the Exact Solution of a Facility Layout Problem. European Journal of Operational Research 173 (2006), pp. 508-518.

[5] Andre R. S. Amaral (2008), An Exact Approach to the One-Dimensional Facility Layout Problem. Operations Research, Vol. 56, No. 4 (July-August, 2008), pp. 1026-1033.

[6] Andre R. S. Amaral (2011), Optimal Solutions for the Double Row Layout Problem. Optimization Letters, DOI 10.1007/s11590-011-0426-8, published on line 30 November 2011, Springer-Verlag 2011.

[7] Andre R. S. Amaral (2012), The Corridor Allocation Problem. Computers and Operations Research 39 (2012), pp. 3325-3330.

[8]  George Anescu (2016), Gradual and cumulative improvements to the classical differential evolution scheme through experiments, Analele  Universitatii de Vest, Timisoara, Seria Mathematica–Informatica, Vol. LIV, 2, (2016), 13-35.   (One can directly read this on Google.)

[9] Oscar Augusto, Bennis Fouad, Stephane Caro (2012). A new method for decision making in multi-objective optimization problems. Pesquisa Operacional, Sociedade Brasileira de Pesquisa Operacional, 2012 32 (3), pp.331-369.

[10] Miguel F. Anjos, Anthony Vannelli, Computing Globally Optimal Solutions for Single-Row Layout Problems Using Semidefinite Programming and Cutting Planes. INFORMS Journal on Computing, Vol. 20, No. 4, Fall 2008, pp. 611-617.

[11] David L. Applegate, Robert E. Bixby, Vasek Chvatal, William J. Cook, The Traveling Salesman Problem: A Computational Study. Princeton and Oxford: Princeton University Press, 2006.

[12] Ritu Arora, S. R. Arora (2015). A cutting plane approach for multi-objective integer indefinite quadratic programming problem: OPSEARCH of the Operational Research Society of India (April-June 2015), 52(2):367-381.

[13]   H. Bernau (1990).  Active constraint strategies in optimization.  Geographical data inversion methods and applications, pp. 15-31.

[14] Victor Blanco, Justo Puerto (2011). Some algebraic methods for solving multiobjective polynomial integer programs, Journal of Symbolic Computation, 46 (2011), pp. 511-533.

[15] Jerome Bracken, Garth P. McCormick, Selected Applications of Nonlinear Programming. New York: John Wiley and Sons, Inc., 1968.

[16] Neima Brauner, Moredechai Shacham, Michael Cutlip (winter 1996)  Computational Results – How Reliable Are They? Chemical Engineering Educatiobn, 30, 1,  pp. 20-25 (1996).  (One can read this via Google.)

[17] Richard L. Burden, Douglas J. Faires, Annette M. Burden, Numerical Analysis, Tenth Edition, 2016, Cengage Learning.

[18] R. C. Carlson and G. L. Nemhauser, Scheduling To Minimize Interaction Cost. Operations Research, Vol. 14, No. 1 (Jan. – Feb., 1966), pp. 52-58.

[19] Ta-Cheng Chen (2006). IAs based approach for reliability redundany allocation problems. Applied Mathematics and Computation 182 (2006) 1556-1567.

[20]  Ward Cheney, David Kincaid (2008), Numerical Mathematics and Computing, Sixth Edition, 2008, Thomsom Brooks/Coles.

[21] Leandro dos Santos Coelho (2009), Self-Organizing Migrating Strategies Applied to Reliability-Redundany Optimization of Systems. IEEE Transactions on Reliability, Vol. 58, No. 3, 2009 September, pp. 501-519.

[22]  A. J. Conejo, L. Baringo (2018), Power System Operations, Springer International Publishing, 2018.

[23] William Conley (1981). Optimization: A Simplified Approach. Published 1981 by Petrocelli Books in New York.

[24] Lino Costa, Pedro (2001). Evolutionary algorithms approach to the solution of mixed integer non-linear programming problems. Computers and Chemical Engineering, Vol. 25, pp. 257-266, 2001.

[25] Michael B. Cutlip, Mordechai Shacham (1998), POLYMATH verssion 4.1 user-friendly numerical analysis programs.  http://www.polymath-software.com.

[26] George B. Dantzig, Discrete-Variable Extremum Problems. Operations Research, Vol. 5, No. 2 (Apr., 1957), pp. 266-277.

[27] Pintu Das, Tapan Kumar Roy (2014). Multi-objective geometric programming and its application in gravel box problem. Journal of global research in computer science volume 5. no.7, July 2014. http://www.jgrcs.info

[28] Kalyanmoy Deb, Amrit Pratap, Subrajyoti Moitra (2000). Mechanical component design for multi objectives using elitist non-dominated sorting GA. Proceedings of the Parallel Probl.Solv. Nat. PPSN VI Conference, Paris, France, pp. 859-868 (2000). (Please see pp. 1-10 in Technical Report No. 200002 via Google search.)

[29] Kusum Deep, Krishna Pratap Singh, M. L. Kansal, C. Mohan (2009), A real coded genetic algorithm for solving integer and mixed integer optimization problems. Applied Mathematics and Computation 212 (2009) 505-518.

[30] Wassila Drici, Mustapha Moulai (2019): An exact method for solving multi-objective integer indefinite quadratic programs, Optimization Methods and Software.

[31] R. J. Duffin, E. L. Peterson, C. Zener (1967), Geometric Programming. John Wiley, New York (1967).

[32]  Simge Ekiz, Pakize Erdogmus, Busra Ozgur (2017 November), Solving constrained  optimization problems with sine-cosine algorithm, Periodicals of Engineering and Natural Sciences, vol. 5, no. 3, November 2017, pp.378~386.  http://pen.ius.edu.ba.  

[33] C. A. Floudas, A. R. Ciric (1989), Strategies for Overcoming Uncertainties in Heat Exchanger Network Synthesis. Computers and Chemical Engineering, Vol 13, No. 10, pp. 1133-1152, 1989.

[34] C. A. Floudas, A. Aggarwal, A. R. Ciric (1989), Global Optimum Search for Nonconvex NLP and MINLP Problems. Computers and Chemical Engineering, Vol 13, No. 10, pp. 1117-1132, 1989.

[35] C. A. Floudas, A. Aggarwal, A. R. Ciric (1989), Global Optimum Search for Nonconvex NLP and MINLP Problems. Computers and Chemical Engineering, Vol 13, No. 10, pp. 1117-1132, 1989.

[36] C. A. Floudas, P. M. Pardalos, A Collection of Test Problems for Constrained Global Optimization Algorithms. Springer-Verlag, 1990.  

[37] C. A. Floudas(1995),  Nonlinear and mixed-integer optimization:  fundamentals and applications.  Oxford University Press, 1995.

[38] Diptesh Ghosh, Ravi Kothari, Population Heuristics for the Corridor Allocation Problem, W.P. No. 2012-09-02, September 2012. Retrieved on September 14 2012 from Google search.

[39]  Crina Grosan, Ajith Abraham  (2008),    A New Approach for Solving Nonlinear Equations Systems.  IEEE Transactions on Systems, Man and Cybernetics–Part A:  Systems and Humans, Volume 38, Number 3, May 2008, pp. 698-714.

[40]  Crina Grosan, Ajith Abraham  (2008), Multiple Solutions for a System pf Nonlinear Equations. International Journal of Innovative Computing, Information and Control, Volume x, Number x, x 2008.

[41]  GUO Chuang-xin,  HU Jia-sheng, YE Bin, CAO Yi-jia (2004), Swarm intelligence for mixed-variable design optimization, J. of Zhejiang University SCIENCE 2004 5(7):851-860.  One can directly read this on Google.

[42] Neha Gupta, Irfan Ali, Abdul Bari (2013). An optimal chance constraint multivariate stratified sampling design using auxiliary infotmation. Journal of Mathematical Modelling and Algorithms, January 2013.

[43] R. Gupta, R. Malhotra (1995). Multi-criteria integer linear fractional programming problem, Optimization, 35:4, 373-389.

[44] M. Hashish, M. P. duPlessis (1979). Prediction equations relating high velocity jet cutting performance to stand-off-distance and multipasses. Transactions of ASME: Journal of Engineering for Industry 101 (1979) 311-318.

[45]  Abdel-Rahman Hedar, Masao Fukushima (2006)  Derivative-free simulated annealing method for constrained continuous optimization, J. of Global Optimization, January 2006.   (One can directly view this on Google.)

[46]  www – optima.amp.i.kyoto – u.ac.jp/member/student/hedar/Hedar_fil…

[47]  Frederick S. Hillier, Gerald J. Liberman (1990), Introduction to Mathematical Programming, McGraw-Hill Publishing Company, 1990.

[48] Willi Hock, Klaus Schittkowski, Test Examples for Nonlinear Programming Codes. Berlin: Springer-Verlag, 1981.

[49] Sana Iftekhar, M. J. Ahsan, Qazi Mazhar Ali (2015). An optimum multivariate stratified sampling design. Research Journal of Mathematical and Statistical Sciences, vol. 3(1),10-14, January (2015).

[50] Philipp Hungerlaender (April 2012), Single-Row Equidistant Facility Layout as a Special Case of Single-Row Facility Layout. Retrieved from http://www.optimization-online.org./DB_HTML/2012/04/3432.html

[51] Golam Reza Jahanshahloo, Farhad Hosseinzadeh, Nagi Shoja, Ghasem Tohidi (2003) A method for solvong 0-1 multiple objective liner programming problem using DEA. Journal of the Operations Research Society of Japan (2003), 46 (2): 189-202. http://www.orsj.or.jp/~archive/pdf/e_mag/Vol.46_02_189.pdf

[52] Ekta Jain, Kalpana Dahiya, Vanita Verma (2018): Integer quadratic fractional programming problems with bounded variables, Annals of Operations Research (October 2018) 269: pp. 269-295.

[53] N. K. Jain, V. K. Jain, K. Deb (2007). Optimization of process parameters of mechanical type advanced machining processes using genetic algorithms. International Journal of Machine Tools and Manufacture 47 (2007), 900-919.

[54] Michael Junger, Thomas M. Liebling, Dennis Naddef, George L. Nemhauser, William R. Pulleybank, Gerhart Reinelt, Giovanni Rinaldi, Lawrence A. Wolsey–Editors, 50 Years of Integer Programming 1958-2008. Berlin: Springer, 2010.

[55] Adhe Kania, Kuntjoro Adji Sidarto (2016). Solving mixed integer nonlinear programming problems using spiral dynamics optimization algorithm. AIP Conference Proceedings 1716, 020004 (2016).  29, 2016.  https://doi:org/10.1063/1.4942987.

[56] Reena Kapoor, S. R. Arora (2006). Linearization of a 0-1 quadratic fractional programming problem: OPSEARCH of the Operational Research Society of India (2006), 43(2):190-207.

[57] M. G. M. Khan, E. A. Khan, M. J. Ahsan (2003). An optimal multivariate stratified sampling design using dynami programming. Australian and New Zealand Journal of Statistics, vol. 45, no. 1, 2003, pp. 107-113.

[58] M. G. M. Khan, T. Maiti, M. J. Ahsan (2010). An optimal multivariate stratified sampling design using auxiliary information: an integer solution using goal programming approach. Journal of Official Statistics, vol. 26, no. 4, 2010, pp. 695-708.

[59] A. H. Land, A. G. Doig, An Automatic Method of Solving Discrete Programming Problems. Econometrica, Vol. 28, No. 3 (Jul., 1960), pp. 497-520.

[60]  E. L. Lawler, M. D. Bell, A Method for Solving Discrete Optimization Problems. Operations Research, Vol. 14, No. 6 (Nov.-Dec., 1966), pp. 1098-1112.

[61]  Sven Leyffer (December 1993)  Deterministic Methods for Mixed Integer Nonlinear Progrmming, PhD Thesis, Department of Mathematics and Computer Science, University of Dundee, Dundee, December 1993.  (One can directly read this on Google.)

[62]  S. Leyffer, J. Linderoth, J. Luedtke, A. Miller, T. Munson (2009), Applications and algorithms for mixed integer nonlinear programming, J. of Physics: Conference Series 180 (2009) 012014, IOP Publishing.  (One can directly read this on Google.)

[63]  Yugui Li, Yanxu Wei, Yantao Chu (2015), Research on solving systems of nonlinear equations based on improved PSO, Mathematical Problems in Engineering, volume 2015, article ID 727218, 13 pages, Hindawi Publishing Corporration.   One can directly read this article on Google.

[64]  F. H. F. Liu, C. C. Huang, Y. L. Yen (2000): Using DEA to obtain efficient solutions for multi-objective 0-1 linear programs. European Journal of Operational Research 126 (2000) 51-68.

[65] Gia-Shi Liu (2006), A combination method for reliability-redundancy optimization, Engineering Optimization, 38:04, 485-499.

[66] Yubao Liu, Guihe Qin (2014), A hybrid TS-DE algorithm for reliability redundancy optimization problem, Journal of Computers, 9, No. 9, September 2014, pp. 2050-2057.

[67] Rein Luus (1975). Optimization of System Reliability by a New Nonlinear Integer Programming Procedure. IEEE Transactions on Reliability, Vol. R-24, No. 1, April 1975, pp. 14-16.

[68] Milos Madic, Miroslav Radovanovic (2014). Optimization of machining processes using pattern search algorithm. International Journal of Industrial Engineering Computations 5 (2014) 223-234. Homepage: http://www.GrowingScience.com/ijiec

[69] F. Masedu, M Angelozzi (2008). Modelling optimum fraction assignment in the 4X100 m relay race by integer linear programming. Italian Journal of Sports Sciences, Anno 13, No. 1, 2008, pp. 74-77.

[70] Mohamed Arezki Mellal, Enrico Zio (2016). A Guided Stochastic Fractal Search Approach for System Reliability Optimization. Reliability Engineering and System Safety 152 (2016) 213-227.

[71] Mohamed Arezki Mellal, Edward J. Williams (2016). Parameter optimization of advanced machining processes using cuckoo optimization algorithm and hoopla heuristic. Journal of Intelligent Manufacturing (2016) 27 (5): 927-942.

[72] Mohamed Arezki Mellal, Edward J. Williams (2018). Large-scale reliability-redundancy allocation optimization problem using three soft computing methods. In Mangey Ram, Editor, in Modeling and simulation based analysis in reliability engineering. Published July 2018, CRC Press.

[73] Microsoft Corp., BASIC, Second Edition (May 1982), Version 1.10. Boca Raton, Florida: IBM Corp., Personal Computer, P. O. Box 1328-C, Boca Raton, Florida 33432, 1981.

[74]  Joaquin Moreno, Miguel Lopez, Raquel Martinez (2018)  A new method for solving  nonlinear systems of equations that is based on functional iterations, Open Physics 2018; 16: 605-630.    

[75] Yuji Nakagawa, Mitsunori Hikita, Hiroshi Kamada (1984). Surrogate Constraints for Reliability Optimization Problems with Multiple Constraints. IEEE Transactions on Reliability, Vol. R-33, No. 4, October 1984, pp. 301-305.

[76] Subhash C. Narula, H. Roland Weistroffer (1989). A flexible method for nonlinear criteria decisionmaking problems. Ieee Transactions on Systems, Man and Cybernetics, vol. 19 , no. 4, July/August 1989, pp. 883-887.

[77] C. E. Nugent, T. E. Vollmann, J. Ruml (1968), An Experimental Comparison of Techniques for the Assignment of Facilities to Locations, Operations Research 16 (1968), pp. 150-173.

[78] A. K. Ojha, K. K. Biswal (2010). Multi-objective geometric programming problem with weighted mean method. (IJCSIS) International Journal of Computer Science and Information Security, vol. 7, no. 2, pp. 82-86, 2010.

[79] Rashmi Ranjan Ota, jayanta kumar dASH, A. K. Ojha (2014). A hybrid optimization techniques for solving non-linear multi-objective optimization problem. amo-advanced modelling and optimization, volume 16,number 1, 2014.

[80] Rashmi Ranjan Ota, A. K. Ojha (2015). A comparative study on optimization techniques for solving multi-objective geometric programming problems. Applied mathematical sciences, vol. 9, 2015, no. 22, 1077-1085. http://sites.google.com/site/ijcsis/

[81] R. R. Ota, J. C. Pati, A. K. Ojha (2019). Geometric programming technique to optimize power distribution system. OPSEARCH of the Operational Research Society of India (2019), 56, pp. 282-299.

[82] OPTI Toolbox, Mixed Integer Nonlinear Program (MINLP). https://www.inverseproblem.co.nz/OPTI/index.php/Probs/MINLP

[83] Panos Y. Papalambros, Douglass J. Wilde, Principles of Optimal Design, Second Edition. Cambridge University Press, 2000.

[84]  K. E. Parsopoulos, M. N. Vrahatis, Particle swarm optimization method for constrained optimization.  One can directly read this on Google Scholar.

[85]  O. Perez, I. Amaya, R. Correa (2013), Numerical solution of certain exponential and nonlinear diophantine systems of equations by using a discrete particles swarm optimization algorithm.  Applied Mathematics and Computation, Volume 225, 1 December, 2013, pp. 737-746.

[86] Ciara Pike-Burke. Multi-Objective Optimization. https://www.lancaster.ac.uk/pg/pikeburc/report1.pdf.

[87] Yashpal Singh Raghav, Irfan Ali, Abdul Bari (2014) Multi-Objective Nonlinear Programming Problem Approach in Multivariate Stratified Sample Surveys in the Case of the Non-Response, Journal of Statitiscal Computation ans Simulation 84:1, 22-36, doi:10.1080/00949655.2012.692370.

[88] R. V. Rao, P. J. Pawar, J. P. Davim (2010). Parameter optimization of ultrasonic machining process using nontraditional optimization algorihms. Materials and Manufacturing Processes, 25 (10),1120-1130.

[89]  W. Rheinboldt,  Some Nonlinear Testproblems.  One can google this 10-page paper.

[90]  John Rice, Numerical Methods, Software, and Analysis, Second Edition, 1993, Academic Press.

[91] H. S. Ryoo, N. V. Sahinidis (1995), Global Optimization of Nonconvex NLPs and MINLPs with Applications in Process Design. Computers and Chemical Engineering, Vol. 19, No. 5, pp. 551-566, 1995.

[92] Ali Sadollah, Hadi Eskandar, Joong Hoon Kim (2015). Water cycle algorithm for solvinfg constrained multi-objective optimization problems. Applied Soft Computing 27 (2015) 279-298.

[93] Shafiullah, Irfan Ali, Abdul Bari (2015). Fuzzy geometric programming approach in multi-objective multivariate stratified sample surveys in presence of non-respnse, International Journal of Operations Research, Vol. 12, No. 2, pp. 021-035 (2015).

[94] Mordechai Shacham (1986). Numerical solution of constrained nonlinear algebraic equations, International Journal for Numerical Methods in Engineering, Vol. 23, pp. 1455-1481 (1986).  (One can read this via Google.)

[95] M. Shacham,  R. S. H. Mah  (1978).  A Newton Type Linearization Method for Solution of Nonlinear Equations, Computers and Chemical Engineering, Vol. 2, pp. 64-66 (1978).  (One can read this via Google.)

[96]  Yi Shang (1997),  Global search methods for solving nonlinear optimization problems, Ph. D. Thesis, University of Illinois at Urbana-Champaign, 1997.

[97] Vikas Sharma (2012). Multiobjective integer nonlinear fractional programming problem: A cutting plane approach, OPSEARCH of the Operational Research Society of India (April-June 2012), 49(2):133-153.

[98] Vikas Sharma, Kalpana Dahiya, Vanita Verma (2017). A ranking algorithm for bi-objective quadratic fractional integer programming problems, Optimization, 66:11, 1913-1929.

[99]  K. A. Sidarto, Adhe Kania (2015),  Finding all solutions of systems of nonlinear equations using spiral dynamics inspired optimization with clustering,

Journal of Advanced Computational Intelligence and Intelligent Informatics, pp. 697-708, vol. 19, number 5, 2015.  (One can directly read this via Google.)

[100] G. Stephanopoulos, A. W. Westerberg, The Use of Hestenes’ Method of Multipliers to Resolve Dual Gaps in Engineering System Optimization. Journal of Optimization Theory and Applications, Vol.15, No. 3, pp. 285-309, 1975.

[101]  Sonja Surjanovic, Derek Bingham (2013), Virtual library of simulation experiments: test functions and datasets.  Simon Fraser University, Burnaby, BC, Canada, 2013.  (One can directly read this on Google.)

[102] Hardi Tambunan, Herman Mawengkang (2016). Solving Mixed Integer Non-Linear Programming Using Active Constraint. Global Journal of Pure and Applied Mathematics, Volume 12, Number 6 (2016), pp. 5267-5281. http://www.ripublication.com/gjpam.htm

[103] Mohamed Tawhid, Vimal Savsani (2018). Epsilon-constraint heat transfer search (epsilon-HTS) algorithm for solvinfg multi-objective engineering design problems. journal of computational design and engineering 5 (2018) 104-119.

[104] Frank A. Tillman, Ching-Lai Hwang, Way Kuo (1977). Determining Component Reliability and Redundancy for Optimun System Reliability. IEEE Transactions on Reliability, Vol. R-26, No. 3, Augusr 1977, pp. 162-165.

[105] Rahul Varshney, Srikant Gupta, Irfan Ali (2017). An optimum multivariate-multiobjective stratified samplinr design: fuzzy programming approach. Pakistan Journal of Statistics and Operations Research, pp. 829-855, December 2017

[106] V. Verma, H. C. Bakhshi, M. C. Puri (1990) Ranking in integer linear fractional programming problems ZOR – Methods and Models of Operations Research (1990) 34:325-334.

[107] Tawan Wasanapradit, Nalinee Mukdasanit, Nachol Chaiyaratana, Thongchai Srinophakun (2011). Solving mixed-integer nonlinear programming problems using improved genetic algorithms. Korean Joutnal of Chemical Engineering 28 (1):32-40 January 2011.

[108] Rahul Varshney, Mradula (2019 May 25). Optimum allocation in multivariate stratified sampling design in the presence of nonresponse with Gamma cost function, Journal of Statistical Computation and Simulation (2019) 89:13, pp. 2454-2467.

[109] Rahul Varshney, Najmussehar, M. J. Ahsan (2012). An optimum multivariate stratified double sampling design in presence of non-response, Optimization Letters (2012) 6: pp. 993-1008.

[110] Rahul Varshney, M. G. M. Khan, Ummatul Fatima, M. J. Ahsan (2015). Integer compromise allocation in multivariate stratified surveys, Annals of Operations Research (2015) 226:659-668.

[111] Rahul Varshney, Srikant Gupta, Irfan Ali (2017). An optimum multivariate-multiobjective stratified sampling design: fuzzy programming approach. Pakistan Journal of Statistics and Operations Research, pp. 829-855, December 2017

[112] V. Verma, H. C. Bakhshi, M. C. Puri (1990) Ranking in integer linear fractional programming problems ZOR – Methods and Models of Operations Research (1990) 34:325-334.

[113] Tawan Wasanapradit, Nalinee Mukdasanit, Nachol Chaiyaratana, Thongchai Srinophakun (2011). Solving mixed-integer nonlinear programming problems using improved genetic algorithms. Korean Joutnal of Chemical Engineering 28 (1):32-40 January 2011.

[114]  Eric W. Weisstein, “Diophantine Equation–8th Powers.”  https://mathworld.wolfram.com/DiophantineEquation8thPowers.html.

[115]  Eric W. Weisstein, “Euler’s Sum of 0wers Conjecture.”  https://mathworld.wolfram.com/EulersSumofPowersConjecture.html.

[116]  Eric W. Weisstein, “Diophantine Equation–5th Powers.”  https://mathworld.wolfram.com/DiophantineEquation5thPowers.html.

[117]  Eric W. Weisstein, “Diophantine Equation–10th Powers.”  https://mathworld.wolfram.com/DiophantineEquation10thPowers.html.

[118]  Eric W. Weisstein, “Diophantine Equation–9th Powers.”  https://mathworld.wolfram.com/DiophantineEquation9thPowers.html.

[119]  Rick Wicklin (2018), Solving a system of nonlinear equations with SAS.  blogs.sas.com>iml>2018/02/28.  One can directly read this on Google.

[120] Wikipedia, QB64, https://en.wikipedia.org/wiki/QB64.

[121] Wikipedia, Test functions for optimization, https://en.wikipedia.org/wiki/Test_functions_for_optimization.

[122] Jsun Yui Wong (2014, March 13).   The Domino Method Applied to a System of Four Simultaneous Nonlinear Equations.  Retrieved from http://myblogsubstance.typepad.com/substance/2014/03.

[123] Zhongkai Xu, Way Kuo, Hsin-Hui Lin (1990). Optimization Limits in Improving System Reliability. IEEE Transactions on Reliability, Vol. 39, No. 1, 1990 April, pp. 51-60.

A Test Problem for Constrained Optimization       

Jsun Yui Wong

The computer program listed below seeks to solve the following test problem in Parsopoulos and Vrahatis [84, Test Problem 2]:

minimize           f=    (X(1) – 10) ^ 3 + (X(2) – 20) ^ 3

subject  to  

    100 – (X(1) – 5) ^ 2 – (X(2) – 5) ^ 2 <= 0,

    (X(1) – 6) ^ 2 + (X(2) – 5) ^ 2 – 82.81 <= 0

           13 <= X(1) <=100,            

        0 <= X(2) <=100,            

The best known solution is f*= -6961.81381.

0 REM      DEFDBL A-Z

2 REM    DEFINT K

3 DIM B(99), N(99), A(100255), H(99), L(99), U(99), X(100250), D(111), P(511), PS(33), J(30003), J44(30003), KKQQ(30003), KLX(30003), W(10111)

12 FOR JJJJ = -32000 TO 32000 STEP .01

    13 RANDOMIZE JJJJ

    16 M = -1D+37

    21 A(1) = RND * 50

    22 A(2) = RND * 50

    128 FOR I = 1 TO 300000

        129 FOR KKQQ = 1 TO 2

            130 X(KKQQ) = A(KKQQ)

        131 NEXT KKQQ

        135 FOR IPP = 1 TO (1 + FIX(RND ^ 5 * 5))

            148 J = 1 + FIX(RND * 2)

            153 REM    GOTO 162

            154 IF RND < .5 THEN GOTO 156 ELSE GOTO 162

            156 REM

            157 R = (1 – RND * 2) * (A(J))

            160 X(J) = A(J) + (RND ^ (RND * 30)) * R

            161 GOTO 165

            162 REM    IF RND < .16666 THEN X(J) = A(J) – FIX(1 + RND * 2.3) ELSE IF RND < .33333 THEN X(J) = A(J) + FIX(1 + RND * 2.3) ELSE IF RND < .5 THEN X(J) = A(J) – FIX(1 + RND * 20.3) ELSE IF RND < .66666 THEN X(J) = A(J) + FIX(1 + RND * 20.3) ELSE IF RND < .83333 THEN X(J) = A(J) – FIX(1 + RND * 200.3) ELSE X(J) = A(J) + FIX(1 + RND * 200.3)

            164 IF RND < .5 THEN X(J) = A(J) – FIX(1 + RND * .3) ELSE X(J) = A(J) + FIX(1 + RND * .3)

        165 NEXT IPP

        175 IF 100 – (X(1) – 5) ^ 2 – (X(2) – 5) ^ 2 > 0## THEN 1670

        177 IF (X(1) – 6) ^ 2 + (X(2) – 5) ^ 2 – 82.81 > 0## THEN 1670

        197 IF X(1) < 13 THEN 1670

        198 IF X(1) > 100 THEN 1670

        199 IF X(2) < 0 THEN 1670

        200 IF X(2) > 100 THEN 1670

        959 P = -(X(1) – 10) ^ 3 – (X(2) – 20) ^ 3

        1111 IF P <= M THEN 1670

        1452 M = P

        1454 FOR KLX = 1 TO 2

            1459 A(KLX) = X(KLX)

        1460 NEXT KLX

        1557 GOTO 128

    1670 NEXT I

    1677 IF M < -9999 THEN 1999

    1908 PRINT A(1), A(2), M, JJJJ

1999 NEXT JJJJ

This BASIC computer program was run with qb64v1000-win [120].  The complete output of one run through JJJJ =  -31999.66 is shown below:

14.09501      .8429765      6961.796      -31999.99

14.09501      .8429726      6961.801      -31999.83

14.09501      .8429765      6961.796      -31999.79

14.09501      .8429765      6961.796      -31999.77

14.09501      .8429706      6961.803      -31999.7

14.09501      .8429706      6961.803      -31999.67

14.095          .842969        6961.805      -31999.66

Above there is no rounding by hand; it is just straight copying by hand from the monitor screen.

The system properties of the computer system used include Intel Core 2 Duo CPU   E8400 @ 3.00GHz  3.00GHz, 4.00GB of RAM, and qb64v1000-win [120].  The wall-clock time (not CPU time) for obtaining the output through JJJJ = -31999.66 was 16 seconds, counting from “Starting program…”.  One can compare the computational results above with those in Parsopoulos and Vrahatis [84, Test Problem 2, Table 1].  

Acknowledgement

I would like to acknowledge the encouragement of Roberta Clark and Tom Clark.

References

[1]  M. Abdollahi, A. Isazadeh, D. Abdollahi (2013), Imperialist competitive algorithm for solving systems of nonlinear equations, Computers and Mathematics with Applications, vol. 65, number 12, pp. 1894-1908, 2013.  (One can directly read this on Google.)

[2] Siby Abraham, Sugata Sanyal, Mukund Sanglikar (2010), Particle Swarm Optimisation Based Diophantine Equation Solver,  Int. J. of Bio-Inspired Computation, 2 (2), 100-114, 2010. 

[3] Siby Abraham, Sugata Sanyal, Mukund Sangrikar (2013), A Connectionist Network Approach to Find Numerical Solutions of Diophantine Equations, Int. J. of Engg. Science and Mgmt., Vol. III, Issue 1, January-June 2013.

[4] Andre R. S. Amaral (2006), On the Exact Solution of a Facility Layout Problem. European Journal of Operational Research 173 (2006), pp. 508-518.

[5] Andre R. S. Amaral (2008), An Exact Approach to the One-Dimensional Facility Layout Problem. Operations Research, Vol. 56, No. 4 (July-August, 2008), pp. 1026-1033.

[6] Andre R. S. Amaral (2011), Optimal Solutions for the Double Row Layout Problem. Optimization Letters, DOI 10.1007/s11590-011-0426-8, published on line 30 November 2011, Springer-Verlag 2011.

[7] Andre R. S. Amaral (2012), The Corridor Allocation Problem. Computers and Operations Research 39 (2012), pp. 3325-3330.

[8]  George Anescu (2016), Gradual and cumulative improvements to the classical differential evolution scheme through experiments, Analele  Universitatii de Vest, Timisoara, Seria Mathematica–Informatica, Vol. LIV, 2, (2016), 13-35.   (One can directly read this on Google.)

[9] Oscar Augusto, Bennis Fouad, Stephane Caro (2012). A new method for decision making in multi-objective optimization problems. Pesquisa Operacional, Sociedade Brasileira de Pesquisa Operacional, 2012 32 (3), pp.331-369.

[10] Miguel F. Anjos, Anthony Vannelli, Computing Globally Optimal Solutions for Single-Row Layout Problems Using Semidefinite Programming and Cutting Planes. INFORMS Journal on Computing, Vol. 20, No. 4, Fall 2008, pp. 611-617.

[11] David L. Applegate, Robert E. Bixby, Vasek Chvatal, William J. Cook, The Traveling Salesman Problem: A Computational Study. Princeton and Oxford: Princeton University Press, 2006.

[12] Ritu Arora, S. R. Arora (2015). A cutting plane approach for multi-objective integer indefinite quadratic programming problem: OPSEARCH of the Operational Research Society of India (April-June 2015), 52(2):367-381.

[13]   H. Bernau (1990).  Active constraint strategies in optimization.  Geographical data inversion methods and applications, pp. 15-31.

[14] Victor Blanco, Justo Puerto (2011). Some algebraic methods for solving multiobjective polynomial integer programs, Journal of Symbolic Computation, 46 (2011), pp. 511-533.

[15] Jerome Bracken, Garth P. McCormick, Selected Applications of Nonlinear Programming. New York: John Wiley and Sons, Inc., 1968.

[16] Neima Brauner, Moredechai Shacham, Michael Cutlip (winter 1996)  Computational Results – How Reliable Are They? Chemical Engineering Educatiobn, 30, 1,  pp. 20-25 (1996).  (One can read this via Google.)

[17] Richard L. Burden, Douglas J. Faires, Annette M. Burden, Numerical Analysis, Tenth Edition, 2016, Cengage Learning.

[18] R. C. Carlson and G. L. Nemhauser, Scheduling To Minimize Interaction Cost. Operations Research, Vol. 14, No. 1 (Jan. – Feb., 1966), pp. 52-58.

[19] Ta-Cheng Chen (2006). IAs based approach for reliability redundany allocation problems. Applied Mathematics and Computation 182 (2006) 1556-1567.

[20]  Ward Cheney, David Kincaid (2008), Numerical Mathematics and Computing, Sixth Edition, 2008, Thomsom Brooks/Coles.

[21] Leandro dos Santos Coelho (2009), Self-Organizing Migrating Strategies Applied to Reliability-Redundany Optimization of Systems. IEEE Transactions on Reliability, Vol. 58, No. 3, 2009 September, pp. 501-519.

[22]  A. J. Conejo, L. Baringo (2018), Power System Operations, Springer International Publishing, 2018.

[23] William Conley (1981). Optimization: A Simplified Approach. Published 1981 by Petrocelli Books in New York.

[24] Lino Costa, Pedro (2001). Evolutionary algorithms approach to the solution of mixed integer non-linear programming problems. Computers and Chemical Engineering, Vol. 25, pp. 257-266, 2001.

[25] Michael B. Cutlip, Mordechai Shacham (1998), POLYMATH verssion 4.1 user-friendly numerical analysis programs.  http://www.polymath-software.com.

[26] George B. Dantzig, Discrete-Variable Extremum Problems. Operations Research, Vol. 5, No. 2 (Apr., 1957), pp. 266-277.

[27] Pintu Das, Tapan Kumar Roy (2014). Multi-objective geometric programming and its application in gravel box problem. Journal of global research in computer science volume 5. no.7, July 2014. http://www.jgrcs.info

[28] Kalyanmoy Deb, Amrit Pratap, Subrajyoti Moitra (2000). Mechanical component design for multi objectives using elitist non-dominated sorting GA. Proceedings of the Parallel Probl.Solv. Nat. PPSN VI Conference, Paris, France, pp. 859-868 (2000). (Please see pp. 1-10 in Technical Report No. 200002 via Google search.)

[29] Kusum Deep, Krishna Pratap Singh, M. L. Kansal, C. Mohan (2009), A real coded genetic algorithm for solving integer and mixed integer optimization problems. Applied Mathematics and Computation 212 (2009) 505-518.

[30] Wassila Drici, Mustapha Moulai (2019): An exact method for solving multi-objective integer indefinite quadratic programs, Optimization Methods and Software.

[31] R. J. Duffin, E. L. Peterson, C. Zener (1967), Geometric Programming. John Wiley, New York (1967).

[32]  Simge Ekiz, Pakize Erdogmus, Busra Ozgur (2017 November), Solving constrained  optimization problems with sine-cosine algorithm, Periodicals of Engineering and Natural Sciences, vol. 5, no. 3, November 2017, pp.378~386.  http://pen.ius.edu.ba.  

[33] C. A. Floudas, A. R. Ciric (1989), Strategies for Overcoming Uncertainties in Heat Exchanger Network Synthesis. Computers and Chemical Engineering, Vol 13, No. 10, pp. 1133-1152, 1989.

[34] C. A. Floudas, A. Aggarwal, A. R. Ciric (1989), Global Optimum Search for Nonconvex NLP and MINLP Problems. Computers and Chemical Engineering, Vol 13, No. 10, pp. 1117-1132, 1989.

[35] C. A. Floudas, A. Aggarwal, A. R. Ciric (1989), Global Optimum Search for Nonconvex NLP and MINLP Problems. Computers and Chemical Engineering, Vol 13, No. 10, pp. 1117-1132, 1989.

[36] C. A. Floudas, P. M. Pardalos, A Collection of Test Problems for Constrained Global Optimization Algorithms. Springer-Verlag, 1990.  

[37] C. A. Floudas(1995),  Nonlinear and mixed-integer optimization:  fundamentals and applications.  Oxford University Press, 1995.

[38] Diptesh Ghosh, Ravi Kothari, Population Heuristics for the Corridor Allocation Problem, W.P. No. 2012-09-02, September 2012. Retrieved on September 14 2012 from Google search.

[39]  Crina Grosan, Ajith Abraham  (2008),    A New Approach for Solving Nonlinear Equations Systems.  IEEE Transactions on Systems, Man and Cybernetics–Part A:  Systems and Humans, Volume 38, Number 3, May 2008, pp. 698-714.

[40]  Crina Grosan, Ajith Abraham  (2008), Multiple Solutions for a System pf Nonlinear Equations. International Journal of Innovative Computing, Information and Control, Volume x, Number x, x 2008.

[41]  GUO Chuang-xin,  HU Jia-sheng, YE Bin, CAO Yi-jia (2004), Swarm intelligence for mixed-variable design optimization, J. of Zhejiang University SCIENCE 2004 5(7):851-860.  One can directly read this on Google.

[42] Neha Gupta, Irfan Ali, Abdul Bari (2013). An optimal chance constraint multivariate stratified sampling design using auxiliary infotmation. Journal of Mathematical Modelling and Algorithms, January 2013.

[43] R. Gupta, R. Malhotra (1995). Multi-criteria integer linear fractional programming problem, Optimization, 35:4, 373-389.

[44] M. Hashish, M. P. duPlessis (1979). Prediction equations relating high velocity jet cutting performance to stand-off-distance and multipasses. Transactions of ASME: Journal of Engineering for Industry 101 (1979) 311-318.

[45]  Abdel-Rahman Hedar, Masao Fukushima (2006)  Derivative-free simulated annealing method for constrained continuous optimization, J. of Global Optimization, January 2006.   (One can directly view this on Google.)

[46]  www – optima.amp.i.kyoto – u.ac.jp/member/student/hedar/Hedar_fil…

[47]  Frederick S. Hillier, Gerald J. Liberman (1990), Introduction to Mathematical Programming, McGraw-Hill Publishing Company, 1990.

[48] Willi Hock, Klaus Schittkowski, Test Examples for Nonlinear Programming Codes. Berlin: Springer-Verlag, 1981.

[49] Sana Iftekhar, M. J. Ahsan, Qazi Mazhar Ali (2015). An optimum multivariate stratified sampling design. Research Journal of Mathematical and Statistical Sciences, vol. 3(1),10-14, January (2015).

[50] Philipp Hungerlaender (April 2012), Single-Row Equidistant Facility Layout as a Special Case of Single-Row Facility Layout. Retrieved from http://www.optimization-online.org./DB_HTML/2012/04/3432.html

[51] Golam Reza Jahanshahloo, Farhad Hosseinzadeh, Nagi Shoja, Ghasem Tohidi (2003) A method for solvong 0-1 multiple objective liner programming problem using DEA. Journal of the Operations Research Society of Japan (2003), 46 (2): 189-202. http://www.orsj.or.jp/~archive/pdf/e_mag/Vol.46_02_189.pdf

[52] Ekta Jain, Kalpana Dahiya, Vanita Verma (2018): Integer quadratic fractional programming problems with bounded variables, Annals of Operations Research (October 2018) 269: pp. 269-295.

[53] N. K. Jain, V. K. Jain, K. Deb (2007). Optimization of process parameters of mechanical type advanced machining processes using genetic algorithms. International Journal of Machine Tools and Manufacture 47 (2007), 900-919.

[54] Michael Junger, Thomas M. Liebling, Dennis Naddef, George L. Nemhauser, William R. Pulleybank, Gerhart Reinelt, Giovanni Rinaldi, Lawrence A. Wolsey–Editors, 50 Years of Integer Programming 1958-2008. Berlin: Springer, 2010.

[55] Adhe Kania, Kuntjoro Adji Sidarto (2016). Solving mixed integer nonlinear programming problems using spiral dynamics optimization algorithm. AIP Conference Proceedings 1716, 020004 (2016).  29, 2016.  https://doi:org/10.1063/1.4942987.

[56] Reena Kapoor, S. R. Arora (2006). Linearization of a 0-1 quadratic fractional programming problem: OPSEARCH of the Operational Research Society of India (2006), 43(2):190-207.

[57] M. G. M. Khan, E. A. Khan, M. J. Ahsan (2003). An optimal multivariate stratified sampling design using dynami programming. Australian and New Zealand Journal of Statistics, vol. 45, no. 1, 2003, pp. 107-113.

[58] M. G. M. Khan, T. Maiti, M. J. Ahsan (2010). An optimal multivariate stratified sampling design using auxiliary information: an integer solution using goal programming approach. Journal of Official Statistics, vol. 26, no. 4, 2010, pp. 695-708.

[59] A. H. Land, A. G. Doig, An Automatic Method of Solving Discrete Programming Problems. Econometrica, Vol. 28, No. 3 (Jul., 1960), pp. 497-520.

[60]  E. L. Lawler, M. D. Bell, A Method for Solving Discrete Optimization Problems. Operations Research, Vol. 14, No. 6 (Nov.-Dec., 1966), pp. 1098-1112.

[61]  Sven Leyffer (December 1993)  Deterministic Methods for Mixed Integer Nonlinear Progrmming, PhD Thesis, Department of Mathematics and Computer Science, University of Dundee, Dundee, December 1993.  (One can directly read this on Google.)

[62]  S. Leyffer, J. Linderoth, J. Luedtke, A. Miller, T. Munson (2009), Applications and algorithms for mixed integer nonlinear programming, J. of Physics: Conference Series 180 (2009) 012014, IOP Publishing.  (One can directly read this on Google.)

[63]  Yugui Li, Yanxu Wei, Yantao Chu (2015), Research on solving systems of nonlinear equations based on improved PSO, Mathematical Problems in Engineering, volume 2015, article ID 727218, 13 pages, Hindawi Publishing Corporration.   One can directly read this article on Google.

[64]  F. H. F. Liu, C. C. Huang, Y. L. Yen (2000): Using DEA to obtain efficient solutions for multi-objective 0-1 linear programs. European Journal of Operational Research 126 (2000) 51-68.

[65] Gia-Shi Liu (2006), A combination method for reliability-redundancy optimization, Engineering Optimization, 38:04, 485-499.

[66] Yubao Liu, Guihe Qin (2014), A hybrid TS-DE algorithm for reliability redundancy optimization problem, Journal of Computers, 9, No. 9, September 2014, pp. 2050-2057.

[67] Rein Luus (1975). Optimization of System Reliability by a New Nonlinear Integer Programming Procedure. IEEE Transactions on Reliability, Vol. R-24, No. 1, April 1975, pp. 14-16.

[68] Milos Madic, Miroslav Radovanovic (2014). Optimization of machining processes using pattern search algorithm. International Journal of Industrial Engineering Computations 5 (2014) 223-234. Homepage: http://www.GrowingScience.com/ijiec

[69] F. Masedu, M Angelozzi (2008). Modelling optimum fraction assignment in the 4X100 m relay race by integer linear programming. Italian Journal of Sports Sciences, Anno 13, No. 1, 2008, pp. 74-77.

[70] Mohamed Arezki Mellal, Enrico Zio (2016). A Guided Stochastic Fractal Search Approach for System Reliability Optimization. Reliability Engineering and System Safety 152 (2016) 213-227.

[71] Mohamed Arezki Mellal, Edward J. Williams (2016). Parameter optimization of advanced machining processes using cuckoo optimization algorithm and hoopla heuristic. Journal of Intelligent Manufacturing (2016) 27 (5): 927-942.

[72] Mohamed Arezki Mellal, Edward J. Williams (2018). Large-scale reliability-redundancy allocation optimization problem using three soft computing methods. In Mangey Ram, Editor, in Modeling and simulation based analysis in reliability engineering. Published July 2018, CRC Press.

[73] Microsoft Corp., BASIC, Second Edition (May 1982), Version 1.10. Boca Raton, Florida: IBM Corp., Personal Computer, P. O. Box 1328-C, Boca Raton, Florida 33432, 1981.

[74]  Joaquin Moreno, Miguel Lopez, Raquel Martinez (2018)  A new method for solving  nonlinear systems of equations that is based on functional iterations, Open Physics 2018; 16: 605-630.    

[75] Yuji Nakagawa, Mitsunori Hikita, Hiroshi Kamada (1984). Surrogate Constraints for Reliability Optimization Problems with Multiple Constraints. IEEE Transactions on Reliability, Vol. R-33, No. 4, October 1984, pp. 301-305.

[76] Subhash C. Narula, H. Roland Weistroffer (1989). A flexible method for nonlinear criteria decisionmaking problems. Ieee Transactions on Systems, Man and Cybernetics, vol. 19 , no. 4, July/August 1989, pp. 883-887.

[77] C. E. Nugent, T. E. Vollmann, J. Ruml (1968), An Experimental Comparison of Techniques for the Assignment of Facilities to Locations, Operations Research 16 (1968), pp. 150-173.

[78] A. K. Ojha, K. K. Biswal (2010). Multi-objective geometric programming problem with weighted mean method. (IJCSIS) International Journal of Computer Science and Information Security, vol. 7, no. 2, pp. 82-86, 2010.

[79] Rashmi Ranjan Ota, jayanta kumar dASH, A. K. Ojha (2014). A hybrid optimization techniques for solving non-linear multi-objective optimization problem. amo-advanced modelling and optimization, volume 16,number 1, 2014.

[80] Rashmi Ranjan Ota, A. K. Ojha (2015). A comparative study on optimization techniques for solving multi-objective geometric programming problems. Applied mathematical sciences, vol. 9, 2015, no. 22, 1077-1085. http://sites.google.com/site/ijcsis/

[81] R. R. Ota, J. C. Pati, A. K. Ojha (2019). Geometric programming technique to optimize power distribution system. OPSEARCH of the Operational Research Society of India (2019), 56, pp. 282-299.

[82] OPTI Toolbox, Mixed Integer Nonlinear Program (MINLP). https://www.inverseproblem.co.nz/OPTI/index.php/Probs/MINLP

[83] Panos Y. Papalambros, Douglass J. Wilde, Principles of Optimal Design, Second Edition. Cambridge University Press, 2000.

[84]  K. E. Parsopoulos, M. N. Vrahatis, Particle swarm optimization method for constrained optimization.  One can directly read this on Google Scholar.

[85]  O. Perez, I. Amaya, R. Correa (2013), Numerical solution of certain exponential and nonlinear diophantine systems of equations by using a discrete particles swarm optimization algorithm.  Applied Mathematics and Computation, Volume 225, 1 December, 2013, pp. 737-746.

[86] Ciara Pike-Burke. Multi-Objective Optimization. https://www.lancaster.ac.uk/pg/pikeburc/report1.pdf.

[87] Yashpal Singh Raghav, Irfan Ali, Abdul Bari (2014) Multi-Objective Nonlinear Programming Problem Approach in Multivariate Stratified Sample Surveys in the Case of the Non-Response, Journal of Statitiscal Computation ans Simulation 84:1, 22-36, doi:10.1080/00949655.2012.692370.

[88] R. V. Rao, P. J. Pawar, J. P. Davim (2010). Parameter optimization of ultrasonic machining process using nontraditional optimization algorihms. Materials and Manufacturing Processes, 25 (10),1120-1130.

[89]  W. Rheinboldt,  Some Nonlinear Testproblems.  One can google this 10-page paper.

[90]  John Rice, Numerical Methods, Software, and Analysis, Second Edition, 1993, Academic Press.

[91] H. S. Ryoo, N. V. Sahinidis (1995), Global Optimization of Nonconvex NLPs and MINLPs with Applications in Process Design. Computers and Chemical Engineering, Vol. 19, No. 5, pp. 551-566, 1995.

[92] Ali Sadollah, Hadi Eskandar, Joong Hoon Kim (2015). Water cycle algorithm for solvinfg constrained multi-objective optimization problems. Applied Soft Computing 27 (2015) 279-298.

[93] Shafiullah, Irfan Ali, Abdul Bari (2015). Fuzzy geometric programming approach in multi-objective multivariate stratified sample surveys in presence of non-respnse, International Journal of Operations Research, Vol. 12, No. 2, pp. 021-035 (2015).

[94] Mordechai Shacham (1986). Numerical solution of constrained nonlinear algebraic equations, International Journal for Numerical Methods in Engineering, Vol. 23, pp. 1455-1481 (1986).  (One can read this via Google.)

[95] M. Shacham,  R. S. H. Mah  (1978).  A Newton Type Linearization Method for Solution of Nonlinear Equations, Computers and Chemical Engineering, Vol. 2, pp. 64-66 (1978).  (One can read this via Google.)

[96]  Yi Shang (1997),  Global search methods for solving nonlinear optimization problems, Ph. D. Thesis, University of Illinois at Urbana-Champaign, 1997.

[97] Vikas Sharma (2012). Multiobjective integer nonlinear fractional programming problem: A cutting plane approach, OPSEARCH of the Operational Research Society of India (April-June 2012), 49(2):133-153.

[98] Vikas Sharma, Kalpana Dahiya, Vanita Verma (2017). A ranking algorithm for bi-objective quadratic fractional integer programming problems, Optimization, 66:11, 1913-1929.

[99]  K. A. Sidarto, Adhe Kania (2015),  Finding all solutions of systems of nonlinear equations using spiral dynamics inspired optimization with clustering,

Journal of Advanced Computational Intelligence and Intelligent Informatics, pp. 697-708, vol. 19, number 5, 2015.  (One can directly read this via Google.)

[100] G. Stephanopoulos, A. W. Westerberg, The Use of Hestenes’ Method of Multipliers to Resolve Dual Gaps in Engineering System Optimization. Journal of Optimization Theory and Applications, Vol.15, No. 3, pp. 285-309, 1975.

[101]  Sonja Surjanovic, Derek Bingham (2013), Virtual library of simulation experiments: test functions and datasets.  Simon Fraser University, Burnaby, BC, Canada, 2013.  (One can directly read this on Google.)

[102] Hardi Tambunan, Herman Mawengkang (2016). Solving Mixed Integer Non-Linear Programming Using Active Constraint. Global Journal of Pure and Applied Mathematics, Volume 12, Number 6 (2016), pp. 5267-5281. http://www.ripublication.com/gjpam.htm

[103] Mohamed Tawhid, Vimal Savsani (2018). Epsilon-constraint heat transfer search (epsilon-HTS) algorithm for solvinfg multi-objective engineering design problems. journal of computational design and engineering 5 (2018) 104-119.

[104] Frank A. Tillman, Ching-Lai Hwang, Way Kuo (1977). Determining Component Reliability and Redundancy for Optimun System Reliability. IEEE Transactions on Reliability, Vol. R-26, No. 3, Augusr 1977, pp. 162-165.

[105] Rahul Varshney, Srikant Gupta, Irfan Ali (2017). An optimum multivariate-multiobjective stratified samplinr design: fuzzy programming approach. Pakistan Journal of Statistics and Operations Research, pp. 829-855, December 2017

[106] V. Verma, H. C. Bakhshi, M. C. Puri (1990) Ranking in integer linear fractional programming problems ZOR – Methods and Models of Operations Research (1990) 34:325-334.

[107] Tawan Wasanapradit, Nalinee Mukdasanit, Nachol Chaiyaratana, Thongchai Srinophakun (2011). Solving mixed-integer nonlinear programming problems using improved genetic algorithms. Korean Joutnal of Chemical Engineering 28 (1):32-40 January 2011.

[108] Rahul Varshney, Mradula (2019 May 25). Optimum allocation in multivariate stratified sampling design in the presence of nonresponse with Gamma cost function, Journal of Statistical Computation and Simulation (2019) 89:13, pp. 2454-2467.

[109] Rahul Varshney, Najmussehar, M. J. Ahsan (2012). An optimum multivariate stratified double sampling design in presence of non-response, Optimization Letters (2012) 6: pp. 993-1008.

[110] Rahul Varshney, M. G. M. Khan, Ummatul Fatima, M. J. Ahsan (2015). Integer compromise allocation in multivariate stratified surveys, Annals of Operations Research (2015) 226:659-668.

[111] Rahul Varshney, Srikant Gupta, Irfan Ali (2017). An optimum multivariate-multiobjective stratified sampling design: fuzzy programming approach. Pakistan Journal of Statistics and Operations Research, pp. 829-855, December 2017

[112] V. Verma, H. C. Bakhshi, M. C. Puri (1990) Ranking in integer linear fractional programming problems ZOR – Methods and Models of Operations Research (1990) 34:325-334.

[113] Tawan Wasanapradit, Nalinee Mukdasanit, Nachol Chaiyaratana, Thongchai Srinophakun (2011). Solving mixed-integer nonlinear programming problems using improved genetic algorithms. Korean Joutnal of Chemical Engineering 28 (1):32-40 January 2011.

[114]  Eric W. Weisstein, “Diophantine Equation–8th Powers.”  https://mathworld.wolfram.com/DiophantineEquation8thPowers.html.

[115]  Eric W. Weisstein, “Euler’s Sum of 0wers Conjecture.”  https://mathworld.wolfram.com/EulersSumofPowersConjecture.html.

[116]  Eric W. Weisstein, “Diophantine Equation–5th Powers.”  https://mathworld.wolfram.com/DiophantineEquation5thPowers.html.

[117]  Eric W. Weisstein, “Diophantine Equation–10th Powers.”  https://mathworld.wolfram.com/DiophantineEquation10thPowers.html.

[118]  Eric W. Weisstein, “Diophantine Equation–9th Powers.”  https://mathworld.wolfram.com/DiophantineEquation9thPowers.html.

[119]  Rick Wicklin (2018), Solving a system of nonlinear equations with SAS.  blogs.sas.com>iml>2018/02/28.  One can directly read this on Google.

[120] Wikipedia, QB64, https://en.wikipedia.org/wiki/QB64.

[121] Wikipedia, Test functions for optimization, https://en.wikipedia.org/wiki/Test_functions_for_optimization.

[122] Jsun Yui Wong (2014, March 13).   The Domino Method Applied to a System of Four Simultaneous Nonlinear Equations.  Retrieved from http://myblogsubstance.typepad.com/substance/2014/03.

[123] Zhongkai Xu, Way Kuo, Hsin-Hui Lin (1990). Optimization Limits in Improving System Reliability. IEEE Transactions on Reliability, Vol. 39, No. 1, 1990 April, pp. 51-60.

Working on the Townsend Test Function (Modified) for Constrained Optimization        

    

Jsun Yui Wong

The computer program listed below seeks to solve the following test problem in the Wikipedia [121]:      

minimize                  –   (COS((X(1) – .1) * X(2))) ^ 2  –  X(1) * SIN(3 * X(1) + X(2))

subject  to  

 X(1) ^ 2 + X(2) ^ 2 <   (2 * COS(t) – .5 * COS(2 * t) – .25 * COS(3 * t) – .125 * COS(4 * t)) ^ 2    +  (2 * SIN(t)) ^ 2     

 where t = ATAN2(X(1), X(2)),  

-2.25<= X(1)     <=2.5,    

-2.5<= X(2)     <=1.75.    

One line 154, whicn is 154 IF RND < 1.5 THEN GOTO 156 ELSE GOTO 162.

0 REM      DEFDBL A-Z

2 REM    DEFINT K

3 DIM B(99), N(99), A(100255), H(99), L(99), U(99), X(100250), D(111), P(511), PS(33), J(30003), J44(30003), KKQQ(30003), KLX(30003), W(10111)

12 FOR JJJJ = -32000 TO 32000 STEP .01

    13 RANDOMIZE JJJJ

    16 M = -1D+37

    21 A(1) = RND * 2.5

    22 A(2) = RND * 1.75

    128 FOR I = 1 TO 300000

        129 FOR KKQQ = 1 TO 2

            130 X(KKQQ) = A(KKQQ)

        131 NEXT KKQQ

        135 FOR IPP = 1 TO (1 + FIX(RND ^ 5 * 5))

            148 J = 1 + FIX(RND * 2)

            153 REM    GOTO 162

            154 IF RND < 1.5 THEN GOTO 156 ELSE GOTO 162

            156 REM

            157 R = (1 – RND * 2) * (A(J))

            160 X(J) = A(J) + (RND ^ (RND * 30)) * R

            161 GOTO 165

            162 REM    IF RND < .16666 THEN X(J) = A(J) – FIX(1 + RND * 2.3) ELSE IF RND < .33333 THEN X(J) = A(J) + FIX(1 + RND * 2.3) ELSE IF RND < .5 THEN X(J) = A(J) – FIX(1 + RND * 20.3) ELSE IF RND < .66666 THEN X(J) = A(J) + FIX(1 + RND * 20.3) ELSE IF RND < .83333 THEN X(J) = A(J) – FIX(1 + RND * 200.3) ELSE X(J) = A(J) + FIX(1 + RND * 200.3)

            164 IF RND < .5 THEN X(J) = A(J) – FIX(1 + RND * .3) ELSE X(J) = A(J) + FIX(1 + RND * .3)

        165 NEXT IPP

        167 t = ATAN2(X(1), X(2))

        173 IF X(1) ^ 2 + X(2) ^ 2 – (2 * SIN(t)) ^ 2 – (2 * COS(t) – .5 * COS(2 * t) – .25 * COS(3 * t) – .125 * COS(4 * t)) ^ 2 >= 0## THEN 1670

        197 IF X(1) < -2.25 THEN 1670

        198 IF X(1) > 2.5 THEN 1670

        199 IF X(2) < -2.5 THEN 1670

        200 IF X(2) > 1.75 THEN 1670

        956 P = (COS((X(1) – .1) * X(2))) ^ 2 + X(1) * SIN(3 * X(1) + X(2))

        1111 IF P <= M THEN 1670

        1452 M = P

        1454 FOR KLX = 1 TO 2

            1459 A(KLX) = X(KLX)

        1460 NEXT KLX

        1557 GOTO 128

    1670 NEXT I

    1677 IF M < -9999 THEN 1999

    1908 PRINT A(1), A(2), M, JJJJ

1999 NEXT JJJJ

This BASIC computer program was run with qb64v1000-win [120].  The complete output of one run through JJJJ =  -31999.96 is shown below:

.6762341         2.412484E-07            1.606569         -32000

.6763346         3.818213E-08            1.606569         -31999.99

.6762227         2.046025E-07            1.606569         -31999.98

.6763284         1.086368E-08            1.606569         -31999.97

.6762912         2.447428E-07            1.606569         -31999.96

Above there is no rounding by hand; it is just straight copying by hand from the monitor screen.

The system properties of the computer system used include Intel Core 2 Duo CPU   E8400 @ 3.00GHz  3.00GHz, 4.00GB of RAM, and qb64v1000-win [120].  The wall-clock time (not CPU time) for obtaining the output through JJJJ = -31999.96 was 5 seconds, counting from “Starting program…”.  One can compare the computational results above with those in the Wikipedia [121].             

Acknowledgement

I would like to acknowledge the encouragement of Roberta Clark and Tom Clark.

References

[1]  M. Abdollahi, A. Isazadeh, D. Abdollahi (2013), Imperialist competitive algorithm for solving systems of nonlinear equations, Computers and Mathematics with Applications, vol. 65, number 12, pp. 1894-1908, 2013.  (One can directly read this on Google.)

[2] Siby Abraham, Sugata Sanyal, Mukund Sanglikar (2010), Particle Swarm Optimisation Based Diophantine Equation Solver,  Int. J. of Bio-Inspired Computation, 2 (2), 100-114, 2010. 

[3] Siby Abraham, Sugata Sanyal, Mukund Sangrikar (2013), A Connectionist Network Approach to Find Numerical Solutions of Diophantine Equations, Int. J. of Engg. Science and Mgmt., Vol. III, Issue 1, January-June 2013.

[4] Andre R. S. Amaral (2006), On the Exact Solution of a Facility Layout Problem. European Journal of Operational Research 173 (2006), pp. 508-518.

[5] Andre R. S. Amaral (2008), An Exact Approach to the One-Dimensional Facility Layout Problem. Operations Research, Vol. 56, No. 4 (July-August, 2008), pp. 1026-1033.

[6] Andre R. S. Amaral (2011), Optimal Solutions for the Double Row Layout Problem. Optimization Letters, DOI 10.1007/s11590-011-0426-8, published on line 30 November 2011, Springer-Verlag 2011.

[7] Andre R. S. Amaral (2012), The Corridor Allocation Problem. Computers and Operations Research 39 (2012), pp. 3325-3330.

[8]  George Anescu (2016), Gradual and cumulative improvements to the classical differential evolution scheme through experiments, Analele  Universitatii de Vest, Timisoara, Seria Mathematica–Informatica, Vol. LIV, 2, (2016), 13-35.   (One can directly read this on Google.)

[9] Oscar Augusto, Bennis Fouad, Stephane Caro (2012). A new method for decision making in multi-objective optimization problems. Pesquisa Operacional, Sociedade Brasileira de Pesquisa Operacional, 2012 32 (3), pp.331-369.

[10] Miguel F. Anjos, Anthony Vannelli, Computing Globally Optimal Solutions for Single-Row Layout Problems Using Semidefinite Programming and Cutting Planes. INFORMS Journal on Computing, Vol. 20, No. 4, Fall 2008, pp. 611-617.

[11] David L. Applegate, Robert E. Bixby, Vasek Chvatal, William J. Cook, The Traveling Salesman Problem: A Computational Study. Princeton and Oxford: Princeton University Press, 2006.

[12] Ritu Arora, S. R. Arora (2015). A cutting plane approach for multi-objective integer indefinite quadratic programming problem: OPSEARCH of the Operational Research Society of India (April-June 2015), 52(2):367-381.

[13]   H. Bernau (1990).  Active constraint strategies in optimization.  Geographical data inversion methods and applications, pp. 15-31.

[14] Victor Blanco, Justo Puerto (2011). Some algebraic methods for solving multiobjective polynomial integer programs, Journal of Symbolic Computation, 46 (2011), pp. 511-533.

[15] Jerome Bracken, Garth P. McCormick, Selected Applications of Nonlinear Programming. New York: John Wiley and Sons, Inc., 1968.

[16] Neima Brauner, Moredechai Shacham, Michael Cutlip (winter 1996)  Computational Results – How Reliable Are They? Chemical Engineering Educatiobn, 30, 1,  pp. 20-25 (1996).  (One can read this via Google.)

[17] Richard L. Burden, Douglas J. Faires, Annette M. Burden, Numerical Analysis, Tenth Edition, 2016, Cengage Learning.

[18] R. C. Carlson and G. L. Nemhauser, Scheduling To Minimize Interaction Cost. Operations Research, Vol. 14, No. 1 (Jan. – Feb., 1966), pp. 52-58.

[19] Ta-Cheng Chen (2006). IAs based approach for reliability redundany allocation problems. Applied Mathematics and Computation 182 (2006) 1556-1567.

[20]  Ward Cheney, David Kincaid (2008), Numerical Mathematics and Computing, Sixth Edition, 2008, Thomsom Brooks/Coles.

[21] Leandro dos Santos Coelho (2009), Self-Organizing Migrating Strategies Applied to Reliability-Redundany Optimization of Systems. IEEE Transactions on Reliability, Vol. 58, No. 3, 2009 September, pp. 501-519.

[22]  A. J. Conejo, L. Baringo (2018), Power System Operations, Springer International Publishing, 2018.

[23] William Conley (1981). Optimization: A Simplified Approach. Published 1981 by Petrocelli Books in New York.

[24] Lino Costa, Pedro (2001). Evolutionary algorithms approach to the solution of mixed integer non-linear programming problems. Computers and Chemical Engineering, Vol. 25, pp. 257-266, 2001.

[25] Michael B. Cutlip, Mordechai Shacham (1998), POLYMATH verssion 4.1 user-friendly numerical analysis programs.  http://www.polymath-software.com.

[26] George B. Dantzig, Discrete-Variable Extremum Problems. Operations Research, Vol. 5, No. 2 (Apr., 1957), pp. 266-277.

[27] Pintu Das, Tapan Kumar Roy (2014). Multi-objective geometric programming and its application in gravel box problem. Journal of global research in computer science volume 5. no.7, July 2014. http://www.jgrcs.info

[28] Kalyanmoy Deb, Amrit Pratap, Subrajyoti Moitra (2000). Mechanical component design for multi objectives using elitist non-dominated sorting GA. Proceedings of the Parallel Probl.Solv. Nat. PPSN VI Conference, Paris, France, pp. 859-868 (2000). (Please see pp. 1-10 in Technical Report No. 200002 via Google search.)

[29] Kusum Deep, Krishna Pratap Singh, M. L. Kansal, C. Mohan (2009), A real coded genetic algorithm for solving integer and mixed integer optimization problems. Applied Mathematics and Computation 212 (2009) 505-518.

[30] Wassila Drici, Mustapha Moulai (2019): An exact method for solving multi-objective integer indefinite quadratic programs, Optimization Methods and Software.

[31] R. J. Duffin, E. L. Peterson, C. Zener (1967), Geometric Programming. John Wiley, New York (1967).

[32]  Simge Ekiz, Pakize Erdogmus, Busra Ozgur (2017 November), Solving constrained  optimization problems with sine-cosine algorithm, Periodicals of Engineering and Natural Sciences, vol. 5, no. 3, November 2017, pp.378~386.  http://pen.ius.edu.ba.  

[33] C. A. Floudas, A. R. Ciric (1989), Strategies for Overcoming Uncertainties in Heat Exchanger Network Synthesis. Computers and Chemical Engineering, Vol 13, No. 10, pp. 1133-1152, 1989.

[34] C. A. Floudas, A. Aggarwal, A. R. Ciric (1989), Global Optimum Search for Nonconvex NLP and MINLP Problems. Computers and Chemical Engineering, Vol 13, No. 10, pp. 1117-1132, 1989.

[35] C. A. Floudas, A. Aggarwal, A. R. Ciric (1989), Global Optimum Search for Nonconvex NLP and MINLP Problems. Computers and Chemical Engineering, Vol 13, No. 10, pp. 1117-1132, 1989.

[36] C. A. Floudas, P. M. Pardalos, A Collection of Test Problems for Constrained Global Optimization Algorithms. Springer-Verlag, 1990.  

[37] C. A. Floudas(1995),  Nonlinear and mixed-integer optimization:  fundamentals and applications.  Oxford University Press, 1995.

[38] Diptesh Ghosh, Ravi Kothari, Population Heuristics for the Corridor Allocation Problem, W.P. No. 2012-09-02, September 2012. Retrieved on September 14 2012 from Google search.

[39]  Crina Grosan, Ajith Abraham  (2008),    A New Approach for Solving Nonlinear Equations Systems.  IEEE Transactions on Systems, Man and Cybernetics–Part A:  Systems and Humans, Volume 38, Number 3, May 2008, pp. 698-714.

[40]  Crina Grosan, Ajith Abraham  (2008), Multiple Solutions for a System pf Nonlinear Equations. International Journal of Innovative Computing, Information and Control, Volume x, Number x, x 2008.

[41]  GUO Chuang-xin,  HU Jia-sheng, YE Bin, CAO Yi-jia (2004), Swarm intelligence for mixed-variable design optimization, J. of Zhejiang University SCIENCE 2004 5(7):851-860.  One can directly read this on Google.

[42] Neha Gupta, Irfan Ali, Abdul Bari (2013). An optimal chance constraint multivariate stratified sampling design using auxiliary infotmation. Journal of Mathematical Modelling and Algorithms, January 2013.

[43] R. Gupta, R. Malhotra (1995). Multi-criteria integer linear fractional programming problem, Optimization, 35:4, 373-389.

[44] M. Hashish, M. P. duPlessis (1979). Prediction equations relating high velocity jet cutting performance to stand-off-distance and multipasses. Transactions of ASME: Journal of Engineering for Industry 101 (1979) 311-318.

[45]  Abdel-Rahman Hedar, Masao Fukushima (2006)  Derivative-free simulated annealing method for constrained continuous optimization, J. of Global Optimization, January 2006.   (One can directly view this on Google.)

[46]  www – optima.amp.i.kyoto – u.ac.jp/member/student/hedar/Hedar_fil…

[47]  Frederick S. Hillier, Gerald J. Liberman (1990), Introduction to Mathematical Programming, McGraw-Hill Publishing Company, 1990.

[48] Willi Hock, Klaus Schittkowski, Test Examples for Nonlinear Programming Codes. Berlin: Springer-Verlag, 1981.

[49] Sana Iftekhar, M. J. Ahsan, Qazi Mazhar Ali (2015). An optimum multivariate stratified sampling design. Research Journal of Mathematical and Statistical Sciences, vol. 3(1),10-14, January (2015).

[50] Philipp Hungerlaender (April 2012), Single-Row Equidistant Facility Layout as a Special Case of Single-Row Facility Layout. Retrieved from http://www.optimization-online.org./DB_HTML/2012/04/3432.html

[51] Golam Reza Jahanshahloo, Farhad Hosseinzadeh, Nagi Shoja, Ghasem Tohidi (2003) A method for solvong 0-1 multiple objective liner programming problem using DEA. Journal of the Operations Research Society of Japan (2003), 46 (2): 189-202. http://www.orsj.or.jp/~archive/pdf/e_mag/Vol.46_02_189.pdf

[52] Ekta Jain, Kalpana Dahiya, Vanita Verma (2018): Integer quadratic fractional programming problems with bounded variables, Annals of Operations Research (October 2018) 269: pp. 269-295.

[53] N. K. Jain, V. K. Jain, K. Deb (2007). Optimization of process parameters of mechanical type advanced machining processes using genetic algorithms. International Journal of Machine Tools and Manufacture 47 (2007), 900-919.

[54] Michael Junger, Thomas M. Liebling, Dennis Naddef, George L. Nemhauser, William R. Pulleybank, Gerhart Reinelt, Giovanni Rinaldi, Lawrence A. Wolsey–Editors, 50 Years of Integer Programming 1958-2008. Berlin: Springer, 2010.

[55] Adhe Kania, Kuntjoro Adji Sidarto (2016). Solving mixed integer nonlinear programming problems using spiral dynamics optimization algorithm. AIP Conference Proceedings 1716, 020004 (2016).  29, 2016.  https://doi:org/10.1063/1.4942987.

[56] Reena Kapoor, S. R. Arora (2006). Linearization of a 0-1 quadratic fractional programming problem: OPSEARCH of the Operational Research Society of India (2006), 43(2):190-207.

[57] M. G. M. Khan, E. A. Khan, M. J. Ahsan (2003). An optimal multivariate stratified sampling design using dynami programming. Australian and New Zealand Journal of Statistics, vol. 45, no. 1, 2003, pp. 107-113.

[58] M. G. M. Khan, T. Maiti, M. J. Ahsan (2010). An optimal multivariate stratified sampling design using auxiliary information: an integer solution using goal programming approach. Journal of Official Statistics, vol. 26, no. 4, 2010, pp. 695-708.

[59] A. H. Land, A. G. Doig, An Automatic Method of Solving Discrete Programming Problems. Econometrica, Vol. 28, No. 3 (Jul., 1960), pp. 497-520.

[60]  E. L. Lawler, M. D. Bell, A Method for Solving Discrete Optimization Problems. Operations Research, Vol. 14, No. 6 (Nov.-Dec., 1966), pp. 1098-1112.

[61]  Sven Leyffer (December 1993)  Deterministic Methods for Mixed Integer Nonlinear Progrmming, PhD Thesis, Department of Mathematics and Computer Science, University of Dundee, Dundee, December 1993.  (One can directly read this on Google.)

[62]  S. Leyffer, J. Linderoth, J. Luedtke, A. Miller, T. Munson (2009), Applications and algorithms for mixed integer nonlinear programming, J. of Physics: Conference Series 180 (2009) 012014, IOP Publishing.  (One can directly read this on Google.)

[63]  Yugui Li, Yanxu Wei, Yantao Chu (2015), Research on solving systems of nonlinear equations based on improved PSO, Mathematical Problems in Engineering, volume 2015, article ID 727218, 13 pages, Hindawi Publishing Corporration.   One can directly read this article on Google.

[64]  F. H. F. Liu, C. C. Huang, Y. L. Yen (2000): Using DEA to obtain efficient solutions for multi-objective 0-1 linear programs. European Journal of Operational Research 126 (2000) 51-68.

[65] Gia-Shi Liu (2006), A combination method for reliability-redundancy optimization, Engineering Optimization, 38:04, 485-499.

[66] Yubao Liu, Guihe Qin (2014), A hybrid TS-DE algorithm for reliability redundancy optimization problem, Journal of Computers, 9, No. 9, September 2014, pp. 2050-2057.

[67] Rein Luus (1975). Optimization of System Reliability by a New Nonlinear Integer Programming Procedure. IEEE Transactions on Reliability, Vol. R-24, No. 1, April 1975, pp. 14-16.

[68] Milos Madic, Miroslav Radovanovic (2014). Optimization of machining processes using pattern search algorithm. International Journal of Industrial Engineering Computations 5 (2014) 223-234. Homepage: http://www.GrowingScience.com/ijiec

[69] F. Masedu, M Angelozzi (2008). Modelling optimum fraction assignment in the 4X100 m relay race by integer linear programming. Italian Journal of Sports Sciences, Anno 13, No. 1, 2008, pp. 74-77.

[70] Mohamed Arezki Mellal, Enrico Zio (2016). A Guided Stochastic Fractal Search Approach for System Reliability Optimization. Reliability Engineering and System Safety 152 (2016) 213-227.

[71] Mohamed Arezki Mellal, Edward J. Williams (2016). Parameter optimization of advanced machining processes using cuckoo optimization algorithm and hoopla heuristic. Journal of Intelligent Manufacturing (2016) 27 (5): 927-942.

[72] Mohamed Arezki Mellal, Edward J. Williams (2018). Large-scale reliability-redundancy allocation optimization problem using three soft computing methods. In Mangey Ram, Editor, in Modeling and simulation based analysis in reliability engineering. Published July 2018, CRC Press.

[73] Microsoft Corp., BASIC, Second Edition (May 1982), Version 1.10. Boca Raton, Florida: IBM Corp., Personal Computer, P. O. Box 1328-C, Boca Raton, Florida 33432, 1981.

[74]  Joaquin Moreno, Miguel Lopez, Raquel Martinez (2018)  A new method for solving  nonlinear systems of equations that is based on functional iterations, Open Physics 2018; 16: 605-630.    

[75] Yuji Nakagawa, Mitsunori Hikita, Hiroshi Kamada (1984). Surrogate Constraints for Reliability Optimization Problems with Multiple Constraints. IEEE Transactions on Reliability, Vol. R-33, No. 4, October 1984, pp. 301-305.

[76] Subhash C. Narula, H. Roland Weistroffer (1989). A flexible method for nonlinear criteria decisionmaking problems. Ieee Transactions on Systems, Man and Cybernetics, vol. 19 , no. 4, July/August 1989, pp. 883-887.

[77] C. E. Nugent, T. E. Vollmann, J. Ruml (1968), An Experimental Comparison of Techniques for the Assignment of Facilities to Locations, Operations Research 16 (1968), pp. 150-173.

[78] A. K. Ojha, K. K. Biswal (2010). Multi-objective geometric programming problem with weighted mean method. (IJCSIS) International Journal of Computer Science and Information Security, vol. 7, no. 2, pp. 82-86, 2010.

[79] Rashmi Ranjan Ota, jayanta kumar dASH, A. K. Ojha (2014). A hybrid optimization techniques for solving non-linear multi-objective optimization problem. amo-advanced modelling and optimization, volume 16,number 1, 2014.

[80] Rashmi Ranjan Ota, A. K. Ojha (2015). A comparative study on optimization techniques for solving multi-objective geometric programming problems. Applied mathematical sciences, vol. 9, 2015, no. 22, 1077-1085. http://sites.google.com/site/ijcsis/

[81] R. R. Ota, J. C. Pati, A. K. Ojha (2019). Geometric programming technique to optimize power distribution system. OPSEARCH of the Operational Research Society of India (2019), 56, pp. 282-299.

[82] OPTI Toolbox, Mixed Integer Nonlinear Program (MINLP). https://www.inverseproblem.co.nz/OPTI/index.php/Probs/MINLP

[83] Panos Y. Papalambros, Douglass J. Wilde, Principles of Optimal Design, Second Edition. Cambridge University Press, 2000.

[84]  O. Perez, I. Amaya, R. Correa (2013), Numerical solution of certain exponential and nonlinear diophantine systems of equations by using a discrete particles swarm optimization algorithm.  Applied Mathematics and Computation, Volume 225, 1 December, 2013, pp. 737-746.

[85] Ciara Pike-Burke. Multi-Objective Optimization. https://www.lancaster.ac.uk/pg/pikeburc/report1.pdf.

[86] Yashpal Singh Raghav, Irfan Ali, Abdul Bari (2014) Multi-Objective Nonlinear Programming Problem Approach in Multivariate Stratified Sample Surveys in the Case of the Non-Response, Journal of Statitiscal Computation ans Simulation 84:1, 22-36, doi:10.1080/00949655.2012.692370.

[87] R. V. Rao, P. J. Pawar, J. P. Davim (2010). Parameter optimization of ultrasonic machining process using nontraditional optimization algorihms. Materials and Manufacturing Processes, 25 (10),1120-1130.

[88]  W. Rheinboldt,  Some Nonlinear Testproblems.  One can google this 10-page paper.

[89]  John Rice, Numerical Methods, Software, and Analysis, Second Edition, 1993, Academic Press.

[90] H. S. Ryoo, N. V. Sahinidis (1995), Global Optimization of Nonconvex NLPs and MINLPs with Applications in Process Design. Computers and Chemical Engineering, Vol. 19, No. 5, pp. 551-566, 1995.

[91] Ali Sadollah, Hadi Eskandar, Joong Hoon Kim (2015). Water cycle algorithm for solvinfg constrained multi-objective optimization problems. Applied Soft Computing 27 (2015) 279-298.

[92] Shafiullah, Irfan Ali, Abdul Bari (2015). Fuzzy geometric programming approach in multi-objective multivariate stratified sample surveys in presence of non-respnse, International Journal of Operations Research, Vol. 12, No. 2, pp. 021-035 (2015).

[93] Mordechai Shacham (1986). Numerical solution of constrained nonlinear algebraic equations, International Journal for Numerical Methods in Engineering, Vol. 23, pp. 1455-1481 (1986).  (One can read this via Google.)

[94] M. Shacham,  R. S. H. Mah  (1978).  A Newton Type Linearization Method for Solution of Nonlinear Equations, Computers and Chemical Engineering, Vol. 2, pp. 64-66 (1978).  (One can read this via Google.)

[95]  Yi Shang (1997),  Global search methods for solving nonlinear optimization problems, Ph. D. Thesis, University of Illinois at Urbana-Champaign, 1997.

[96] Vikas Sharma (2012). Multiobjective integer nonlinear fractional programming problem: A cutting plane approach, OPSEARCH of the Operational Research Society of India (April-June 2012), 49(2):133-153.

[97] Vikas Sharma, Kalpana Dahiya, Vanita Verma (2017). A ranking algorithm for bi-objective quadratic fractional integer programming problems, Optimization, 66:11, 1913-1929.

[98]  K. A. Sidarto, Adhe Kania (2015),  Finding all solutions of systems of nonlinear equations using spiral dynamics inspired optimization with clustering,

Journal of Advanced Computational Intelligence and Intelligent Informatics, pp. 697-708, vol. 19, number 5, 2015.  (One can directly read this via Google.)

[99] Donald M. Simmons (1969), One-Dimensional Space Allocation: An Ordering Algorithm. Operations Research, Vol. 17, No. 5 (Sep. – Oct., 1969), pp. 812-826.

[100] G. Stephanopoulos, A. W. Westerberg, The Use of Hestenes’ Method of Multipliers to Resolve Dual Gaps in Engineering System Optimization. Journal of Optimization Theory and Applications, Vol.15, No. 3, pp. 285-309, 1975.

[101]  Sonja Surjanovic, Derek Bingham (2013), Virtual library of simulation experiments: test functions and datasets.  Simon Fraser University, Burnaby, BC, Canada, 2013.  (One can directly read this on Google.)

[102] Hardi Tambunan, Herman Mawengkang (2016). Solving Mixed Integer Non-Linear Programming Using Active Constraint. Global Journal of Pure and Applied Mathematics, Volume 12, Number 6 (2016), pp. 5267-5281. http://www.ripublication.com/gjpam.htm

[103] Mohamed Tawhid, Vimal Savsani (2018). Epsilon-constraint heat transfer search (epsilon-HTS) algorithm for solvinfg multi-objective engineering design problems. journal of computational design and engineering 5 (2018) 104-119.

[104] Frank A. Tillman, Ching-Lai Hwang, Way Kuo (1977). Determining Component Reliability and Redundancy for Optimun System Reliability. IEEE Transactions on Reliability, Vol. R-26, No. 3, Augusr 1977, pp. 162-165.

[105] Rahul Varshney, Srikant Gupta, Irfan Ali (2017). An optimum multivariate-multiobjective stratified samplinr design: fuzzy programming approach. Pakistan Journal of Statistics and Operations Research, pp. 829-855, December 2017

[106] V. Verma, H. C. Bakhshi, M. C. Puri (1990) Ranking in integer linear fractional programming problems ZOR – Methods and Models of Operations Research (1990) 34:325-334.

[107] Tawan Wasanapradit, Nalinee Mukdasanit, Nachol Chaiyaratana, Thongchai Srinophakun (2011). Solving mixed-integer nonlinear programming problems using improved genetic algorithms. Korean Joutnal of Chemical Engineering 28 (1):32-40 January 2011.

[108] Rahul Varshney, Mradula (2019 May 25). Optimum allocation in multivariate stratified sampling design in the presence of nonresponse with Gamma cost function, Journal of Statistical Computation and Simulation (2019) 89:13, pp. 2454-2467.

[109] Rahul Varshney, Najmussehar, M. J. Ahsan (2012). An optimum multivariate stratified double sampling design in presence of non-response, Optimization Letters (2012) 6: pp. 993-1008.

[110] Rahul Varshney, M. G. M. Khan, Ummatul Fatima, M. J. Ahsan (2015). Integer compromise allocation in multivariate stratified surveys, Annals of Operations Research (2015) 226:659-668.

[111] Rahul Varshney, Srikant Gupta, Irfan Ali (2017). An optimum multivariate-multiobjective stratified samplinr design: fuzzy programming approach. Pakistan Journal of Statistics and Operations Research, pp. 829-855, December 2017

[112] V. Verma, H. C. Bakhshi, M. C. Puri (1990) Ranking in integer linear fractional programming problems ZOR – Methods and Models of Operations Research (1990) 34:325-334.

[113] Tawan Wasanapradit, Nalinee Mukdasanit, Nachol Chaiyaratana, Thongchai Srinophakun (2011). Solving mixed-integer nonlinear programming problems using improved genetic algorithms. Korean Joutnal of Chemical Engineering 28 (1):32-40 January 2011.

[114]  Eric W. Weisstein, “Diophantine Equation–8th Powers.”  https://mathworld.wolfram.com/DiophantineEquation8thPowers.html.

[115]  Eric W. Weisstein, “Euler’s Sum of 0wers Conjecture.”  https://mathworld.wolfram.com/EulersSumofPowersConjecture.html.

[116]  Eric W. Weisstein, “Diophantine Equation–5th Powers.”  https://mathworld.wolfram.com/DiophantineEquation5thPowers.html.

[117]  Eric W. Weisstein, “Diophantine Equation–10th Powers.”  https://mathworld.wolfram.com/DiophantineEquation10thPowers.html.

[118]  Eric W. Weisstein, “Diophantine Equation–9th Powers.”  https://mathworld.wolfram.com/DiophantineEquation9thPowers.html.

[119]  Rick Wicklin (2018), Solving a system of nonlinear equations with SAS.  blogs.sas.com>iml>2018/02/28.  One can directly read this on Google.

[120] Wikipedia, QB64, https://en.wikipedia.org/wiki/QB64.

[121] Wikipedia, Test functions for optimization, https://en.wikipedia.org/wiki/Test_functions_for_optimization.

[122] Jsun Yui Wong (2014, March 13).   The Domino Method Applied to a System of Four Simultaneous Nonlinear Equations.  Retrieved from http://myblogsubstance.typepad.com/substance/2014/03.

[123] Zhongkai Xu, Way Kuo, Hsin-Hui Lin (1990). Optimization Limits in Improving System Reliability. IEEE Transactions on Reliability, Vol. 39, No. 1, 1990 April, pp. 51-60.

Working on the Simionescu Test  Function for Constrained Optimization       

    

Jsun Yui Wong

The computer program listed below seeks to solve the following test problem in the Wikipedia [121]:

minimize                .1 * X(1) * X(2)

subject  to  

 X(1) ^ 2 + X(2) ^ 2 <= (1 + .2 * COS(8 * arctan X(1) / X(2))) ^ 2

where -1.25<= X(1), X(2)     <=1.25.

0 REM      DEFDBL A-Z

2 REM    DEFINT K

3 DIM B(99), N(99), A(100255), H(99), L(99), U(99), X(100250), D(111), P(511), PS(33), J(30003), J44(30003), KKQQ(30003), KLX(30003), W(10111)

12 FOR JJJJ = -32000 TO 32000 STEP .01

    13 RANDOMIZE JJJJ

    16 M = -1D+37

    18 FOR J44 = 1 TO 2

        21 A(J44) = -1 + RND * 2

    25 NEXT J44

    128 FOR I = 1 TO 100000

        129 FOR KKQQ = 1 TO 2

            130 X(KKQQ) = A(KKQQ)

        131 NEXT KKQQ

        135 FOR IPP = 1 TO (1 + FIX(RND ^ 5 * 5))

            148 J = 1 + FIX(RND * 2)

            153 REM    GOTO 162

            154 IF RND < .5 THEN GOTO 156 ELSE GOTO 162

            156 REM

            157 R = (1 – RND * 2) * (A(J))

            160 X(J) = A(J) + (RND ^ (RND * 30)) * R

            161 GOTO 165

            162 REM    IF RND < .16666 THEN X(J) = A(J) – FIX(1 + RND * 2.3) ELSE IF RND < .33333 THEN X(J) = A(J) + FIX(1 + RND * 2.3) ELSE IF RND < .5 THEN X(J) = A(J) – FIX(1 + RND * 20.3) ELSE IF RND < .66666 THEN X(J) = A(J) + FIX(1 + RND * 20.3) ELSE IF RND < .83333 THEN X(J) = A(J) – FIX(1 + RND * 200.3) ELSE X(J) = A(J) + FIX(1 + RND * 200.3)

            164 IF RND < .5 THEN X(J) = A(J) – FIX(1 + RND * .3) ELSE X(J) = A(J) + FIX(1 + RND * .3)

        165 NEXT IPP

        168 REM

        195 FOR J44 = 1 TO 2

            197 IF X(J44) < -1.25 THEN 1670

            198 IF X(J44) > 1.25 THEN 1670

        204 NEXT J44

        222 IF X(1) ^ 2 + X(2) ^ 2 – (1 + .2 * COS(8 * arctanX(1) / X(2))) ^ 2 > 0 THEN 1670

        956 P = -.1 * X(1) * X(2)

        1111 IF P <= M THEN 1670

        1452 M = P

        1454 FOR KLX = 1 TO 2

            1459 A(KLX) = X(KLX)

        1460 NEXT KLX

        1557 GOTO 128

    1670 NEXT I

    1677 IF M < -9999 THEN 1999

    1908 PRINT A(1), A(2), M, JJJJ

1999 NEXT JJJJ

This BASIC computer program was run with qb64v1000-win [120].  The complete output of one run through JJJJ =  -31999.97 is shown below:

.8564162        -.840566            7.198744E-02          -32000

-.8392708        .8576855          7.198304E-02         -31999.99

.8179137        -.8780758          7.181902E-02        -31999.98

-.8720713        .8243128          7.188596E-02        -31999.97

Above there is no rounding by hand; it is just straight copying by hand from the monitor screen.

The system properties of the computer system used include Intel Core 2 Duo CPU   E8400 @ 3.00GHz  3.00GHz, 4.00GB of RAM, and qb64v1000-win [120].  The wall-clock time (not CPU time) for obtaining the output through JJJJ = -31999.97 was 4 seconds, counting from “Starting program…”.  One can compare the computational results above with those in the

Wikipedia [121].  

Acknowledgement

I would like to acknowledge the encouragement of Roberta Clark and Tom Clark.

References

[1]  M. Abdollahi, A. Isazadeh, D. Abdollahi (2013), Imperialist competitive algorithm for solving systems of nonlinear equations, Computers and Mathematics with Applications, vol. 65, number 12, pp. 1894-1908, 2013.  (One can directly read this on Google.)

[2] Siby Abraham, Sugata Sanyal, Mukund Sanglikar (2010), Particle Swarm Optimisation Based Diophantine Equation Solver,  Int. J. of Bio-Inspired Computation, 2 (2), 100-114, 2010. 

[3] Siby Abraham, Sugata Sanyal, Mukund Sangrikar (2013), A Connectionist Network Approach to Find Numerical Solutions of Diophantine Equations, Int. J. of Engg. Science and Mgmt., Vol. III, Issue 1, January-June 2013.

[4] Andre R. S. Amaral (2006), On the Exact Solution of a Facility Layout Problem. European Journal of Operational Research 173 (2006), pp. 508-518.

[5] Andre R. S. Amaral (2008), An Exact Approach to the One-Dimensional Facility Layout Problem. Operations Research, Vol. 56, No. 4 (July-August, 2008), pp. 1026-1033.

[6] Andre R. S. Amaral (2011), Optimal Solutions for the Double Row Layout Problem. Optimization Letters, DOI 10.1007/s11590-011-0426-8, published on line 30 November 2011, Springer-Verlag 2011.

[7] Andre R. S. Amaral (2012), The Corridor Allocation Problem. Computers and Operations Research 39 (2012), pp. 3325-3330.

[8]  George Anescu (2016), Gradual and cumulative improvements to the classical differential evolution scheme through experiments, Analele  Universitatii de Vest, Timisoara, Seria Mathematica–Informatica, Vol. LIV, 2, (2016), 13-35.   (One can directly read this on Google.)

[9] Oscar Augusto, Bennis Fouad, Stephane Caro (2012). A new method for decision making in multi-objective optimization problems. Pesquisa Operacional, Sociedade Brasileira de Pesquisa Operacional, 2012 32 (3), pp.331-369.

[10] Miguel F. Anjos, Anthony Vannelli, Computing Globally Optimal Solutions for Single-Row Layout Problems Using Semidefinite Programming and Cutting Planes. INFORMS Journal on Computing, Vol. 20, No. 4, Fall 2008, pp. 611-617.

[11] David L. Applegate, Robert E. Bixby, Vasek Chvatal, William J. Cook, The Traveling Salesman Problem: A Computational Study. Princeton and Oxford: Princeton University Press, 2006.

[12] Ritu Arora, S. R. Arora (2015). A cutting plane approach for multi-objective integer indefinite quadratic programming problem: OPSEARCH of the Operational Research Society of India (April-June 2015), 52(2):367-381.

[13]   H. Bernau (1990).  Active constraint strategies in optimization.  Geographical data inversion methods and applications, pp. 15-31.

[14] Victor Blanco, Justo Puerto (2011). Some algebraic methods for solving multiobjective polynomial integer programs, Journal of Symbolic Computation, 46 (2011), pp. 511-533.

[15] Jerome Bracken, Garth P. McCormick, Selected Applications of Nonlinear Programming. New York: John Wiley and Sons, Inc., 1968.

[16] Neima Brauner, Moredechai Shacham, Michael Cutlip (winter 1996)  Computational Results – How Reliable Are They? Chemical Engineering Educatiobn, 30, 1,  pp. 20-25 (1996).  (One can read this via Google.)

[17] Richard L. Burden, Douglas J. Faires, Annette M. Burden, Numerical Analysis, Tenth Edition, 2016, Cengage Learning.

[18] R. C. Carlson and G. L. Nemhauser, Scheduling To Minimize Interaction Cost. Operations Research, Vol. 14, No. 1 (Jan. – Feb., 1966), pp. 52-58.

[19] Ta-Cheng Chen (2006). IAs based approach for reliability redundany allocation problems. Applied Mathematics and Computation 182 (2006) 1556-1567.

[20]  Ward Cheney, David Kincaid (2008), Numerical Mathematics and Computing, Sixth Edition, 2008, Thomsom Brooks/Coles.

[21] Leandro dos Santos Coelho (2009), Self-Organizing Migrating Strategies Applied to Reliability-Redundany Optimization of Systems. IEEE Transactions on Reliability, Vol. 58, No. 3, 2009 September, pp. 501-519.

[22]  A. J. Conejo, L. Baringo (2018), Power System Operations, Springer International Publishing, 2018.

[23] William Conley (1981). Optimization: A Simplified Approach. Published 1981 by Petrocelli Books in New York.

[24] Lino Costa, Pedro (2001). Evolutionary algorithms approach to the solution of mixed integer non-linear programming problems. Computers and Chemical Engineering, Vol. 25, pp. 257-266, 2001.

[25] Michael B. Cutlip, Mordechai Shacham (1998), POLYMATH verssion 4.1 user-friendly numerical analysis programs.  http://www.polymath-software.com.

[26] George B. Dantzig, Discrete-Variable Extremum Problems. Operations Research, Vol. 5, No. 2 (Apr., 1957), pp. 266-277.

[27] Pintu Das, Tapan Kumar Roy (2014). Multi-objective geometric programming and its application in gravel box problem. Journal of global research in computer science volume 5. no.7, July 2014. http://www.jgrcs.info

[28] Kalyanmoy Deb, Amrit Pratap, Subrajyoti Moitra (2000). Mechanical component design for multi objectives using elitist non-dominated sorting GA. Proceedings of the Parallel Probl.Solv. Nat. PPSN VI Conference, Paris, France, pp. 859-868 (2000). (Please see pp. 1-10 in Technical Report No. 200002 via Google search.)

[29] Kusum Deep, Krishna Pratap Singh, M. L. Kansal, C. Mohan (2009), A real coded genetic algorithm for solving integer and mixed integer optimization problems. Applied Mathematics and Computation 212 (2009) 505-518.

[30] Wassila Drici, Mustapha Moulai (2019): An exact method for solving multi-objective integer indefinite quadratic programs, Optimization Methods and Software.

[31] R. J. Duffin, E. L. Peterson, C. Zener (1967), Geometric Programming. John Wiley, New York (1967).

[32]  Simge Ekiz, Pakize Erdogmus, Busra Ozgur (2017 November), Solving constrained  optimization problems with sine-cosine algorithm, Periodicals of Engineering and Natural Sciences, vol. 5, no. 3, November 2017, pp.378~386.  http://pen.ius.edu.ba.  

[33] C. A. Floudas, A. R. Ciric (1989), Strategies for Overcoming Uncertainties in Heat Exchanger Network Synthesis. Computers and Chemical Engineering, Vol 13, No. 10, pp. 1133-1152, 1989.

[34] C. A. Floudas, A. Aggarwal, A. R. Ciric (1989), Global Optimum Search for Nonconvex NLP and MINLP Problems. Computers and Chemical Engineering, Vol 13, No. 10, pp. 1117-1132, 1989.

[35] C. A. Floudas, A. Aggarwal, A. R. Ciric (1989), Global Optimum Search for Nonconvex NLP and MINLP Problems. Computers and Chemical Engineering, Vol 13, No. 10, pp. 1117-1132, 1989.

[36] C. A. Floudas, P. M. Pardalos, A Collection of Test Problems for Constrained Global Optimization Algorithms. Springer-Verlag, 1990.  

[37] C. A. Floudas(1995),  Nonlinear and mixed-integer optimization:  fundamentals and applications.  Oxford University Press, 1995.

[38] Diptesh Ghosh, Ravi Kothari, Population Heuristics for the Corridor Allocation Problem, W.P. No. 2012-09-02, September 2012. Retrieved on September 14 2012 from Google search.

[39]  Crina Grosan, Ajith Abraham  (2008),    A New Approach for Solving Nonlinear Equations Systems.  IEEE Transactions on Systems, Man and Cybernetics–Part A:  Systems and Humans, Volume 38, Number 3, May 2008, pp. 698-714.

[40]  Crina Grosan, Ajith Abraham  (2008), Multiple Solutions for a System pf Nonlinear Equations. International Journal of Innovative Computing, Information and Control, Volume x, Number x, x 2008.

[41]  GUO Chuang-xin,  HU Jia-sheng, YE Bin, CAO Yi-jia (2004), Swarm intelligence for mixed-variable design optimization, J. of Zhejiang University SCIENCE 2004 5(7):851-860.  One can directly read this on Google.

[42] Neha Gupta, Irfan Ali, Abdul Bari (2013). An optimal chance constraint multivariate stratified sampling design using auxiliary infotmation. Journal of Mathematical Modelling and Algorithms, January 2013.

[43] R. Gupta, R. Malhotra (1995). Multi-criteria integer linear fractional programming problem, Optimization, 35:4, 373-389.

[44] M. Hashish, M. P. duPlessis (1979). Prediction equations relating high velocity jet cutting performance to stand-off-distance and multipasses. Transactions of ASME: Journal of Engineering for Industry 101 (1979) 311-318.

[45]  Abdel-Rahman Hedar, Masao Fukushima (2006)  Derivative-free simulated annealing method for constrained continuous optimization, J. of Global Optimization, January 2006.   (One can directly view this on Google.)

[46]  www – optima.amp.i.kyoto – u.ac.jp/member/student/hedar/Hedar_fil…

[47]  Frederick S. Hillier, Gerald J. Liberman (1990), Introduction to Mathematical Programming, McGraw-Hill Publishing Company, 1990.

[48] Willi Hock, Klaus Schittkowski, Test Examples for Nonlinear Programming Codes. Berlin: Springer-Verlag, 1981.

[49] Sana Iftekhar, M. J. Ahsan, Qazi Mazhar Ali (2015). An optimum multivariate stratified sampling design. Research Journal of Mathematical and Statistical Sciences, vol. 3(1),10-14, January (2015).

[50] Philipp Hungerlaender (April 2012), Single-Row Equidistant Facility Layout as a Special Case of Single-Row Facility Layout. Retrieved from http://www.optimization-online.org./DB_HTML/2012/04/3432.html

[51] Golam Reza Jahanshahloo, Farhad Hosseinzadeh, Nagi Shoja, Ghasem Tohidi (2003) A method for solvong 0-1 multiple objective liner programming problem using DEA. Journal of the Operations Research Society of Japan (2003), 46 (2): 189-202. http://www.orsj.or.jp/~archive/pdf/e_mag/Vol.46_02_189.pdf

[52] Ekta Jain, Kalpana Dahiya, Vanita Verma (2018): Integer quadratic fractional programming problems with bounded variables, Annals of Operations Research (October 2018) 269: pp. 269-295.

[53] N. K. Jain, V. K. Jain, K. Deb (2007). Optimization of process parameters of mechanical type advanced machining processes using genetic algorithms. International Journal of Machine Tools and Manufacture 47 (2007), 900-919.

[54] Michael Junger, Thomas M. Liebling, Dennis Naddef, George L. Nemhauser, William R. Pulleybank, Gerhart Reinelt, Giovanni Rinaldi, Lawrence A. Wolsey–Editors, 50 Years of Integer Programming 1958-2008. Berlin: Springer, 2010.

[55] Adhe Kania, Kuntjoro Adji Sidarto (2016). Solving mixed integer nonlinear programming problems using spiral dynamics optimization algorithm. AIP Conference Proceedings 1716, 020004 (2016).  29, 2016.  https://doi:org/10.1063/1.4942987.

[56] Reena Kapoor, S. R. Arora (2006). Linearization of a 0-1 quadratic fractional programming problem: OPSEARCH of the Operational Research Society of India (2006), 43(2):190-207.

[57] M. G. M. Khan, E. A. Khan, M. J. Ahsan (2003). An optimal multivariate stratified sampling design using dynami programming. Australian and New Zealand Journal of Statistics, vol. 45, no. 1, 2003, pp. 107-113.

[58] M. G. M. Khan, T. Maiti, M. J. Ahsan (2010). An optimal multivariate stratified sampling design using auxiliary information: an integer solution using goal programming approach. Journal of Official Statistics, vol. 26, no. 4, 2010, pp. 695-708.

[59] A. H. Land, A. G. Doig, An Automatic Method of Solving Discrete Programming Problems. Econometrica, Vol. 28, No. 3 (Jul., 1960), pp. 497-520.

[60]  E. L. Lawler, M. D. Bell, A Method for Solving Discrete Optimization Problems. Operations Research, Vol. 14, No. 6 (Nov.-Dec., 1966), pp. 1098-1112.

[61]  Sven Leyffer (December 1993)  Deterministic Methods for Mixed Integer Nonlinear Progrmming, PhD Thesis, Department of Mathematics and Computer Science, University of Dundee, Dundee, December 1993.  (One can directly read this on Google.)

[62]  S. Leyffer, J. Linderoth, J. Luedtke, A. Miller, T. Munson (2009), Applications and algorithms for mixed integer nonlinear programming, J. of Physics: Conference Series 180 (2009) 012014, IOP Publishing.  (One can directly read this on Google.)

[63]  Yugui Li, Yanxu Wei, Yantao Chu (2015), Research on solving systems of nonlinear equations based on improved PSO, Mathematical Problems in Engineering, volume 2015, article ID 727218, 13 pages, Hindawi Publishing Corporration.   One can directly read this article on Google.

[64]  F. H. F. Liu, C. C. Huang, Y. L. Yen (2000): Using DEA to obtain efficient solutions for multi-objective 0-1 linear programs. European Journal of Operational Research 126 (2000) 51-68.

[65] Gia-Shi Liu (2006), A combination method for reliability-redundancy optimization, Engineering Optimization, 38:04, 485-499.

[66] Yubao Liu, Guihe Qin (2014), A hybrid TS-DE algorithm for reliability redundancy optimization problem, Journal of Computers, 9, No. 9, September 2014, pp. 2050-2057.

[67] Rein Luus (1975). Optimization of System Reliability by a New Nonlinear Integer Programming Procedure. IEEE Transactions on Reliability, Vol. R-24, No. 1, April 1975, pp. 14-16.

[68] Milos Madic, Miroslav Radovanovic (2014). Optimization of machining processes using pattern search algorithm. International Journal of Industrial Engineering Computations 5 (2014) 223-234. Homepage: http://www.GrowingScience.com/ijiec

[69] F. Masedu, M Angelozzi (2008). Modelling optimum fraction assignment in the 4X100 m relay race by integer linear programming. Italian Journal of Sports Sciences, Anno 13, No. 1, 2008, pp. 74-77.

[70] Mohamed Arezki Mellal, Enrico Zio (2016). A Guided Stochastic Fractal Search Approach for System Reliability Optimization. Reliability Engineering and System Safety 152 (2016) 213-227.

[71] Mohamed Arezki Mellal, Edward J. Williams (2016). Parameter optimization of advanced machining processes using cuckoo optimization algorithm and hoopla heuristic. Journal of Intelligent Manufacturing (2016) 27 (5): 927-942.

[72] Mohamed Arezki Mellal, Edward J. Williams (2018). Large-scale reliability-redundancy allocation optimization problem using three soft computing methods. In Mangey Ram, Editor, in Modeling and simulation based analysis in reliability engineering. Published July 2018, CRC Press.

[73] Microsoft Corp., BASIC, Second Edition (May 1982), Version 1.10. Boca Raton, Florida: IBM Corp., Personal Computer, P. O. Box 1328-C, Boca Raton, Florida 33432, 1981.

[74]  Joaquin Moreno, Miguel Lopez, Raquel Martinez (2018)  A new method for solving  nonlinear systems of equations that is based on functional iterations, Open Physics 2018; 16: 605-630.    

[75] Yuji Nakagawa, Mitsunori Hikita, Hiroshi Kamada (1984). Surrogate Constraints for Reliability Optimization Problems with Multiple Constraints. IEEE Transactions on Reliability, Vol. R-33, No. 4, October 1984, pp. 301-305.

[76] Subhash C. Narula, H. Roland Weistroffer (1989). A flexible method for nonlinear criteria decisionmaking problems. Ieee Transactions on Systems, Man and Cybernetics, vol. 19 , no. 4, July/August 1989, pp. 883-887.

[77] C. E. Nugent, T. E. Vollmann, J. Ruml (1968), An Experimental Comparison of Techniques for the Assignment of Facilities to Locations, Operations Research 16 (1968), pp. 150-173.

[78] A. K. Ojha, K. K. Biswal (2010). Multi-objective geometric programming problem with weighted mean method. (IJCSIS) International Journal of Computer Science and Information Security, vol. 7, no. 2, pp. 82-86, 2010.

[79] Rashmi Ranjan Ota, jayanta kumar dASH, A. K. Ojha (2014). A hybrid optimization techniques for solving non-linear multi-objective optimization problem. amo-advanced modelling and optimization, volume 16,number 1, 2014.

[80] Rashmi Ranjan Ota, A. K. Ojha (2015). A comparative study on optimization techniques for solving multi-objective geometric programming problems. Applied mathematical sciences, vol. 9, 2015, no. 22, 1077-1085. http://sites.google.com/site/ijcsis/

[81] R. R. Ota, J. C. Pati, A. K. Ojha (2019). Geometric programming technique to optimize power distribution system. OPSEARCH of the Operational Research Society of India (2019), 56, pp. 282-299.

[82] OPTI Toolbox, Mixed Integer Nonlinear Program (MINLP). https://www.inverseproblem.co.nz/OPTI/index.php/Probs/MINLP

[83] Panos Y. Papalambros, Douglass J. Wilde, Principles of Optimal Design, Second Edition. Cambridge University Press, 2000.

[84]  O. Perez, I. Amaya, R. Correa (2013), Numerical solution of certain exponential and nonlinear diophantine systems of equations by using a discrete particles swarm optimization algorithm.  Applied Mathematics and Computation, Volume 225, 1 December, 2013, pp. 737-746.

[85] Ciara Pike-Burke. Multi-Objective Optimization. https://www.lancaster.ac.uk/pg/pikeburc/report1.pdf.

[86] Yashpal Singh Raghav, Irfan Ali, Abdul Bari (2014) Multi-Objective Nonlinear Programming Problem Approach in Multivariate Stratified Sample Surveys in the Case of the Non-Response, Journal of Statitiscal Computation ans Simulation 84:1, 22-36, doi:10.1080/00949655.2012.692370.

[87] R. V. Rao, P. J. Pawar, J. P. Davim (2010). Parameter optimization of ultrasonic machining process using nontraditional optimization algorihms. Materials and Manufacturing Processes, 25 (10),1120-1130.

[88]  W. Rheinboldt,  Some Nonlinear Testproblems.  One can google this 10-page paper.

[89]  John Rice, Numerical Methods, Software, and Analysis, Second Edition, 1993, Academic Press.

[90] H. S. Ryoo, N. V. Sahinidis (1995), Global Optimization of Nonconvex NLPs and MINLPs with Applications in Process Design. Computers and Chemical Engineering, Vol. 19, No. 5, pp. 551-566, 1995.

[91] Ali Sadollah, Hadi Eskandar, Joong Hoon Kim (2015). Water cycle algorithm for solvinfg constrained multi-objective optimization problems. Applied Soft Computing 27 (2015) 279-298.

[92] Shafiullah, Irfan Ali, Abdul Bari (2015). Fuzzy geometric programming approach in multi-objective multivariate stratified sample surveys in presence of non-respnse, International Journal of Operations Research, Vol. 12, No. 2, pp. 021-035 (2015).

[93] Mordechai Shacham (1986). Numerical solution of constrained nonlinear algebraic equations, International Journal for Numerical Methods in Engineering, Vol. 23, pp. 1455-1481 (1986).  (One can read this via Google.)

[94] M. Shacham,  R. S. H. Mah  (1978).  A Newton Type Linearization Method for Solution of Nonlinear Equations, Computers and Chemical Engineering, Vol. 2, pp. 64-66 (1978).  (One can read this via Google.)

[95]  Yi Shang (1997),  Global search methods for solving nonlinear optimization problems, Ph. D. Thesis, University of Illinois at Urbana-Champaign, 1997.

[96] Vikas Sharma (2012). Multiobjective integer nonlinear fractional programming problem: A cutting plane approach, OPSEARCH of the Operational Research Society of India (April-June 2012), 49(2):133-153.

[97] Vikas Sharma, Kalpana Dahiya, Vanita Verma (2017). A ranking algorithm for bi-objective quadratic fractional integer programming problems, Optimization, 66:11, 1913-1929.

[98]  K. A. Sidarto, Adhe Kania (2015),  Finding all solutions of systems of nonlinear equations using spiral dynamics inspired optimization with clustering,

Journal of Advanced Computational Intelligence and Intelligent Informatics, pp. 697-708, vol. 19, number 5, 2015.  (One can directly read this via Google.)

[99] Donald M. Simmons (1969), One-Dimensional Space Allocation: An Ordering Algorithm. Operations Research, Vol. 17, No. 5 (Sep. – Oct., 1969), pp. 812-826.

[100] G. Stephanopoulos, A. W. Westerberg, The Use of Hestenes’ Method of Multipliers to Resolve Dual Gaps in Engineering System Optimization. Journal of Optimization Theory and Applications, Vol.15, No. 3, pp. 285-309, 1975.

[101]  Sonja Surjanovic, Derek Bingham (2013), Virtual library of simulation experiments: test functions and datasets.  Simon Fraser University, Burnaby, BC, Canada, 2013.  (One can directly read this on Google.)

[102] Hardi Tambunan, Herman Mawengkang (2016). Solving Mixed Integer Non-Linear Programming Using Active Constraint. Global Journal of Pure and Applied Mathematics, Volume 12, Number 6 (2016), pp. 5267-5281. http://www.ripublication.com/gjpam.htm

[103] Mohamed Tawhid, Vimal Savsani (2018). Epsilon-constraint heat transfer search (epsilon-HTS) algorithm for solvinfg multi-objective engineering design problems. journal of computational design and engineering 5 (2018) 104-119.

[104] Frank A. Tillman, Ching-Lai Hwang, Way Kuo (1977). Determining Component Reliability and Redundancy for Optimun System Reliability. IEEE Transactions on Reliability, Vol. R-26, No. 3, Augusr 1977, pp. 162-165.

[105] Rahul Varshney, Srikant Gupta, Irfan Ali (2017). An optimum multivariate-multiobjective stratified samplinr design: fuzzy programming approach. Pakistan Journal of Statistics and Operations Research, pp. 829-855, December 2017

[106] V. Verma, H. C. Bakhshi, M. C. Puri (1990) Ranking in integer linear fractional programming problems ZOR – Methods and Models of Operations Research (1990) 34:325-334.

[107] Tawan Wasanapradit, Nalinee Mukdasanit, Nachol Chaiyaratana, Thongchai Srinophakun (2011). Solving mixed-integer nonlinear programming problems using improved genetic algorithms. Korean Joutnal of Chemical Engineering 28 (1):32-40 January 2011.

[108] Rahul Varshney, Mradula (2019 May 25). Optimum allocation in multivariate stratified sampling design in the presence of nonresponse with Gamma cost function, Journal of Statistical Computation and Simulation (2019) 89:13, pp. 2454-2467.

[109] Rahul Varshney, Najmussehar, M. J. Ahsan (2012). An optimum multivariate stratified double sampling design in presence of non-response, Optimization Letters (2012) 6: pp. 993-1008.

[110] Rahul Varshney, M. G. M. Khan, Ummatul Fatima, M. J. Ahsan (2015). Integer compromise allocation in multivariate stratified surveys, Annals of Operations Research (2015) 226:659-668.

[111] Rahul Varshney, Srikant Gupta, Irfan Ali (2017). An optimum multivariate-multiobjective stratified samplinr design: fuzzy programming approach. Pakistan Journal of Statistics and Operations Research, pp. 829-855, December 2017

[112] V. Verma, H. C. Bakhshi, M. C. Puri (1990) Ranking in integer linear fractional programming problems ZOR – Methods and Models of Operations Research (1990) 34:325-334.

[113] Tawan Wasanapradit, Nalinee Mukdasanit, Nachol Chaiyaratana, Thongchai Srinophakun (2011). Solving mixed-integer nonlinear programming problems using improved genetic algorithms. Korean Joutnal of Chemical Engineering 28 (1):32-40 January 2011.

[114]  Eric W. Weisstein, “Diophantine Equation–8th Powers.”  https://mathworld.wolfram.com/DiophantineEquation8thPowers.html.

[115]  Eric W. Weisstein, “Euler’s Sum of 0wers Conjecture.”  https://mathworld.wolfram.com/EulersSumofPowersConjecture.html.

[116]  Eric W. Weisstein, “Diophantine Equation–5th Powers.”  https://mathworld.wolfram.com/DiophantineEquation5thPowers.html.

[117]  Eric W. Weisstein, “Diophantine Equation–10th Powers.”  https://mathworld.wolfram.com/DiophantineEquation10thPowers.html.

[118]  Eric W. Weisstein, “Diophantine Equation–9th Powers.”  https://mathworld.wolfram.com/DiophantineEquation9thPowers.html.

[119]  Rick Wicklin (2018), Solving a system of nonlinear equations with SAS.  blogs.sas.com>iml>2018/02/28.  One can directly read this on Google.

[120] Wikipedia, QB64, https://en.wikipedia.org/wiki/QB64.

[121] Wikipedia, Test functions for optimization, https://en.wikipedia.org/wiki/Test_functions_for_optimization.

[122] Jsun Yui Wong (2014, March 13).   The Domino Method Applied to a System of Four Simultaneous Nonlinear Equations.  Retrieved from http://myblogsubstance.typepad.com/substance/2014/03.

[123] Zhongkai Xu, Way Kuo, Hsin-Hui Lin (1990). Optimization Limits in Improving System Reliability. IEEE Transactions on Reliability, Vol. 39, No. 1, 1990 April, pp. 51-60.

Working on Global Optimization Test Problem G11

Jsun Yui Wong

The computer program listed below seeks to solve the following test problem G11 in Hedar and Fukushima [45, Problem G11, p. 23].    

minimize         X(1) ^ 2 + (X(2) – 1) ^ 2

subject  to   X(2) = X(1) ^ 2=0

-1<= x(i)     <=1, i=1, 2,

0 REM      DEFDBL A-Z

2 REM    DEFINT K

3 DIM B(99), N(99), A(100255), H(99), L(99), U(99), X(100250), D(111), P(511), PS(33), J(30003), J44(30003), KKQQ(30003), KLX(30003), W(10111)

12 FOR JJJJ = -32000 TO 32000 STEP .01

    13 RANDOMIZE JJJJ

    16 M = -1D+37

    18 FOR J44 = 1 TO 2

        21 A(J44) = -1 + RND * 2

    25 NEXT J44

    128 FOR I = 1 TO 100000

        129 FOR KKQQ = 1 TO 2

            130 X(KKQQ) = A(KKQQ)

        131 NEXT KKQQ

        135 FOR IPP = 1 TO (1 + FIX(RND ^ 5 * 5))

            148 J = 1 + FIX(RND * 2)

            153 REM    GOTO 162

            154 IF RND < .5 THEN GOTO 156 ELSE GOTO 162

            156 REM

            157 R = (1 – RND * 2) * (A(J))

            160 X(J) = A(J) + (RND ^ (RND * 30)) * R

            161 GOTO 165

            162 REM    IF RND < .16666 THEN X(J) = A(J) – FIX(1 + RND * 2.3) ELSE IF RND < .33333 THEN X(J) = A(J) + FIX(1 + RND * 2.3) ELSE IF RND < .5 THEN X(J) = A(J) – FIX(1 + RND * 20.3) ELSE IF RND < .66666 THEN X(J) = A(J) + FIX(1 + RND * 20.3) ELSE IF RND < .83333 THEN X(J) = A(J) – FIX(1 + RND * 200.3) ELSE X(J) = A(J) + FIX(1 + RND * 200.3)

            164 IF RND < .5 THEN X(J) = A(J) – FIX(1 + RND * .3) ELSE X(J) = A(J) + FIX(1 + RND * .3)

        165 NEXT IPP

        168 X(2) = X(1) ^ 2

        195 FOR J44 = 1 TO 2

            197 IF X(J44) < -1 THEN 1670

            198 IF X(J44) > 1 THEN 1670

        204 NEXT J44

        944 P = -X(1) ^ 2 – (X(2) – 1) ^ 2

        1111 IF P <= M THEN 1670

        1452 M = P

        1454 FOR KLX = 1 TO 2

            1459 A(KLX) = X(KLX)

        1460 NEXT KLX

        1557 GOTO 128

    1670 NEXT I

    1677 IF M < -.76 THEN 1999

    1908 PRINT A(1), A(2), M, JJJJ

1999 NEXT JJJJ

This BASIC computer program was run with qb64v1000-win [120].  The complete output of one run through JJJJ =  -31999.96 is shown below:

-.7070217      .4998796        -.75      -32000

.7071044      .4999966         -.75      -31999.99

.7071526      .5000648         -.75      -31999.98

.7070019      .4998516         -.75      -31999.97

-.707117       .5000144        -.75      -31999.96

Two distinct solutions are shown above.

Above there is no rounding by hand; it is just straight copying by hand from the monitor screen.

The system properties of the computer system used include Intel Core 2 Duo CPU   E8400 @ 3.00GHz  3.00GHz, 4.00GB of RAM, and qb64v1000-win [120].  The wall-clock time (not CPU time) for obtaining the output through JJJJ = -31999.96 was 3 seconds, counting from “Starting program…”.  One can compare the computational results above with those in Hedar and Fukushima [45, Table 2, p. 15; 45, p. 23].          

Acknowledgement

I would like to acknowledge the encouragement of Roberta Clark and Tom Clark.

References

[1]  M. Abdollahi, A. Isazadeh, D. Abdollahi (2013), Imperialist competitive algorithm for solving systems of nonlinear equations, Computers and Mathematics with Applications, vol. 65, number 12, pp. 1894-1908, 2013.  (One can directly read this on Google.)

[2] Siby Abraham, Sugata Sanyal, Mukund Sanglikar (2010), Particle Swarm Optimisation Based Diophantine Equation Solver,  Int. J. of Bio-Inspired Computation, 2 (2), 100-114, 2010. 

[3] Siby Abraham, Sugata Sanyal, Mukund Sangrikar (2013), A Connectionist Network Approach to Find Numerical Solutions of Diophantine Equations, Int. J. of Engg. Science and Mgmt., Vol. III, Issue 1, January-June 2013.

[4] Andre R. S. Amaral (2006), On the Exact Solution of a Facility Layout Problem. European Journal of Operational Research 173 (2006), pp. 508-518.

[5] Andre R. S. Amaral (2008), An Exact Approach to the One-Dimensional Facility Layout Problem. Operations Research, Vol. 56, No. 4 (July-August, 2008), pp. 1026-1033.

[6] Andre R. S. Amaral (2011), Optimal Solutions for the Double Row Layout Problem. Optimization Letters, DOI 10.1007/s11590-011-0426-8, published on line 30 November 2011, Springer-Verlag 2011.

[7] Andre R. S. Amaral (2012), The Corridor Allocation Problem. Computers and Operations Research 39 (2012), pp. 3325-3330.

[8]  George Anescu (2016), Gradual and cumulative improvements to the classical differential evolution scheme through experiments, Analele  Universitatii de Vest, Timisoara, Seria Mathematica–Informatica, Vol. LIV, 2, (2016), 13-35.   (One can directly read this on Google.)

[9] Oscar Augusto, Bennis Fouad, Stephane Caro (2012). A new method for decision making in multi-objective optimization problems. Pesquisa Operacional, Sociedade Brasileira de Pesquisa Operacional, 2012 32 (3), pp.331-369.

[10] Miguel F. Anjos, Anthony Vannelli, Computing Globally Optimal Solutions for Single-Row Layout Problems Using Semidefinite Programming and Cutting Planes. INFORMS Journal on Computing, Vol. 20, No. 4, Fall 2008, pp. 611-617.

[11] David L. Applegate, Robert E. Bixby, Vasek Chvatal, William J. Cook, The Traveling Salesman Problem: A Computational Study. Princeton and Oxford: Princeton University Press, 2006.

[12] Ritu Arora, S. R. Arora (2015). A cutting plane approach for multi-objective integer indefinite quadratic programming problem: OPSEARCH of the Operational Research Society of India (April-June 2015), 52(2):367-381.

[13]   H. Bernau (1990).  Active constraint strategies in optimization.  Geographical data inversion methods and applications, pp. 15-31.

[14] Victor Blanco, Justo Puerto (2011). Some algebraic methods for solving multiobjective polynomial integer programs, Journal of Symbolic Computation, 46 (2011), pp. 511-533.

[15] Jerome Bracken, Garth P. McCormick, Selected Applications of Nonlinear Programming. New York: John Wiley and Sons, Inc., 1968.

[16] Neima Brauner, Moredechai Shacham, Michael Cutlip (winter 1996)  Computational Results – How Reliable Are They? Chemical Engineering Educatiobn, 30, 1,  pp. 20-25 (1996).  (One can read this via Google.)

[17] Richard L. Burden, Douglas J. Faires, Annette M. Burden, Numerical Analysis, Tenth Edition, 2016, Cengage Learning.

[18] R. C. Carlson and G. L. Nemhauser, Scheduling To Minimize Interaction Cost. Operations Research, Vol. 14, No. 1 (Jan. – Feb., 1966), pp. 52-58.

[19] Ta-Cheng Chen (2006). IAs based approach for reliability redundany allocation problems. Applied Mathematics and Computation 182 (2006) 1556-1567.

[20]  Ward Cheney, David Kincaid (2008), Numerical Mathematics and Computing, Sixth Edition, 2008, Thomsom Brooks/Coles.

[21] Leandro dos Santos Coelho (2009), Self-Organizing Migrating Strategies Applied to Reliability-Redundany Optimization of Systems. IEEE Transactions on Reliability, Vol. 58, No. 3, 2009 September, pp. 501-519.

[22]  A. J. Conejo, L. Baringo (2018), Power System Operations, Springer International Publishing, 2018.

[23] William Conley (1981). Optimization: A Simplified Approach. Published 1981 by Petrocelli Books in New York.

[24] Lino Costa, Pedro (2001). Evolutionary algorithms approach to the solution of mixed integer non-linear programming problems. Computers and Chemical Engineering, Vol. 25, pp. 257-266, 2001.

[25] Michael B. Cutlip, Mordechai Shacham (1998), POLYMATH verssion 4.1 user-friendly numerical analysis programs.  http://www.polymath-software.com.

[26] George B. Dantzig, Discrete-Variable Extremum Problems. Operations Research, Vol. 5, No. 2 (Apr., 1957), pp. 266-277.

[27] Pintu Das, Tapan Kumar Roy (2014). Multi-objective geometric programming and its application in gravel box problem. Journal of global research in computer science volume 5. no.7, July 2014. http://www.jgrcs.info

[28] Kalyanmoy Deb, Amrit Pratap, Subrajyoti Moitra (2000). Mechanical component design for multi objectives using elitist non-dominated sorting GA. Proceedings of the Parallel Probl.Solv. Nat. PPSN VI Conference, Paris, France, pp. 859-868 (2000). (Please see pp. 1-10 in Technical Report No. 200002 via Google search.)

[29] Kusum Deep, Krishna Pratap Singh, M. L. Kansal, C. Mohan (2009), A real coded genetic algorithm for solving integer and mixed integer optimization problems. Applied Mathematics and Computation 212 (2009) 505-518.

[30] Wassila Drici, Mustapha Moulai (2019): An exact method for solving multi-objective integer indefinite quadratic programs, Optimization Methods and Software.

[31] R. J. Duffin, E. L. Peterson, C. Zener (1967), Geometric Programming. John Wiley, New York (1967).

[32]  Simge Ekiz, Pakize Erdogmus, Busra Ozgur (2017 November), Solving constrained  optimization problems with sine-cosine algorithm, Periodicals of Engineering and Natural Sciences, vol. 5, no. 3, November 2017, pp.378~386.  http://pen.ius.edu.ba.  

[33] C. A. Floudas, A. R. Ciric (1989), Strategies for Overcoming Uncertainties in Heat Exchanger Network Synthesis. Computers and Chemical Engineering, Vol 13, No. 10, pp. 1133-1152, 1989.

[34] C. A. Floudas, A. Aggarwal, A. R. Ciric (1989), Global Optimum Search for Nonconvex NLP and MINLP Problems. Computers and Chemical Engineering, Vol 13, No. 10, pp. 1117-1132, 1989.

[35] C. A. Floudas, A. Aggarwal, A. R. Ciric (1989), Global Optimum Search for Nonconvex NLP and MINLP Problems. Computers and Chemical Engineering, Vol 13, No. 10, pp. 1117-1132, 1989.

[36] C. A. Floudas, P. M. Pardalos, A Collection of Test Problems for Constrained Global Optimization Algorithms. Springer-Verlag, 1990.  

[37] C. A. Floudas(1995),  Nonlinear and mixed-integer optimization:  fundamentals and applications.  Oxford University Press, 1995.

[38] Diptesh Ghosh, Ravi Kothari, Population Heuristics for the Corridor Allocation Problem, W.P. No. 2012-09-02, September 2012. Retrieved on September 14 2012 from Google search.

[39]  Crina Grosan, Ajith Abraham  (2008),    A New Approach for Solving Nonlinear Equations Systems.  IEEE Transactions on Systems, Man and Cybernetics–Part A:  Systems and Humans, Volume 38, Number 3, May 2008, pp. 698-714.

[40]  Crina Grosan, Ajith Abraham  (2008), Multiple Solutions for a System pf Nonlinear Equations. International Journal of Innovative Computing, Information and Control, Volume x, Number x, x 2008.

[41]  GUO Chuang-xin,  HU Jia-sheng, YE Bin, CAO Yi-jia (2004), Swarm intelligence for mixed-variable design optimization, J. of Zhejiang University SCIENCE 2004 5(7):851-860.  One can directly read this on Google.

[42] Neha Gupta, Irfan Ali, Abdul Bari (2013). An optimal chance constraint multivariate stratified sampling design using auxiliary infotmation. Journal of Mathematical Modelling and Algorithms, January 2013.

[43] R. Gupta, R. Malhotra (1995). Multi-criteria integer linear fractional programming problem, Optimization, 35:4, 373-389.

[44] M. Hashish, M. P. duPlessis (1979). Prediction equations relating high velocity jet cutting performance to stand-off-distance and multipasses. Transactions of ASME: Journal of Engineering for Industry 101 (1979) 311-318.

[45]  Abdel-Rahman Hedar, Masao Fukushima (2006)  Derivative-free simulated annealing method for constrained continuous optimization, J. of Global Optimization, January 2006.   (One can directly view this on Google.)

[46]  www – optima.amp.i.kyoto – u.ac.jp/member/student/hedar/Hedar_fil…

[47]  Frederick S. Hillier, Gerald J. Liberman (1990), Introduction to Mathematical Programming, McGraw-Hill Publishing Company, 1990.

[48] Willi Hock, Klaus Schittkowski, Test Examples for Nonlinear Programming Codes. Berlin: Springer-Verlag, 1981.

[49] Sana Iftekhar, M. J. Ahsan, Qazi Mazhar Ali (2015). An optimum multivariate stratified sampling design. Research Journal of Mathematical and Statistical Sciences, vol. 3(1),10-14, January (2015).

[50] Philipp Hungerlaender (April 2012), Single-Row Equidistant Facility Layout as a Special Case of Single-Row Facility Layout. Retrieved from http://www.optimization-online.org./DB_HTML/2012/04/3432.html

[51] Golam Reza Jahanshahloo, Farhad Hosseinzadeh, Nagi Shoja, Ghasem Tohidi (2003) A method for solvong 0-1 multiple objective liner programming problem using DEA. Journal of the Operations Research Society of Japan (2003), 46 (2): 189-202. http://www.orsj.or.jp/~archive/pdf/e_mag/Vol.46_02_189.pdf

[52] Ekta Jain, Kalpana Dahiya, Vanita Verma (2018): Integer quadratic fractional programming problems with bounded variables, Annals of Operations Research (October 2018) 269: pp. 269-295.

[53] N. K. Jain, V. K. Jain, K. Deb (2007). Optimization of process parameters of mechanical type advanced machining processes using genetic algorithms. International Journal of Machine Tools and Manufacture 47 (2007), 900-919.

[54] Michael Junger, Thomas M. Liebling, Dennis Naddef, George L. Nemhauser, William R. Pulleybank, Gerhart Reinelt, Giovanni Rinaldi, Lawrence A. Wolsey–Editors, 50 Years of Integer Programming 1958-2008. Berlin: Springer, 2010.

[55] Adhe Kania, Kuntjoro Adji Sidarto (2016). Solving mixed integer nonlinear programming problems using spiral dynamics optimization algorithm. AIP Conference Proceedings 1716, 020004 (2016).  29, 2016.  https://doi:org/10.1063/1.4942987.

[56] Reena Kapoor, S. R. Arora (2006). Linearization of a 0-1 quadratic fractional programming problem: OPSEARCH of the Operational Research Society of India (2006), 43(2):190-207.

[57] M. G. M. Khan, E. A. Khan, M. J. Ahsan (2003). An optimal multivariate stratified sampling design using dynami programming. Australian and New Zealand Journal of Statistics, vol. 45, no. 1, 2003, pp. 107-113.

[58] M. G. M. Khan, T. Maiti, M. J. Ahsan (2010). An optimal multivariate stratified sampling design using auxiliary information: an integer solution using goal programming approach. Journal of Official Statistics, vol. 26, no. 4, 2010, pp. 695-708.

[59] A. H. Land, A. G. Doig, An Automatic Method of Solving Discrete Programming Problems. Econometrica, Vol. 28, No. 3 (Jul., 1960), pp. 497-520.

[60]  E. L. Lawler, M. D. Bell, A Method for Solving Discrete Optimization Problems. Operations Research, Vol. 14, No. 6 (Nov.-Dec., 1966), pp. 1098-1112.

[61]  Sven Leyffer (December 1993)  Deterministic Methods for Mixed Integer Nonlinear Progrmming, PhD Thesis, Department of Mathematics and Computer Science, University of Dundee, Dundee, December 1993.  (One can directly read this on Google.)

[62]  S. Leyffer, J. Linderoth, J. Luedtke, A. Miller, T. Munson (2009), Applications and algorithms for mixed integer nonlinear programming, J. of Physics: Conference Series 180 (2009) 012014, IOP Publishing.  (One can directly read this on Google.)

[63]  Yugui Li, Yanxu Wei, Yantao Chu (2015), Research on solving systems of nonlinear equations based on improved PSO, Mathematical Problems in Engineering, volume 2015, article ID 727218, 13 pages, Hindawi Publishing Corporration.   One can directly read this article on Google.

[64]  F. H. F. Liu, C. C. Huang, Y. L. Yen (2000): Using DEA to obtain efficient solutions for multi-objective 0-1 linear programs. European Journal of Operational Research 126 (2000) 51-68.

[65] Gia-Shi Liu (2006), A combination method for reliability-redundancy optimization, Engineering Optimization, 38:04, 485-499.

[66] Yubao Liu, Guihe Qin (2014), A hybrid TS-DE algorithm for reliability redundancy optimization problem, Journal of Computers, 9, No. 9, September 2014, pp. 2050-2057.

[67] Rein Luus (1975). Optimization of System Reliability by a New Nonlinear Integer Programming Procedure. IEEE Transactions on Reliability, Vol. R-24, No. 1, April 1975, pp. 14-16.

[68] Milos Madic, Miroslav Radovanovic (2014). Optimization of machining processes using pattern search algorithm. International Journal of Industrial Engineering Computations 5 (2014) 223-234. Homepage: http://www.GrowingScience.com/ijiec

[69] F. Masedu, M Angelozzi (2008). Modelling optimum fraction assignment in the 4X100 m relay race by integer linear programming. Italian Journal of Sports Sciences, Anno 13, No. 1, 2008, pp. 74-77.

[70] Mohamed Arezki Mellal, Enrico Zio (2016). A Guided Stochastic Fractal Search Approach for System Reliability Optimization. Reliability Engineering and System Safety 152 (2016) 213-227.

[71] Mohamed Arezki Mellal, Edward J. Williams (2016). Parameter optimization of advanced machining processes using cuckoo optimization algorithm and hoopla heuristic. Journal of Intelligent Manufacturing (2016) 27 (5): 927-942.

[72] Mohamed Arezki Mellal, Edward J. Williams (2018). Large-scale reliability-redundancy allocation optimization problem using three soft computing methods. In Mangey Ram, Editor, in Modeling and simulation based analysis in reliability engineering. Published July 2018, CRC Press.

[73] Microsoft Corp., BASIC, Second Edition (May 1982), Version 1.10. Boca Raton, Florida: IBM Corp., Personal Computer, P. O. Box 1328-C, Boca Raton, Florida 33432, 1981.

[74]  Joaquin Moreno, Miguel Lopez, Raquel Martinez (2018)  A new method for solving  nonlinear systems of equations that is based on functional iterations, Open Physics 2018; 16: 605-630.    

[75] Yuji Nakagawa, Mitsunori Hikita, Hiroshi Kamada (1984). Surrogate Constraints for Reliability Optimization Problems with Multiple Constraints. IEEE Transactions on Reliability, Vol. R-33, No. 4, October 1984, pp. 301-305.

[76] Subhash C. Narula, H. Roland Weistroffer (1989). A flexible method for nonlinear criteria decisionmaking problems. Ieee Transactions on Systems, Man and Cybernetics, vol. 19 , no. 4, July/August 1989, pp. 883-887.

[77] C. E. Nugent, T. E. Vollmann, J. Ruml (1968), An Experimental Comparison of Techniques for the Assignment of Facilities to Locations, Operations Research 16 (1968), pp. 150-173.

[78] A. K. Ojha, K. K. Biswal (2010). Multi-objective geometric programming problem with weighted mean method. (IJCSIS) International Journal of Computer Science and Information Security, vol. 7, no. 2, pp. 82-86, 2010.

[79] Rashmi Ranjan Ota, jayanta kumar dASH, A. K. Ojha (2014). A hybrid optimization techniques for solving non-linear multi-objective optimization problem. amo-advanced modelling and optimization, volume 16,number 1, 2014.

[80] Rashmi Ranjan Ota, A. K. Ojha (2015). A comparative study on optimization techniques for solving multi-objective geometric programming problems. Applied mathematical sciences, vol. 9, 2015, no. 22, 1077-1085. http://sites.google.com/site/ijcsis/

[81] R. R. Ota, J. C. Pati, A. K. Ojha (2019). Geometric programming technique to optimize power distribution system. OPSEARCH of the Operational Research Society of India (2019), 56, pp. 282-299.

[82] OPTI Toolbox, Mixed Integer Nonlinear Program (MINLP). https://www.inverseproblem.co.nz/OPTI/index.php/Probs/MINLP

[83] Panos Y. Papalambros, Douglass J. Wilde, Principles of Optimal Design, Second Edition. Cambridge University Press, 2000.

[84]  O. Perez, I. Amaya, R. Correa (2013), Numerical solution of certain exponential and nonlinear diophantine systems of equations by using a discrete particles swarm optimization algorithm.  Applied Mathematics and Computation, Volume 225, 1 December, 2013, pp. 737-746.

[85] Ciara Pike-Burke. Multi-Objective Optimization. https://www.lancaster.ac.uk/pg/pikeburc/report1.pdf.

[86] Yashpal Singh Raghav, Irfan Ali, Abdul Bari (2014) Multi-Objective Nonlinear Programming Problem Approach in Multivariate Stratified Sample Surveys in the Case of the Non-Response, Journal of Statitiscal Computation ans Simulation 84:1, 22-36, doi:10.1080/00949655.2012.692370.

[87] R. V. Rao, P. J. Pawar, J. P. Davim (2010). Parameter optimization of ultrasonic machining process using nontraditional optimization algorihms. Materials and Manufacturing Processes, 25 (10),1120-1130.

[88]  W. Rheinboldt,  Some Nonlinear Testproblems.  One can google this 10-page paper.

[89]  John Rice, Numerical Methods, Software, and Analysis, Second Edition, 1993, Academic Press.

[90] H. S. Ryoo, N. V. Sahinidis (1995), Global Optimization of Nonconvex NLPs and MINLPs with Applications in Process Design. Computers and Chemical Engineering, Vol. 19, No. 5, pp. 551-566, 1995.

[91] Ali Sadollah, Hadi Eskandar, Joong Hoon Kim (2015). Water cycle algorithm for solvinfg constrained multi-objective optimization problems. Applied Soft Computing 27 (2015) 279-298.

[92] Shafiullah, Irfan Ali, Abdul Bari (2015). Fuzzy geometric programming approach in multi-objective multivariate stratified sample surveys in presence of non-respnse, International Journal of Operations Research, Vol. 12, No. 2, pp. 021-035 (2015).

[93] Mordechai Shacham (1986). Numerical solution of constrained nonlinear algebraic equations, International Journal for Numerical Methods in Engineering, Vol. 23, pp. 1455-1481 (1986).  (One can read this via Google.)

[94] M. Shacham,  R. S. H. Mah  (1978).  A Newton Type Linearization Method for Solution of Nonlinear Equations, Computers and Chemical Engineering, Vol. 2, pp. 64-66 (1978).  (One can read this via Google.)

[95]  Yi Shang (1997),  Global search methods for solving nonlinear optimization problems, Ph. D. Thesis, University of Illinois at Urbana-Champaign, 1997.

[96] Vikas Sharma (2012). Multiobjective integer nonlinear fractional programming problem: A cutting plane approach, OPSEARCH of the Operational Research Society of India (April-June 2012), 49(2):133-153.

[97] Vikas Sharma, Kalpana Dahiya, Vanita Verma (2017). A ranking algorithm for bi-objective quadratic fractional integer programming problems, Optimization, 66:11, 1913-1929.

[98]  K. A. Sidarto, Adhe Kania (2015),  Finding all solutions of systems of nonlinear equations using spiral dynamics inspired optimization with clustering,

Journal of Advanced Computational Intelligence and Intelligent Informatics, pp. 697-708, vol. 19, number 5, 2015.  (One can directly read this via Google.)

[99] Donald M. Simmons (1969), One-Dimensional Space Allocation: An Ordering Algorithm. Operations Research, Vol. 17, No. 5 (Sep. – Oct., 1969), pp. 812-826.

[100] G. Stephanopoulos, A. W. Westerberg, The Use of Hestenes’ Method of Multipliers to Resolve Dual Gaps in Engineering System Optimization. Journal of Optimization Theory and Applications, Vol.15, No. 3, pp. 285-309, 1975.

[101]  Sonja Surjanovic, Derek Bingham (2013), Virtual library of simulation experiments: test functions and datasets.  Simon Fraser University, Burnaby, BC, Canada, 2013.  (One can directly read this on Google.)

[102] Hardi Tambunan, Herman Mawengkang (2016). Solving Mixed Integer Non-Linear Programming Using Active Constraint. Global Journal of Pure and Applied Mathematics, Volume 12, Number 6 (2016), pp. 5267-5281. http://www.ripublication.com/gjpam.htm

[103] Mohamed Tawhid, Vimal Savsani (2018). Epsilon-constraint heat transfer search (epsilon-HTS) algorithm for solvinfg multi-objective engineering design problems. journal of computational design and engineering 5 (2018) 104-119.

[104] Frank A. Tillman, Ching-Lai Hwang, Way Kuo (1977). Determining Component Reliability and Redundancy for Optimun System Reliability. IEEE Transactions on Reliability, Vol. R-26, No. 3, Augusr 1977, pp. 162-165.

[105] Rahul Varshney, Srikant Gupta, Irfan Ali (2017). An optimum multivariate-multiobjective stratified samplinr design: fuzzy programming approach. Pakistan Journal of Statistics and Operations Research, pp. 829-855, December 2017

[106] V. Verma, H. C. Bakhshi, M. C. Puri (1990) Ranking in integer linear fractional programming problems ZOR – Methods and Models of Operations Research (1990) 34:325-334.

[107] Tawan Wasanapradit, Nalinee Mukdasanit, Nachol Chaiyaratana, Thongchai Srinophakun (2011). Solving mixed-integer nonlinear programming problems using improved genetic algorithms. Korean Joutnal of Chemical Engineering 28 (1):32-40 January 2011.

[108] Rahul Varshney, Mradula (2019 May 25). Optimum allocation in multivariate stratified sampling design in the presence of nonresponse with Gamma cost function, Journal of Statistical Computation and Simulation (2019) 89:13, pp. 2454-2467.

[109] Rahul Varshney, Najmussehar, M. J. Ahsan (2012). An optimum multivariate stratified double sampling design in presence of non-response, Optimization Letters (2012) 6: pp. 993-1008.

[110] Rahul Varshney, M. G. M. Khan, Ummatul Fatima, M. J. Ahsan (2015). Integer compromise allocation in multivariate stratified surveys, Annals of Operations Research (2015) 226:659-668.

[111] Rahul Varshney, Srikant Gupta, Irfan Ali (2017). An optimum multivariate-multiobjective stratified samplinr design: fuzzy programming approach. Pakistan Journal of Statistics and Operations Research, pp. 829-855, December 2017

[112] V. Verma, H. C. Bakhshi, M. C. Puri (1990) Ranking in integer linear fractional programming problems ZOR – Methods and Models of Operations Research (1990) 34:325-334.

[113] Tawan Wasanapradit, Nalinee Mukdasanit, Nachol Chaiyaratana, Thongchai Srinophakun (2011). Solving mixed-integer nonlinear programming problems using improved genetic algorithms. Korean Joutnal of Chemical Engineering 28 (1):32-40 January 2011.

[114]  Eric W. Weisstein, “Diophantine Equation–8th Powers.”  https://mathworld.wolfram.com/DiophantineEquation8thPowers.html.

[115]  Eric W. Weisstein, “Euler’s Sum of 0wers Conjecture.”  https://mathworld.wolfram.com/EulersSumofPowersConjecture.html.

[116]  Eric W. Weisstein, “Diophantine Equation–5th Powers.”  https://mathworld.wolfram.com/DiophantineEquation5thPowers.html.

[117]  Eric W. Weisstein, “Diophantine Equation–10th Powers.”  https://mathworld.wolfram.com/DiophantineEquation10thPowers.html.

[118]  Eric W. Weisstein, “Diophantine Equation–9th Powers.”  https://mathworld.wolfram.com/DiophantineEquation9thPowers.html.

[119]  Rick Wicklin (2018), Solving a system of nonlinear equations with SAS.  blogs.sas.com>iml>2018/02/28 can directly read this on Google.

[120] Wikipedia, QB64, https://en.wikipedia.org/wiki/QB64.

[121] Jsun Yui Wong (2014, March 13).   The Domino Method Applied to a System of Four Simultaneous Nonlinear Equations.  Retrieved from http://myblogsubstance.typepad.com/substance/2014/03.

[122] Zhongkai Xu, Way Kuo, Hsin-Hui Lin (1990). Optimization Limits in Improving System Reliability. IEEE Transactions on Reliability, Vol. 39, No. 1, 1990 April, pp. 51-60.